Sunteți pe pagina 1din 173

DEPARTAMENTUL

MATEMATICĂ-INFORMATICĂ
UNIVERSITATEA DIN PITES, TI

MATINF
PUBLICAT, IE BIANUALĂ
DE MATEMATICĂ S, I INFORMATICĂ
PENTRU ELEVI S, I PROFESORI

Anul I, nr. 1
AUGUST 2018

ISSN 2601-9426
ISSN-L 2601-8829

Editura
Universităt, ii din Pites, ti
Editată de: DEPARTAMENTUL MATEMATICĂ-INFORMATICĂ,
UNIVERSITATEA DIN PITES, TI
Comitetul de redact, ie:

Stelian Corneliu ANDRONESCU Eduard ASADURIAN


Tudor BĂLĂNESCU Costel BĂLCĂU - redactor s, ef
Loredana BĂLILESCU Doru CONSTANTIN
Şerban COSTEA Laurent, iu DEACONU
Ionut, DINCĂ Mihaela DUMITRACHE
Mihai Armand IONESCU Florentin IPATE
Constantin GEORGESCU Raluca Mihaela GEORGESCU
Camelia GHELDIU Marius MACARIE
Maria MIROIU Emil MOLDOVEANU
Gheorghe NISTOR Antonio Mihail NUICĂ
Viorel PĂUN Doru Anastasiu POPESCU
Marin POPESCU Doru Nicolae STĂNESCU
Alina Florentina S, TEFAN Cristina TUDOSE
Adrian ŢURCANU Corneliu UDREA

Tehnoredactare computerizată: Mihail TĂNASE, e-mail: mihaimit@yahoo.it

Redact, ia: Departamentul Matematică-Informatică, Universitatea din Pites, ti., Str. Târgu
din Vale, nr. 1, Pites, ti, tel. 0348453247, e-mail: revista.matinf@upit.ro
Forma digitală a revistei poate fi accesată la adresa: http://matinf.upit.ro

Publicată de: Editura Universităt, ii din Pites, ti, https://www.upit.ro/ro/relatii-cu-mediul-


socio-economic/centre-suport/editura

Anul I, Nr. 1, August 2018


Cuprins 3

Cuprins

DIN ACTIVITATEA DEPARTAMENTULUI 5

D. Constantin
Prezentarea Departamentului Matematică-Informatică . . . . . . . . . . . . . . . . . . . 5

D.A. Popescu, C. Bălcău, D. Constantin


Prezentarea Concursului de Informatică ,,Programming Day for High School”, Edit, ia I,
Pites, ti, 27 mai 2017 . . . . . . . . . . . . . . . . . . . . . . . . . . . . . . . . . . . . . . 10

ARTICOLE S, I NOTE DE MATEMATICĂ 21

V. Alexandru, S.C. Andronescu


Numere prime de forma p = x2 + M y 2 cu x, y ∈ Z s, i M ∈ {1, 2, 3} . . . . . . . . . . . . . 21

M. Chirciu
În legătură cu o inegalitate din ,,Mathematical Reflections” nr. 6/2015 . . . . . . . . . . 25

L.M. Giugiuc, D. Trăilescu Gogan


Coordonate analitice practice . . . . . . . . . . . . . . . . . . . . . . . . . . . . . . . . . 27

M. Ionescu
Asupra unei probleme propuse la Olimpiada Nat, ională de Matematică . . . . . . . . . . 31

F. Stănescu
Probleme de calcul integral. Inegalităt, i integrale I . . . . . . . . . . . . . . . . . . . . . . 33

ARTICOLE S, I NOTE DE INFORMATICĂ 39

C. Bălcău
Optimalitatea algoritmului de căutare binară . . . . . . . . . . . . . . . . . . . . . . . . 39

ROWEB Development
Perspective profesionale pentru pasionat, ii de programare . . . . . . . . . . . . . . . . . . 52

MATEMATICĂ PENTRU PROGRAMATORI S, I PROGRAMARE PENTRU MATE-


MATICIENI 55

L. Deaconu
Rezolvarea sistemelor de ecuat, ii liniare . . . . . . . . . . . . . . . . . . . . . . . . . . . . 55

R.M. Georgescu
Determinarea pozit, iilor relative a mai multor drepte ı̂n plan cu ajutorul mediului Maple . 65

RUBRICA DE ROBOTICĂ 70

D.A. Popescu
Programarea robot, ilor LEGO folosind mediul Mindstorms EV3 . . . . . . . . . . . . . . 70
4 Cuprins

PROBLEME DE MATEMATICĂ PENTRU EXAMENE 79


Teste pentru examenul de Evaluare Nat, ională . . . . . . . . . . . . . . . . . . . . . . . 79
Teste pentru examenul de Bacalaureat, specializarea S, tiint, e ale naturii . . . . . . . . . . 86
Teste pentru examenul de Bacalaureat, specializarea Matematică-Informatică . . . . . . 91
Teste pentru admiterea la facultate . . . . . . . . . . . . . . . . . . . . . . . . . . . . . 97
Teste grilă pentru admiterea la facultate . . . . . . . . . . . . . . . . . . . . . . . . . . 100

PROBLEME DE INFORMATICĂ PENTRU EXAMENE 110


Teste pentru examenul de Bacalaureat, specializarea S, tiint, e ale naturii . . . . . . . . . . 110
Teste pentru examenul de Bacalaureat, specializarea Matematică-Informatică . . . . . . 120
Teste pentru admiterea la facultate . . . . . . . . . . . . . . . . . . . . . . . . . . . . . 135
Teste grilă pentru admiterea la facultate . . . . . . . . . . . . . . . . . . . . . . . . . . 136

PROBLEME DE MATEMATICĂ PENTRU CONCURSURI 143


Probleme propuse . . . . . . . . . . . . . . . . . . . . . . . . . . . . . . . . . . . . . . . 143

PROBLEME DE INFORMATICĂ PENTRU CONCURSURI 150


Probleme propuse . . . . . . . . . . . . . . . . . . . . . . . . . . . . . . . . . . . . . . . 150

ISTORIOARE DIN LUMEA MATEMATICII S, I A INFORMATICII 172

S.C. Andronescu
Formula de alungare a lui L. Euler . . . . . . . . . . . . . . . . . . . . . . . . . . . . . . 172
DIN ACTIVITATEA DEPARTAMENTULUI

Prezentarea Departamentului Matematică-Informatică

1
Doru Constantin

Învăt, ământul superior din Pites, ti a debutat ı̂n anul 1962 prin ı̂nfiint, area Institutului Pedago-
gic din Pites, ti. Din anul 1974 Institutul Pedagogic s-a transformat ı̂n Institutul de Învăt, ământ
Superior Pites, ti, iar ı̂n cadrul acestuia a funct, ionat specializarea Matematică.
Universitatea din Pites, ti, ca institut, ie de ı̂nvăt, ământ superior cu statut juridic propriu, a fost
ı̂nfiint, ată prin ordinul Ministerului Învăt, ământului nr. 4844 / 22.03.1991. În cadrul Facultăt, ii
de S, tiint, e au funct, ionat specializările Matematică s, i Matematică-Informatică, iar din anul 2002
s-a acreditat programul de studii universitare de licent, ă Informatică.
Facultatea de Matematică-Informatică a fost ı̂nfiint, ată ı̂n 2002, iar din 2016 ı̂n urma procesului
de reorganizare administrativă a Universităt, ii din Pites, ti, ı̂n conformitate cu H.G. 654/2016, a
format ı̂mpreună cu entităt, ile anterioare reprezentate de Facultatea de S, tiint, e s, i de Facultatea
de Educat, ie Fizică s, i Sport, facultatea actuală denumită Facultatea de S, tiint, e, Educat, ie Fizică
s, i Informatică.
Facultatea de S, tiint, e, Educat, ie Fizică s, i Informatică (F.S, .E.F.I.) este ı̂n prezent alcătuită
din 5 departamente (Matematică-Informatică, S, tiint, e ale Naturii, Ingineria Mediului s, i S, tiint, e
Ingineres, ti Aplicate, Asistent, ă Medicală s, i Kinetoterapie, Educat, ie Fizică s, i Sport) la care
predau peste 130 de cadre didactice titulare s, i asociate.
Facultatea de S, tiint, e, Educat, ie Fizică s, i Informatică organizează, coordonează s, i verifică
activitatea de producere, de transmitere s, i de valorificare a cunoas, terii ı̂n 13 programe de studii
universitare de licent, ă, 10 programe de studii universitare de masterat, 4 programe de studii
universitare de doctorat s, i 4 programe de studii postuniversitare de conversie profesională active
ı̂n anul universitar curent. De asemenea, ı̂n cadrul facultăt, ii sunt acreditate 2 programe de
studii universitare de masterat cu predare ı̂n limbă străină (un program de studii universitare
de masterat ı̂n limba engleză ı̂n domeniul Informatică s, i unul ı̂n limba franceză ı̂n domeniul
Educat, ie Fizică s, i Sport).
Departamentul Matematică-Informatică (D.M.I.) oferă posibilitatea dezvoltării s, tiint, ifice
la nivel superior prin oferta educat, ională ce cuprinde: 2 programe de studii universitare de
licent, ă (ı̂n domeniile Informatică s, i, respectiv, Matematică), 4 programe de studii universitare
de masterat (ı̂n domeniile Informatică s, i, respectiv, Matematică), precum s, i 2 programe de
studii universitare de doctorat (ı̂n domeniile Informatică s, i, respectiv, Matematică).
În Departamentul Matematică-Informatică activităt, ile didactice de specialitate din cadrul
programelor de studii universitare de licent, ă, masterat s, i doctorat ı̂n domeniile Matematică s, i
Informatică sunt acoperite de peste 30 de cadre didactice titulare s, i asociate care au pregătirea
1
Conf. univ. dr., Universitatea din Pites, ti, doru.constantin@upit.ro

5
6 D. Constantin

s, tiint, ifică adecvată atestată prin studiile de licent, ă s, i/sau doctorat ı̂n domeniile disciplinelor
predate.
Programele de studii universitare de licent, ă, masterat s, i doctorat gestionate, ı̂n prezent, de
Departamentul Matematică-Informatică sunt sintetizate ı̂n următorul tabel.
1. Domenii de licent, ă s, i programe de studii universitare de licent, ă
Domeniul de Programul de Statutul Cifra Aracis de
licent, ă studii programului s, colarizare
Informatică Informatică Acreditat 100
Matematică Matematică Acreditat 30
2. Programele de studii universitare de masterat
Statutul Cifra Aracis de
Programul de studii programului s, colarizare
Tehnici avansate pentru prelucrarea
informat, iei (Advanced Techniques for Acreditat 50
Information Processing)
Modelarea, proiectarea si managementul
sistemelor software Acreditat 50
Matematică aplicată Acreditat 50
Matematică didactică Acreditat 50
3. Programele de studii universitare de doctorat
Programul de studii S, coala Doctorală
Informatică S, coala doctorală de Informatică
Matematică S, coala doctorală de Matematică
În concordant, ă cu misiunea Universităt, ii, prezentată ı̂n Carta Universităt, ii din Pites, ti
s, i cu solicitările actuale de pe piat, a fort, ei de muncă regionale s, i nat, ionale, Departamentul
Matematică-Informatică din cadrul Facultatăt, ii de S, tiint, e, Educat, ie Fizică s, i Informatică are
următoarele misiuni didactice s, i de cercetare s, tiint, ifică:

1. asigurarea pentru fiecare program de studiu a unor planuri de ı̂nvăt, ământ s, i programe
analitice adecvate, care să permită recunoas, terea competent, elor obt, inute, atât ı̂n t, ară cât
s, i ı̂n străinătate;
2. pregătirea de specialis, ti de performant, ă care să fie capabili a asigura dezvoltarea societăt, ii
informat, ionale din România ı̂n contextul integrării t, ării noastre ı̂n Comunitatea Europeană;
3. participarea la programe europene de mobilitate a student, ilor s, i a cadrelor didactice;
4. formarea de specialis, ti cu ı̂naltă calificare ı̂n domeniile informatică s, i matematică, la un
nivel compatibil cu cel din UE, cât s, i cadre didactice necesare ı̂nvăt, ământului nat, ional s, i
internat, ional ı̂n domeniile matematică s, i informatică;
5. pregătirea psiho-pedagogică s, i metodică adecvată pentru exercitarea funct, iei de profesor
ı̂n ı̂nvăt, ământul preuniversitar s, i universitar;
6. formarea de specialis, ti ı̂n domeniile matematică s, i informatică, profesori de matematică,
profesori de informatică, matematicieni, informaticieni, etc;
7. asigurarea pregătirii continue ı̂n domeniile matematică s, i informatică;
8. efectuarea de servicii sociale s, i culturale pentru membrii comunităt, ii academice s, i din afara
ei, prin dezvoltarea s, i difuzarea valorilor culturale, s, tiint, ifice, realizarea echilibrului necesar
ı̂ntre cultura s, tiint, ifică, tehnologică s, i umanistă, schimbarea mentalităt, ilor s, i atitudinilor,
promovarea relat, iilor internat, ionale;
9. valorificarea potent, ialului s, tiint, ific pe care-l reprezintă cadrele didactice ale departamen-
Prezentarea Departamentului Matematică-Informatică 7

tului, ı̂n activitatea de cercetare s, tiint, ifică s, i de publicare, prin prezentarea rezultatelor
cercetării s, i a experient, ei s, tiint, ifice acumulate prin participarea la conferint, e nat, ionale s, i
internat, ionale de specialitate.

În acest context, programele de studii universitare din domeniile Matematică s, i Informatică
contribuie la ı̂ndeplinirea misiunii facultăt, ii s, i aduce elemente de specificitate s, i oportunitate,
după cum rezultă din obiectivele generale s, i din obiectivele specifice ale fiecărui program de
studiu.
Obiectivele generale al programelor de studii universitare se pot sintetiza astfel:

1. asigurarea unei pregătiri de ı̂nalt nivel a student, ilor ı̂n domeniile matematică s, i informatică,
astfel ı̂ncât aces, tia să poată ocupa un loc de muncă ı̂n ı̂nvăt, ământ (profesor de matematică,
informatică sau tehnologia informat, iei), ı̂n mediul socio-economic (analist programator,
administrator de ret, ele, proiectant sisteme informatice, statistician etc.) sau să fie capabili
a continua studiile ı̂n cadrul masteratelor din universitatea noastră sau din alte universităt, i
(din t, ară sau din străinătate);
2. formarea de specialis, ti cu ı̂naltă calificare, cu competent, e bine definite ı̂n domeniul
tehnologiilor informat, ionale s, i al sistemelor informatice, pentru mediul de afaceri, ı̂n toate
tipurile de activităt, i economico-sociale, la un nivel compatibil cu cel din UE, cât s, i cadre
necesare ı̂nvăt, ământului;
3. adaptarea ofertei educat, ionale la evolut, ia structurală a mediului economic românesc;
4. integrarea ı̂n ret, ele universitare nat, ionale s, i internat, ionale pe criterii de performant, ă s, i
complementaritate;
5. sust, inerea accesului absolvent, ilor la oportunităt, ile existente pe piat, a muncii nat, ionale s, i
europene;
6. prestarea de activităt, i specifice solicitate de beneficiari ı̂n condit, ii comerciale competitive.

La nivelul Departamentului Matematică-Informatică funct, ionează Centrul de Cercetare


,,Modele de calcul si metodologii de programare” (MCMP). Planul de cercetare al
Centrului de Cercetare include o serie de obiective s, i de activităt, i, structurate pe direct, ii de
cercetare, teme de cercetare s, i colective de cadre didactice.
Obiectivele principale ale planului de cercetare vizează ı̂mbunătăt, irea calităt, ii activităt, ii ı̂n
cercetare prin:
- eficientizarea activităt, ii de cercetare s, tiint, ifică a cadrelor didactice din Departamentul
Matematică-Informatică, prin orientarea activităt, ilor specifice pe direct, iile prioritare stabilite la
nivel nat, ional;
- realizarea unor programe de cercetare cu implicat, ii directe asupra modernizării s, i eficien-
tizării bazei materiale a facultăt, ii, prin participarea la competit, iile de proiecte de cercetare
nat, ionale s, i internat, ionale;
- implicarea unui număr tot mai mare de student, i, masteranzi s, i doctoranzi ı̂n activitatea de
cercetare;
- identificarea s, i realizarea unor noi colaborări cu firme software sau alt, i parteneri din mediul
socio-economic ı̂n vederea valorificării potent, ialului de cercetare;
- realizarea unor colective de cercetare interdisciplinară, ı̂n colaborare cu alte institut, ii de pe
plan nat, ional s, i internat, ional.
8 D. Constantin

Activitatea de cercetare a cadrelor didactice ale Departamentului Matematică-Informatică


se desfăs, oară ı̂n următoarele domenii de cercetare:

1. Aplicat, ii WEB s, i E-learning;


2. Modele de calcul s, i metodologii de programare;
3. Teoria informat, iei s, i sisteme avansate de baze de date;
4. Inteligent, ă artificială s, i modele matematice cu aplicat, ii ı̂n informatică;
5. Recunoas, terea formelor s, i ret, ele neuronale;
6. Verificarea s, i testarea sistemelor de programe, specificarea s, i proiectarea sistemelor software
complexe;
7. Algebră s, i geometrie computat, ională, calcul numeric, optimizări, teoria codurilor s, i
criptografie;
8. Metode numerice ı̂n mecanica fluidelor s, i solidelor, sisteme dinamice, teoria bifurcat, iei,
ecuat, ii diferent, iale;
9. Mecanica mediilor continue, calcul optimal, metode numerice pentru ecuat, ii cu derivate
part, iale, teoria omogenizării;
10. Teoria potent, ialului, procese stocastice, analiză neliniară s, i operatori;
11. Teoria geometrică a funct, iilor univalente.

Anual, Departamentul Matematică-Informatică organizează Sesiunea de comunicări s, tiint, ifice


student, es, ti (SCSS), la care participă un număr mare de student, i. Departamentul Matematică-
Informatică coordonează sect, iunile aferente domeniilor Informatică s, i Matematică astfel:
- Informatică (la care participă ı̂n special student, i de la programul de studii universitare de
licent, ă Informatică s, i de la programele de studii universitare de masterat Advanced Techniques
for Information Processing s, i Modelarea, Proiectarea s, i Managementul Sistemelor Software);
- Informatică aplicată (la care participă ı̂n special student, i de la programul de studii
universitare de licent, ă Informatică s, i de la programele de studii universitare de masterat
Advanced Techniques for Information Processing s, i Modelarea, Proiectarea s, i Managementul
Sistemelor Software);
- Matematică (la care participă ı̂n special student, i de la programul de studii universitare de
licent, ă Matematică s, i de la programul de studii universitare de masterat Matematică Didactică);
- Matematică aplicată (la care participă ı̂n special student, i de la programul de studii
universitare de licent, ă Matematică s, i de la programul de studii universitare de masterat
Matematică Aplicată).
Tot anual, departamentul organizează sesiunea de comunicări s, tiint, ifice pentru doctoranzi
(The Doctoral Student Workshop). Sesiunea s, tiint, ifică dedicată doctoranzilor cuprinde două
sect, iuni astfel:
- The Doctoral Student Workshop in Computer Science (la care participă ı̂n special student, ii
de la programele de studii universitare de doctorat ı̂n Informatică de la S, coala Doctorală de
Informatică a Universităt, ii din Pites, ti, dar s, i din alte centre universitare);
- The Doctoral Student Workshop in Mathematics (la care participă ı̂n special student, ii
de la programele de studii universitare de doctorat ı̂n Matematică de la S, coala Doctorală de
Matematică a Universităt, ii din Pites, ti, dar s, i din alte centre universitare).
Începând din anul 2017 Departamentul Matematică-Informatică organizează anual eveni-
mentul ,,Programming Day for High School” - concurs de programare C/C++ pentru
Prezentarea Departamentului Matematică-Informatică 9

elevi cu 4 sect, iuni corespunzătoare claselor a IX-a, a X-a, a XI-a s, i a XII-a s, i programarea
robot, ilor LEGO Mindstorms EV3, la care participă un număr mare de elevi provenit, i de la
liceele reprezentative din judet, ul Arges, dar s, i din judet, ele ı̂nvecinate.
Tot ı̂ncepând din anul 2016 Departamentul Matematică-Informatică organizează anual
evenimentul ,,WAC&WS - WEB Applications Contest and Workshop for Students” -
concurs de programare WEB s, i workshop WEB pentru student, i, cu participarea unui număr mare
de student, i de la programele de specialitate din cadrul Departamentului Matematică-Informatică,
din universitate s, i din alte universităt, i.
Departamentul Matematică-Informatică implementează misiunea Facultăt, ii de S, tiint, e, Edu-
cat, ie Fizică s, i Informatică de dezvoltare ca entitate academică de elită la nivel regional s, i
nat, ional care sprijină activităt, ile de cercetare, de cres, tere a calităt, ii s, i eficient, ei proceselor de
ı̂nvăt, ământ, de relat, ionare cu comunitatea academică s, i cu cea a mediului socio-economic din
regiune.
Prezentarea Concursului de Informatică ,,Programming
Day for High School”, Edit, ia I, Pites, ti, 27 mai 2017

Doru Anastasiu Popescu 1, Costel Bălcău 2 s, i Doru Constantin 3

Departamentul Matematică-Informatică
al Universităt, ii din Pites, ti a organizat, ı̂n
data de 27 mai 2017, prima edit, ie a concur-
sului de programare clasică (folosind limba-
jele Pascal/C/C++, mediul de programare
Code::Blocks) s, i de programarea robot, ilor
LEGO (folosind mediul Mindstorms EV3) cu
numele ,,Programming Day for High School”.
La această edit, ie au participat elevi de la
unităt, i s, colare din judet, ele Arges, s, i Olt.

Premiile câs, tigătorilor au fost sponsorizate de firma ROWEB Development, director general
Viorel Costea (profesor de Informatică) s, i de firma Velox Logistics Center, director economic
Adriana Neaga.

Concursul de programare clasică

Acesta a fost organizat pe trei sect, iuni, corespunzătoare clasei a IX-a, clasei a X-a s, i claselor a
XI-a s, i a XII-a. Concurent, ii au avut de rezolvat câte două probleme, fiecare având un punctaj
de 100 de puncte. În continuare prezentăm problemele date ı̂n concurs, ı̂mpreună cu indicat, ii de
rezolvare.

Clasa a IX-a
Problema 1 – pasi
Vasile contorizează pas, ii făcut, i de acasă până la s, coală s, i observă că ı̂n unele zile numărul
acestora verifică anumite proprietăt, i. În unele zile este un produs de exact trei numere prime
(numit 3-prim), alteori este factorialul unui număr (numit factorial), iar alteori este cub
perfect (numit cub).
Cerint, ă
Cunoscând numărul de zile N s, i numărul de pas, i din fiecare zi x1 , x2 , ..., xN se cere:
1. numărul de numere 3-prim;
1
Conf. univ. dr., Universitatea din Pites, ti, dopopan@yahoo.com
2
Conf. univ. dr., Universitatea din Pites, ti, cbalcau@yahoo.com
3
Conf. univ. dr., Universitatea din Pites, ti, doru.constantin@upit.ro

10
Concursul de Informatică Programming Day 11

2. numărul de numere factorial;


3. numărul de numere cub.
Date de intrare
Fis, ierul de intrare pasi.in cont, ine pe prima linie un număr natural p. Pentru toate testele de
intrare, numărul p poate avea doar valoarea 1, 2 sau 3.
Pe linia a doua se află N , iar pe linia a treia se află numerele x1 , x2 , . . . , xN separate prin
câte un spat, iu, reprezentând numărul de pas, i până la s, coală ı̂n fiecare din cele N zile.
Date de ies, ire
Dacă valoarea lui p este 1, se va rezolva numai punctul 1) din cerint, ă.
În acest caz, ı̂n fis, ierul de ies, ire pasi.out se va scrie un singur număr natural reprezentând
numărul de numere 3-prim din s, irul x1 , x2 , . . . , xN .
Dacă valoarea lui p este 2, se va rezolva numai punctul 2) din cerint, ă.
În acest caz, ı̂n fis, ierul de ies, ire pasi.out se va scrie un singur număr natural reprezentând
numărul de numere factorial din s, irul x1 , x2 , . . . , xN .
Dacă valoarea lui p este 3, se va rezolva numai punctul 3) din cerint, ă.
În acest caz, ı̂n fis, ierul de ies, ire pasi.out se va scrie un singur număr natural reprezentând
numărul de numere cub din s, irul x1 , x2 , . . . , xN .
Restrict, ii s, i precizări

• 1≤N ≤100000

• 1≤x1 , x2 , . . . , xN ≤10000

• Pentru rezolvarea corectă a cerint, ei 1 se acordă 40% din punctaj, a cerint, ei 2 se acordă
30% din punctaj s, i a cerint, ei 3 se acordă 30% din punctaj

• Pentru 30% din teste N ≤1000

Exemple
pasi.in pasi.out Explicat, ie
1 2 p=1
3 În s, irul dat există două numere 3-prim s, i
66 11 30 anume: 66 = 2·3·11, 30 = 2·3·5.
pasi.in pasi.out Explicat, ie
2 1 p=2
3 În s, irul dat există un număr factorial s, i
66 24 30 anume: 24 = 1·2·3·4.
pasi.in pasi.out Explicat, ie
3 2 p=3
3 În s, irul dat există două numere cub s, i
27 11 125 anume: 27 = 33 , 125 = 53 .
Timp maxim de execut, ie: 0.3 secunde/test.
Memorie totală disponibilă 4 MB, din care 2 MB pentru stivă.
Doru Anastasiu Popescu, Costel Bălcău, Pites, ti, Gabriel Boroghină, Bucures, ti
12 D.A. Popescu, C. Bălcău, D. Constantin

Solut, ie de 30 de puncte
1. Se citesc p s, i N .
2. N r = 0.
3. Pentru fiecare număr din s, ir, ı̂n funct, ie de valoarea lui p (1, 2, 3) se verifică dacă
ı̂ndeplines, te una din condit, iile de 3-prim, factorial sau cub. Dacă este ı̂ndeplinită condit, ia,
atunci N r = N r + 1.
4. Se afis, ează N r.
Solut, ie de 100 de puncte
1. Se construiesc trei tablouri unidimensionale a, b, c de dimensiune 10000, unde:
a[i] = 1, dacă i este 3-prim s, i 0 ı̂n caz contrar,
b[i] = 1, dacă i este factorial s, i 0 ı̂n caz contrar,
c[i] = 1, dacă i este cub s, i 0 ı̂n caz contrar,
unde i ia valorile 1, 2, ..., 10000.
2. Se citesc p s, i N .
3. N r = 0.
4. Pentru fiecare număr din s, ir, memorat ı̂n variabila k, ı̂n funct, ie de valoarea lui p (1, 2, 3),
se verifică dacă ı̂ndeplines, te una din condit, iile a[k] == 1 (3-prim), b[k] == 1 (factorial) sau
c[k] == 1 (cub). Dacă este ı̂ndeplinită condit, ia, atunci N r = N r + 1.
5. Se afis, ează N r.

Problema 2 – continent
Pe un continent se dores, te organizarea unei olimpiade de robotică. Pentru a stabili ordinea de
alegere a t, ărilor organizatoare pe ani calendaristici se stabiles, te drept criteriu suprafat, a. Astfel,
o t, ară care are o suprafat, ă mai mare va organiza mai repede o edit, ie de olimpiadă.
Pentru stabilirea suprafet, elor t, ărilor continentului se foloses, te o hartă dată folosind un
tablou bidimensional cu M linii s, i N coloane. Un element din hartă reprezintă 1 km2 din teren.
Elementele tabloului sunt numere consecutive 0, 1, 2, . . .. T, ările sunt codificate cu 1, 2, . . . . Cifra
0 ı̂n tablou reprezintă teren sau apă care nu apart, in niciunei t, ări.
Dacă există mai multe t, ări cu aceeas, i suprafat, ă, cea care are codul asociat mai mic va
organiza prima o edit, ie de olimpiadă.
Cerint, ă
Cunoscând numerele M, N s, i tabloul ce codifică harta, se cere:
1. suprafat, a t, ării cu codul cel mai mare;
2. codurile t, ărilor ı̂n ordinea descrescătoare a suprafet, ei.
Date de intrare
Fis, ierul de intrare continent.in cont, ine pe prima linie un număr natural p. Pentru toate
testele de intrare, numărul p poate avea doar valoarea 1 sau 2.
Concursul de Informatică Programming Day 13

Pe linia a doua se află M, N , separate printr-un spat, iu, iar pe liniile următoare tabloul ce
codifică harta, pe fiecare linie componentele sunt separate prin câte un spat, iu.
Date de ies, ire
Dacă valoarea lui p este 1, se va rezolva numai punctul 1) din cerint, ă.
În acest caz, ı̂n fis, ierul de ies, ire continent.out se va scrie un număr ce reprezintă suprafat, a
t, ării cu codul cel mai mare
Dacă valoarea lui p este 2, se va rezolva numai punctul 2) din cerint, ă.
În acest caz, ı̂n fis, ierul de ies, ire continent.out se vor scrie codurile t, ărilor ı̂n ordinea
organizării olimpiadelor, câte unul pe un rând.
Restrict, ii s, i precizări

• 1≤M, N ≤300

• 0≤ coduri t, ări ≤M ·N

• Pentru rezolvarea corectă a fiecărei cerint, e se acordă 50% din punctaj

Exemple
continent.in continent.out Explicat, ie
1 3 p=1
34 Pe continent există 4 t, ări. T, ara cu codul cel
4011 mai mare, 4, are suprafat, a egală cu 3.
4401
2133
continent.in continent.out Explicat, ie
2 4 p=2
34 1 Cele patru t, ări de pe continent codificate cu
4011 3 1, 2, 3, 4 au suprafet, ele: 3, 1, 3, respectiv 4.
4431 2 Ordinea de desfăs, urare a olimpiadelor este 4,
2433 1, 3, 2.
Timp maxim de execut, ie: 0.2 secunde/test.
Memorie totală disponibilă 2 MB, din care 1 MB pentru stivă.
Doru Anastasiu Popescu, Costel Bălcău, Pites, ti, Gabriel Boroghină, Bucures, ti
Solut, ie cerint, a 1
Se determină cel mai mare număr din tablou (notat cu max) s, i frecvent, a sa F . Nu trebuie
memorat tabloul. Valoarea lui F se va afis, a.
Solut, ie cerint, a 2
Se determină cel mai mare număr din tablou (notat cu max). Pentru fiecare numar i =
1, 2, ..., max se determină de câte ori apare ı̂n tablou: x[i]. Se notează cu o[i] = i, ordinea
init, ială a t, ărilor, după care se ordonează descrescător tabloul unidimensional x s, i se interschimbă
elementele lui o odată cu cele ale lui x. Apoi se afis, ează elementele tabloului o.
14 D.A. Popescu, C. Bălcău, D. Constantin

Clasa a X-a
Problema 1 – anagrama
Ionică trebuie să rezolve o problemă de informatică ce prelucrează texte. Mai precis, are la
dispozit, ie un cuvânt C format din litere mici s, i un text T format din litere mici s, i spat, ii.
Cuvintele ı̂n T sunt separate prin unul sau mai multe spat, ii. Problema pe care trebuie să o
rezolve Ionică este de a determina numărul de vocale comune lui C s, i primului cuvânt din T s, i
apoi numărul de cuvinte anagramă ale lui C ı̂n T . Un cuvânt X este anagramă pentru un alt
cuvânt Y dacă X este format din exact literele lui Y , eventual ı̂n altă ordine.
Cerint, ă
Cunoscând C s, i T se cere:
1. numărul de vocale comune din C s, i din primul cuvânt din T ;
2. numărul de cuvinte anagramă ale lui C ı̂n T .
Date de intrare
Fis, ierul de intrare anagrama.in cont, ine pe prima linie un număr natural p. Pentru toate testele
de intrare, numărul p poate avea doar valoarea 1 sau 2.
Pe linia a doua se află C, iar pe linia a treia se află T . Fiecare linie se termină cu ,,Enter”.
Date de ies, ire
Dacă valoarea lui p este 1, se va rezolva numai punctul 1) din cerint, ă.
În acest caz, ı̂n fis, ierul de ies, ire anagrama.out se va scrie un singur număr natural repre-
zentând numărul de vocale comune din C s, i din primul cuvânt din T .
Dacă valoarea lui p este 2, se va rezolva numai punctul 2) din cerint, ă.
În acest caz, ı̂n fis, ierul de ies, ire anagrama.out se va scrie un singur număr natural repre-
zentând numărul de cuvinte anagramă ale lui C ı̂n T .
Restrict, ii s, i precizări

• 1 ≤ lungimea lui C≤ 100000

• 1 ≤ lungimea lui T ≤ 100000

• Vocalele sunt a, e, i, o, u

• Pentru rezolvarea corectă a fiecărei cerint, e se acordă 50% din punctaj

Exemple
anagrama.in anagrama.out Explicat, ie
1 2 p=1
maria C este maria
marinela se joaca Primul cuvânt din T este
marinela
Sunt 2 vocale comune. a
apare de două ori, dar este
numărat o singură dată.
Concursul de Informatică Programming Day 15

anagrama.in anagrama.out Explicat, ie


2 4 p=2
mara Cuvintele anagramă cu C ı̂n
vara aram dica rama aarm rama T sunt aram, rama, aarm s, i
rama. Numărul lor este 4.
Timp maxim de execut, ie: 0.2 secunde/test.
Memorie totală disponibilă 2 MB, din care 1 MB pentru stivă.
Doru Anastasiu Popescu, Costel Bălcău, Pites, ti, Gabriel Boroghină, Bucures, ti
Solut, ie
Cerint, a 1
1. Se citesc C s, i T .
2. N r = 0.
3. Se determină ı̂n i pozit, ia primului spat, iu ı̂n T . Dacă T are un singur cuvânt, atunci i
este numărul de litere.
4. Pentru fiecare vocală se verifică dacă se găses, te atât ı̂n C, cât s, i ı̂n T 0 T 1 . . . T i−1 . În caz
afirmativ N r = N r + 1.
5. Se afis, ează N r.
Cerint, a 2
1. Se construies, te vectorul de frecvent, e al literelor mici pentru cuvântul C, fie acesta
x = (x0 , x1 , . . . , x25 ), xi este numărul de aparit, ii a celei de-a i − a literă din alfabet ı̂n C.
2. N r = 0.
3. Pentru fiecare cuvânt din T se determină vectorul de frecvent, e al literelor mici: y =
(y 0 , y 1 , . . . , y 25 ). Dacă x s, i y au aceleas, i componente atunci N r = N r + 1.
4. Se afis, ează N r.

Problema 2 – cuburi
Ionică are un sac plin cu cuburi, numerotate cu 1, 2, . . . , N s, i M cutii numerotate cu 1, 2, . . . , M
ı̂n care ı̂ncap x1 , x2 , . . . , xM cuburi. Din păcate cutiile nu sunt suficiente pentru a introduce
cuburile din sac s, i mai rămân cuburi neintroduse. În fiecare zi după ce se joacă cu cuburile le
introduce ı̂n cutii la capacitate maximă s, i ce rămâne le pune ı̂n sac. După multe zile de joacă s, i
as, ezare a cuburilor ı̂n cutii, Ionică se ı̂ntrebă ı̂n câte moduri diferite poate as, eza cuburile ı̂n
cutii, s, tiind că ordinea cuburilor dintr-o cutie nu are important, ă.
Cerint, ă
Cunoscând numerele N, M s, i capacităt, ile cutiilor, se cere:
1. numărul maxim de cuburi pe care le poate pune ı̂n [M/2] cutii;
2. ı̂n câte moduri diferite se pot as, eza cuburile ı̂n cutii, s, tiind că ordinea cuburilor ı̂n cutii
nu are important, ă.
Date de intrare
Fis, ierul de intrare cuburi.in cont, ine pe prima linie un număr natural p. Pentru toate testele
de intrare, numărul p poate avea doar valoarea 1 sau 2.
16 D.A. Popescu, C. Bălcău, D. Constantin

Pe linia a doua se află N, M , separate printr-un spat, iu, iar pe linia următoare numerele
x1 , x2 , . . . , xM separate prin câte un spat, iu.
Date de ies, ire
Dacă valoarea lui p este 1, se va rezolva numai punctul 1) din cerint, ă.
În acest caz, ı̂n fis, ierul de ies, ire cuburi.out se va scrie un număr ce reprezintă numărul
maxim de cuburi pe care le poate pune ı̂n [M/2] cutii.
Dacă valoarea lui p este 2, se va rezolva numai punctul 2) din cerint, ă.
În acest caz, ı̂n fis, ierul de ies, ire cuburi.out se va scrie numărul de moduri diferite ı̂n care
se pot as, eza cuburile ı̂n cutii. Numărul va fi afis, at modulo 60013.
Restrict, ii s, i precizări

• 1≤N ≤1000

• 1≤M ≤1000

• 0≤x1 + x2 + ... + xM ≤N

• x modulo y reprezintă restul ı̂mpărt, irii lui x la y

• Pentru rezolvarea corectă a fiecărei cerint, e se acordă 50% din punctaj

Exemple
cuburi.in cuburi.out Explicat, ie
1 2 p=1
42 Cutia cea mai mare cont, ine 2 cu-
12 buri.
cuburi.in cuburi.out Explicat, ie
2 12 p=2
42 La cerint, a 2 numărul de modalităt, i
12 de as, ezare a cuburilor este 12.
Timp maxim de execut, ie: 0.2 secunde/test.
Memorie totală disponibilă 8 MB, din care 2 MB pentru stivă.
Doru Anastasiu Popescu, Costel Bălcău, Pites, ti, Gabriel Boroghină, Bucures, ti
Solut, ie
Cerint, a 1
Se ordonează descrescător vectorul x s, i se calculează s = x1 + x2 + ... + x[M/2] . Apoi se
afis, ează s.
Cerint, a 2
Numărul căutat este C(N, x1 ) · C(N − x1 , x2 ) · . . . · C(N − (x1 + ... + xM −1 ), xM ) modulo
60013. Combinările din produs se calculează folosind triunghiul lui Pascal modulo 60013.
Concursul de Informatică Programming Day 17

Clasele a XI-a s, i a XII-a


Problema 1 – paranteze
La ora de informatică Georgică a ı̂nvăt, at să scrie parantezările corecte folosind N perechi de
paranteze rotunde. O parantezare corectă respectă regulile din aritmetică. Adâncimea unei
parantezări este numărul maxim de perechi de paranteze una ı̂n alta. De exemplu, pentru N = 2,
parantezările corecte sunt ()(), (()). Prima are adâncimea 1, iar a doua 2. Pentru numerele
N, M s, i k, Georgică vrea să determine numărul de parantezări cu M perechi de paranteze cu
adâncimea cel mult k, precum s, i numărul total de parantezări distincte cu 1, 2, ..., N perechi de
paranteze.
Cerint, ă
Cunoscând numerele N, M s, i k se cere:
1. numărul de parantezări cu M perechi de paranteze s, i adâncimea cel mult k, modulo
60013;
2. numărul total de parantezări distincte cu 1, 2, ..., N perechi de paranteze modulo 60013.
Date de intrare
Fis, ierul de intrare paranteze.in cont, ine pe prima linie un număr natural p. Pentru toate
testele de intrare, numărul p poate avea doar valoarea 1 sau 2.
Pe linia a doua se află N, M s, i k cu spat, iu ı̂ntre ele.
Date de ies, ire
Dacă valoarea lui p este 1, se va rezolva numai punctul 1) din cerint, ă.
În acest caz, ı̂n fis, ierul de ies, ire paranteze.out se va scrie un singur număr natural repre-
zentând numărul de la cerint, a 1.
Dacă valoarea lui p este 2, se va rezolva numai punctul 2) din cerint, ă.
În acest caz, ı̂n fis, ierul de ies, ire paranteze.out se va scrie un singur număr natural repre-
zentând numărul de la cerint, a 2.
Restrict, ii s, i precizări

• 1≤N ≤1000

• 1≤M ≤15

• 1≤k≤M

• x modulo y reprezintă restul ı̂mpărt, irii lui x la y

• Pentru rezolvarea corectă a fiecărei cerint, e se acordă 50% din punctaj

Exemple
paranteze.in paranteze.out Explicat, ie
1 1 p=1
221 N = 2, M = 2, k = 1. Avem două pa-
rantezări ()(), (()). Prima are adâncimea
1, iar a doua adâncimea 2, astfel se
numără doar prima.
18 D.A. Popescu, C. Bălcău, D. Constantin

paranteze.in paranteze.out Explicat, ie


2 3 p=2
221 N = 2, M = 2, k = 1. Avem o paran-
tezare cu o pereche de paranteze: () s, i
două parntezări cu două perechi de pa-
ranteze: ()(), (()). Total 3 parantezări.
Timp maxim de execut, ie: 0.5 secunde/test.
Memorie totală disponibilă 32 MB, din care 4 MB pentru stivă.
Doru Constantin, Costel Bălcău, Pites, ti, Gabriel Boroghină, Bucures, ti
Solut, ie
Cerint, a 1
Se generează parantezările de adâncime cel mult k folosind metoda backtracking s, i se incre-
meantează o variabilă N r. Apoi se afis, ează N r.
Cerint, a 2
Numărul de parantezări cu n perechi de paranteze este numărul lui Catalan:

C(n) = comb(2n, n)/(n + 1) = comb(2n, n) − comb(2n, n − 1).

Numărul căutat este N r = (C(1) + C(2) + ... + C(n)) modulo 60013.


Combinările din sumă se calculează cu triunghiul lui Pascal modulo 60013.
Problema 2 – grupuri
La sfârs, itul clasei se contorizează situat, iile ı̂n care doi elevi s-au ajutat reciproc. Elevii sunt
codificat, i prin numerele 1, 2, ..., N . Sunt M perechi de elevi care s-au ajutat reciproc. Pentru
fiecare astfel de pereche se cunoas, te numărul de situat, ii ı̂n care s-au ajutat reciproc. În funct, ie
de aceste situat, ii s-au constituit grupuri de elevi. Doi elevi i, j fac parte din acelas, i grup dacă
există un s, ir de elevi x1 , x2 , . . . , xk cu x1 = i, xk = j s, i (x1 , x2 ), (x2 , x3 ), . . . , (xk−1 , xk ) s-au
ajutat reciproc cel put, in o dată.
Cerint, ă
Cunoscând numărul de elevi N , numărul M de perechi de elevi care s-au ajutat cel put, in o
dată, precum s, i perechile de elevi ı̂mpreună cu numărul de situat, ii ı̂n care s-au ajutat, se cere:
1. numărul de grupuri;
2. valoarea maximă a numărului total de situat, ii de ajutor reciproc ı̂ntr-un grup.
Date de intrare
Fis, ierul de intrare grupuri.in cont, ine pe prima linie un număr natural p. Pentru toate testele
de intrare, numărul p poate avea doar valoarea 1 sau 2.
Pe linia a doua se află N, M cu spat, iu ı̂ntre ele, iar pe următoarele M linii se află triplete de
forma i j k separate prin câte un spat, iu, reprezentând k situat, ii de ajutor reciproc ı̂ntre elevii i
s, i j.
Date de ies, ire
Dacă valoarea lui p este 1, se va rezolva numai punctul 1) din cerint, ă.
În acest caz, ı̂n fis, ierul de ies, ire grupuri.out se va scrie un singur număr natural reprezentând
numărul de grupuri.
Dacă valoarea lui p este 2, se va rezolva numai punctul 2) din cerint, ă.
Concursul de Informatică Programming Day 19

În acest caz, ı̂n fis, ierul de ies, ire grupuri.out se va scrie un singur număr natural reprezentând
valoarea maximă a numărului total de situat, ii de ajutor reciproc ı̂ntr-un grup.
Restrict, ii s, i precizări

• 1≤N ≤500

• 1≤M ≤100000

• Pentru rezolvarea corectă a fiecărei cerint, e se acordă 50% din punctaj

Exemple
grupuri.in grupuri.out Explicat, ie
1 2 p=1
53 Elevii sunt ı̂mpărt, it, i ı̂n două grupuri:
145 Grupul 1 cu elevii 1 s, i 4;
2 3 10 Grupul 2 cu elevii 2, 3, 5.
3 5 15
grupuri.in grupuri.out Explicat, ie
2 25 p=2
53 Elevii sunt ı̂mpărt, it, i ı̂n două grupuri:
145 Grupul 1 cu elevii 1 s, i 4 (total situatii de ajutor reciproc:
2 3 10 5);
3 5 15 Grupul 2 cu elevii 2, 3, 5 (total situat, ii de ajutor reciproc:
10+15=25).
Valoarea cea mai mare este 25.
Timp maxim de execut, ie: 0.1 secunde/test.
Memorie totală disponibilă 8 MB, din care 2 MB pentru stivă.
Doru Constantin, Costel Bălcău, Pites, ti, Gabriel Boroghină, Bucures, ti
Solut, ie
Cerint, a 1
Se construies, te un graf neorientat ponderat ı̂n care nodurile sunt elevii, iar o muchie are costul
egal cu numărul de situat, ii ı̂n care elevii asociat, i s-au ajutat reciproc. Pentru acest graf se
determină numărul de componente conexe.
Cerint, a 2
Se construies, te un graf neorientat ponderat ı̂n care nodurile sunt elevii, iar o muchie are costul
egal cu numărul de situat, ii ı̂n care elevii asociat, i s-au ajutat reciproc. Pentru acest graf se
determină componentele conexe: C1 , C2 , . . . , Ck . Apoi pentru fiecare componentă conexă Ci
se determină ı̂n xi suma totală a costurilor muchiilor ei. Valoarea maximă din vectorul x este
numărul căutat.

Concursul de programare a robot, ilor LEGO


La acest concurs a fost propusă următoarea problemă.
Problema – robotică
Punctaj maxim 120+20=140 puncte
20 D.A. Popescu, C. Bălcău, D. Constantin

Durata deplasării robotului: maxim 2 minute


Plans, ă utilizată: WRO 2017, REGULAR CATEGORY, ELEMENTARY
(http://www.wro2017.org/Regular Category/Elementary/WRO-2017-Regular-Category-Elementary-PrintV2.pdf)

Punct de plecare: pătrat verde, cu robotul orientat ı̂n orice direct, ie, dar cu toate rot, ile ı̂n
pătratul verde.
Activitate robot: să atingă cât mai multe cuburi. Pentru atingerea unui cub indiferent de
dimensiune se primesc 10 puncte (chiar dacă a fost atins de mai multe ori).
Oprirea robotului:

1. Implicit după 2 minute.


2. Când unul din concurent, ii strigă stop.
3. Când se opres, te robotul sau când acesta părăses, te tabla.
4. Dacă robotul se opres, te ı̂n locul de unde a plecat (cu toate rot, ile, pe culoarea verde) atunci
echipa mai primes, te 20 puncte.
5. Dacă se pune mâna pe robot.

Punctaj final:
Punctele se contorizează s, i la sfârs, it constituie punctajul rundei.
O echipă are 4 runde (ı̂ncercări pe plans, ă), cel mai mare punctaj ı̂mpreună cu timpul ei va fi
folosit la ı̂ntocmirea clasamentului.
Masa
La masă acesul se face ı̂n ordinea sosirii unui membru al echipei. Robotul se as, ează la
punctul de plecare s, i se pornes, te doar când un membru al juriului spune start, moment ı̂n care
se pornes, te cronometrul.
Nicolae Bold, Slatina

Câs, tigătorii concursului

Premiant, ii concursului de programare clasică, respectiv de programare a robot, ilor LEGO


Mindstorm EV3 au fost:
Sect, iunea A - Clasa a IX-a: Vădăstreanu Robert de la S, c. Gimn. ,,Eugen Ionescu”
Slatina – premiul I, Coneschi Vlad de la S, c. Gimn. ,,Eugen Ionescu” Slatina – premiul al II-lea,
Stanciu Ciprian de la C.N. ,,Radu Greceanu” Slatina – premiul al III-lea;
Sect, iunea B - Clasa a X-a: Staicu Alexandru de la C.N. ,,Ion Minulescu” Slatina –
premiul I, Mincu Daniel de la C.N. ,,Alexandru Odobescu” Pites, ti – premiul al II-lea, Popescu
Răzvan de la C.N. ,,Alexandru Odobescu” Pites, ti – premiul al III-lea;
Sect, iunea C - Clasele a XI-a s, i a XII-a: Popescu Ion Alexandru de la C.N. ,,Radu
Greceanu” Slatina – premiul I, Dumitru Corneliu Vlad de la C.N. ,,Ion C. Brătianu” Pites, ti –
premiul al II-lea, Cent, iu Emil Vasilian de la C.N. ,,Ion C. Brătianu” Pites, ti – premiul al III-lea;
Sect, iunea R - Programarea robot, ilor LEGO Mindstorm EV3: Belegan David &
Nicola Alex de la C.N. ,,Ion C. Bratianu” Pites, ti – premiul I, Conseschi Vlad & Vădăstreanu
Robert & Constantinescu Gabriel de la S, c. Gimn. ,,Eugen Ionescu” Slatina – premiul al II-lea,
Popescu Ion Alexandru & Stanciu Ciprian de la C.N. ,,Radu Greceanu” Slatina – premiul al
III-lea.
ARTICOLE S, I NOTE DE MATEMATICĂ

Numere prime de forma p = x2 + M y 2 cu x, y ∈ Z s, i


M ∈ {1, 2, 3}

Victor Alexandru 1 s, i Stelian Corneliu Andronescu 2

În acest articol vom prezenta un rezultat clasic asupra reprezentării numerelor prime,
ı̂mpreună cu câteva aplicat, ii. Primul fapt semnificativ ı̂n această direct, ie a fost descoperit de P.
Fermat: numerele prime de forma 4k + 1 se pot reprezenta ca sumă de două pătrate perfecte.
Problema reprezentării numerelor prime sub forma x2 + M y 2 , pentru M ∈ {1, 2, 3} a fost
rezolvată de L. Euler.

Propozit, ia 1. Fie p ≥ 3 un număr prim cu (p, M ) = 1 s, i astfel ı̂ncât congruent, a x2 ≡


−M (mod p) are solut, ie, iar M ∈ {1, 2, 3}. Atunci p poate fi reprezentat sub forma x2 + M y 2
cu x, y ∈ Z.

Prezentăm ı̂n continuare o demonstrat, ie a lui L. Euler, care prezintă avantajul de a fi


constructivă.

Demonstraţie. Deoarece −M este rest pătratic modulo p, există a, b ∈ N astfel că p|a2 + M b2 ,
p p p2 p2
iar (p, ab) = 1. Putem alege a, b cu |a| < , |b| < , de unde a2 + M b2 < +M ≤ p2 .
2 2 4 4
Prin urmare avem a2 + M b2 = p · q, cu 1 ≤ q < p. Dacă q 6= 1, putem reprezenta a, b sub
q q
forma a = q · α + r, b = q · β + s, cu α, β, r, s ∈ Z, |r| ≤ , |s| ≤ . Deoarece a ≡ r(mod q) s, i
2 2
q2 q2
b ≡ s(mod q), rezultă r2 + M s2 ≡ a2 + M b2 ≡ 0(mod q) s, i r2 + M s2 ≤ +M ≤ q 2 . Să
4 4
observăm că de fapt avem r2 + M s2 < q 2 chiar dacă M = 3. Într-adevăr, pentru a avea egalitate
q
ar trebui ca |r| = |s| = . De aici se obt, ine q|2a s, i q|2b s, i q 2 |4(a2 + 3b2 ) = 4pq. Însă p este prim,
2
iar 1 < q < p. Rezultă q = 2 s, i a2 + 3b2 = 2p. Însă p ≡ 1(mod 3), cum vom proba ulterior, de
unde 2 ≡ a2 (mod 3), ceea ce este imposibil. Prin urmare r2 + M s2 = qq1 , cu 1 ≤ q1 < q. În
continuare folosim identitatea

(pq) · (qq1 ) = (a2 + M b2 )(r2 + M s2 ) = (ar ± M bs)2 + M (as ∓ br)2 .

Deoarece a ≡ r(mod q) s, i b ≡ s(mod q), rezultă că as − br ≡ 0(mod q), iar de aici se obt, ine s, i
ar + M bs ≡ 0(mod q). Rezultă că există a1 , b1 ∈ Z astfel ı̂ncât q 2 pq1 = (qa1 )2 + M (qb1 )2 sau
pq1 = a21 + M b21 , cu 1 ≤ q1 < q. Dacă q1 > 1, continuăm ı̂n acelas, i mod. După un număr finit
de pas, i se ajunge la reprezentarea lui p sub forma dorită.
1
Prof. univ. dr., Universitatea din Bucures, ti, vralexandru@yahoo.com
2
Lect. univ. dr., Universitatea din Pites, ti, corneliuandronescu@yahoo.com

21
22 V. Alexandru, S.C. Andronescu

Aplicat, ii

A1. Avem x2 + 1|x4 − 1|xp−1 − 1 dacă p = 4t + 1 iar congruent, a xp−1 = 1(mod p) are p − 1
solut, ii (vezi Mica teoremă a lui Fermat) ı̂n {1, 2, . . . , p − 1}. Deoarece numărul de solut, ii ale
unei congruent, e polinomiale f (x) ≡ 0(mod p) nu poate depăs, i gradul f (folosind Teorema lui
Lagrange) ⇒ congruent, a x2 + 1 ≡ 0(mod p) are (exact) două solut, ii (dacă p ≡ 1(mod 4)) ı̂n
{1, 2, . . . , p − 1}.
Fie acum p ≡ 1(mod 8). Atunci x4 + 1|x8 − 1|xp−1 − 1. Cu acelas, i argument ca mai sus
rezultă că congruent, a x4 + 1 ≡ 0(mod p) are patru solut, ii ı̂n {1, 2, . . . , p − 1}.
Avem x4 + 1 ≡ (x2 + 1)2 − 2x2 = (x2 − 1)2 + 2x2 . Dacă a4 ≡ −1(mod p) ⇒ (a2 + 1)2 − 2a2 ≡
(a2 − 1)2 + 2a2 ≡ 0(mod p).
Cum a 6≡ 0(mod p), rezultă că ∃b, b0 ∈ Z s, i 2 ≡ b2 (mod p) s, i −2 ≡ (b0 )2 (mod p).
Fie acum p ≡ 1(mod 3), p = 3t + 1. Avem x3 − 1|xp−1 − 1 ⇒ (cu acelas, i argument) că
∃a ∈ Z, a 6≡ 1(mod p) s, i a3 − 1 = (a − 1)(a2 + a + 1) ≡ 0(mod p) de unde 4(a2 + a + 1) =
(2a + 1)2 + 3 ≡ 0(mod p) ⇔ −3 ≡ (2a + 1)2 (mod p).
Se obt, in astfel următoarele consecint, e:
a) Dacă p ≡ 1(mod 4) atunci p = a2 + b2 cu a, b ∈ Z.
b) Dacă p ≡ 1(mod 8) atunci p = a2 + 2b2 cu a, b ∈ Z.
c) Dacă p ≡ 1(mod 3) atunci p = a2 + 3b2 cu a, b ∈ Z.
Observat, ia 1. Dacă ∃b ∈ Z, b 6= 0 astfel ı̂ncât b2 ≡ −3(mod p) cu p 6= 3 atunci (conform
rat, ionamentului anterior), există a ∈ N astfel ı̂ncât a2 + a + 1 ≡ 0(mod p) ⇒ a3 ≡ 1(mod p).
Dacă (p − 1, 3) = 1 ar rezulta a ≡ 1(mod p) iar de aici p=3, fals. Prin urmare 3|p − 1.

A2. Dacă un număr natural nenul n se scrie ı̂n două moduri ca sumă de două pătrate
perfecte, atunci n este compus.
Solut, ie. Fie n = a2 + b2 = c2 + d2 , a, b, c, d ∈ N? , (a, b) 6= (c, d). Dacă a s, i b au aceleas, i paritate,
atunci n este par, deci compus. Deci putem presupune că a s, i b, respectiv c s, i d sunt numere
naturale distincte cu parităt, i diferite.
Fie a s, i c pare, b s, i d impare, deci a 6= c, b 6= d.
Avem a2 + b2 = c2 + d2 , a2 − c2 = d2 − b2 , (a − c)(a + c) = (d − b)(d + b), adică

a−c d+b u
= = , (u, v) = 1.
d−b a+c v
Rezultă a + c = hv, b + d = hu, a − c = ku, d − b = kv, cu h, k ∈ N? s, i deci

hv + ku hu − kv (u2 + v 2 )(k 2 + h2 )
a= , b= , s, i n = .
2 2 4
Se deduce us, or că ı̂nsus, i n admite o descompunere proprie.
Prin urmare un număr prim admite cel mult o reprezentare ca sumă de două pătrate perfecte.
A3. Dacă un număr n se poate scrie ca sumă de două pătrate perfecte nenule distincte,
atunci orice putere a sa se poate scrie de asemenea ca sumă de două pătrate perfecte nenule.
Solut, ie. Fie n = a2 + b2 , a, b ∈ N? .
Numere prime de forma ... 23

Dacă t = 2k + 1, atunci nt = n2k · n = n2k (a2 + b2 ) = (nk · a)2 + (nk · b)2 cu nk a, nk b ∈ N? .


Dacă t = 2k demonstrăm afirmat, ia pentru k = 1

n2 = (a2 + b2 )2 = a4 + b4 + 2a2 b2 = (a2 − b2 )2 + (2ab)2 ,

adică n2 = A2 + B 2 cu A = a2 − b2 , B = 2ab.
Pentru k ≥ 2, nt = n2k−2 · n2 = n2(k−1) (A2 + B 2 ) = (nk−1 A)2 + (nk−1 B)2 cu ak−1 A, ak−1 B ∈
?
N.
Mai general, dacă n s, i m se pot scrie ca sumă de două pătrate perfecte nenule, atunci
s, i produsul lor este o sumă de două pătrate perfecte nenule. Într-adevăr, dacă n = a2 + b2 ,
m = c2 + d2 , a, b, c, d ∈ N? atunci mn = (a2 + b2 )(c2 + d2 ) = a2 c2 + a2 d2 + b2 c2 + b2 d2 =
(ac + bd)2 + (ad − bc)2 .
Putem concluziona astfel că mult, imea M = {n ∈ N? | n = a2 + b2 , a, b ∈ N? } este parte
stabilă ı̂n raport cu ı̂nmult, irea.
Ç å
a
Dacă p este un număr prim, p ≥ 3, iar a ∈ Z, (a, p) = 1 atunci simbolul lui Legendre
p
se defines, te astfel:

dacă congruent, a x2 ≡ a(mod p) are solut, ii


Ç å ®
a 1
= .
p −1 dacă congruent, a x2 ≡ a(mod p) nu are solut, ii

Ment, ionăm câteva proprietăt, i ale simbolului lui Legendre:


Ç å Ç å
a a
1. se poate extinde pe Z prin = 0, dacă p|a.
p p
Ç å Ç å
a b
2. Dacă a ≡ b(mod p), atunci = .
p p
Ç å
a p−1
3. În condit, iile de mai sus, ≡a 2 (mod p) (Criteriul lui Euler).
p
Ç å Ç åÇ å
ab a b
4. = .
p p p
Un rezultat de seamă ı̂n teoria numerelor este Legea de reciprocitate pătratică, intuită de
Euler, formulată de Legendre s, i demonstrată de Gauss.

Teorema 1. Fie p ≥ 3 un număr prim. Atunci:


−1
Ç å
p−1
1. = (−1) 2 .
p
Ç å
2 p2 −1
2. = (−1) 8 .
p
3. Dacă q este număr prim, q ≥ 3 s, i q 6= p, atunci

Ç åÇ å
p q p−1 q−1
= (−1) 2 2 .
q p
24 V. Alexandru, S.C. Andronescu

Rezultatele mai noi privind reprezentarea numerelor prime p = x2 + M y 2 cu x, y ∈ Z s, i M


liber de pătrate (rezultate ce folosesc proprietăt, ii ale simbolului lui Legendre) pot fi găsite ı̂n
lucrarea [4].
Aceste rezultate sunt legate de proprietăt, i aritmetice profunde ale inelelor de ı̂ntregi algebrici
din corpurile pătratice. Ele au la bază studiul formelor pătratice binare cu coeficient, i ı̂ntregi
realizat de Gauss ı̂n lucrarea [3].

Bibliografie

[1] V. Alexandru, N. Gos, oniu, Elemente de teoria numerelor, Editura Universităt, ii din Bucures, ti,
1999.

[2] L. Panaitopol, A. Gica, Probleme de aritmetică s, i teoria numerelor. Idei s, i metode de


rezolvare, Editura Gill, 2006.

[3] C.Fr. Gauss, Cercetări aritmetice, Editura Amarcord, Timis, oara, 1999.

[4] D.A. Cox, Primes of the form x2 + ny 2 , Cambridge Univ. Press, 1984.
În legătură cu o inegalitate din ,,Mathematical
Reflections” nr. 6/2015

1
Marin Chirciu

În Revista ,,Mathematical Reflections” nr. 6/2015 sub semnătura Titu Andreescu, USA s, i
Oleg Mushkarov, Bulgaria este propusă problema O357:
,,Să se arate că ı̂n orice triunghi ABC este adevărată inegalitatea:
X bc + 4mb mc 16S
≥ .”
a R

Articolul ı̂s, i propune o generalizare s, i anume:


,,Să se arate că ı̂n orice triunghi ABC este adevărată inegalitatea:
X bc + k · mb mc S
≥ (3k + 4) · ,
a R
unde .k ≥ 4.”
»
Solut, ie. Vom folosi inegalităt, ile: a = (p − b) + (p − c) ≥ 2 (p − b)(p − c) s, i
2(b2 +c2 )−a2 (b+c)2 −a2
»
m2a = 4
≥ 4
= p(p − a) ⇒ ma ≥ p(p − a). Obt, inem:
Ñ » » » » é
X bc + k · mb mc X 2 (p − a)(p − c) · 2 (p − a)(p − b) k · p(p − b) · p(p − c)
≥ +
a a a
»
(p − b)(p − c) [4(p − a) + kp] X » (k + 4)(b + c) + (k − 4)a
Ç å
X
= = (p − b)(p − c) ·
a 2a
s
Am −Gm 3 3 Y (k + 4)(b + c) + (k − 4)a Y
≥ · (p − a)
2 a
(1) 3q3
Y 3 qY
≥ (3k + 4)3 · (p − a) = (3k + 4) 3 (p − a)
2 2
Gm −Hm 3 3 9(3k + 4) 1
≥ (3k + 4) · 1 1 1 = · ra rb rc
2 p−a
+ p−b + p−c 2 S
+ S + S

9(3k + 4) S 9(3k + 4) S
= · = ·
2 ra + rb + rc 2 4R + r
Euler 9(3k + 4) S S
≥ · 9R = (3k + 4) · ,
2 2
R
1
Profesor, Colegiul Nat, ional ,,Zinca Golescu”, Pites, ti, marin.chirciu@yahoo.com

25
26 M. Chirciu

Q (k+4)(b+c)+(k−4)a
unde (1) ⇔ a
≥ (3k + 4)3 , care rezultă din:
Qî ó
(k + 4) · b+c + k − 4 = (x + α) = (x + α)(y + α)(z + α) = xyz + α xy + α2 x + α3 ,
Q P P
a
unde x = (k + 4) · b+c si α = k − 4. Avem: xyz = (k + 4)3 b+c ≥ 8(k + 4)3 ;
Q
a , a
Gerretsen
P (a+b)(a+c) P (a+b)(a+c) p2 −r2 −Rr
xy = (k + 4)2 s, i = ≥ 12 ⇒
P
bc bc Rr

b+c
xy ≥ 12(k + 4)2 ;
X X X
⇒ x = (k + 4) ≥ (k + 4) · 6.
a
Obt, inem: (x + α) ≥ 8(k + 4)3 + 12(k + 4)2 (k − 4) + 6(k + 4)(k − 4)2 + (k − 4)3
Q

= [2(k + 4) + (k − 4)]3 = (3k + 4)3 , pentru k ≥ 4.


Din aceeas, i clasă de probleme vă propun să rezolvat, i:
,,Să se arate că ı̂n orice triunghi ABC este adevărată inegalitatea:
X 3bc + 4mb mc 24S
≥ .”
a R
Solut, ie. Se foloses, te inegalitatea

mediilor pentru 3bc s, i 4mb mc , inegalităt, ile din solut, ia precedentă
P1 3
s, i inegalitatea a ≥ R . Egalitatea are loc pentru triunghiul echilateral, caz ı̂n care are loc s, i
egalitatea 3bc = 4mb mc .

Bibliografie

[1] T. Andreescu, O. Mushkarov, Mathematical Reflections, nr. 6/2015.

[2] M. Chirciu, Inegalităt, i geometrice, de la init, iere la performant, ă, Editura Paralela 45, Pites, ti,
2015.
Coordonate analitice practice

Leonard Mihai Giugiuc 1 s, i Diana Trăilescu Gogan 2

În general, pentru rezolvarea unor probleme de geometrie a triunghiului prin metoda analitică,
punctele sunt alese astfel: A (0, a), B (b, 0), C (c, 0) cu a > 0 si b < c. Inconvenientul acestei
alegeri a punctelor constă ı̂n faptul că vom avea numitori nedorit, i ı̂n cazul coordonatelor
ortocentrului sau ale centrului cercului circumscris triunghiului.
Vom face următoarea alegere: A (0, a), B (−ab, 0), C (ac, 0), unde a este distant, a de la
punctul A la dreapta BC, b = ctg B, c = ctg C. Evident b + c > 0.
Printr-o omotetie de centru O s, i raport a1 (sau a2 ), putem alege A (0, 1), B (−b, 0), C (c, 0)
sau A (0, 2), B (−2b, 0), C (2c, 0). În primul caz vom avea ortocentrul H (0, bc), iar ı̂n cel de-al
doilea caz ortocentrul H (0, 2bc) s, i centrul cercului circumscris triunghiului O1 (c − b, 1 − bc).
Ca aplicat, ii ale acestei metode vom prezenta ı̂n cele ce urmează solut, ii inedite la trei probleme
date la Olimpiade Internat, ionale de Matematică.
Aplicat, ia 1. Fie H ortocentrul unui triunghi ascutitunghic ABC. Cercul centrat ı̂n mijlocul
lui (BC) s, i care trece prin H intersectează latura BC ı̂n punctele A1 s, i A2 . Analog definim
punctele B1 , B2 pe latura AC s, i C1 , C2 pe latura AB.
Demonstrat, i că punctele A1 , A2 , B1 , B2 , C1 s, i C2 sunt conciclice.
I.M.O. 2008

Solut, ie. Alegem punctele A (0, 2), B (−2b, 0) s, i C (2c, 0). Cum 4ABC este ascut, itunghic
rezultă b > 0, c > 0 s, i bc < 1. Evident, H (0, 2bc) s, i centrul cercului circumscris 4ABC
este O1 (c − b, 1 − bc). Din modul de definire a punctelor A1 , A2 , B1 , B2 , C1 , C2 , obt, inem că
mediatoarea lui [BC] coincide cu mediatoarea lui [A1 A2 ] s, i analog celelalte. Deci e suficient să
arătăm că O1 este egal depărtat de cele s, ase puncte. De asemenea, există numerele reale strict
pozitive m, n, p cu:

A1 (c − b − m, 0) , A2 (c − b + m, 0) ; B1 (c (1 − n) , 1 + n) ,

B2 (c (1 + n) , 1 − n) , C1 (−b (1 − p) , 1 + p) s, i C2 (−b (1 + p) , 1 − p) .
Deci:
O1 A1 = O1 A2 , O1 B1 = O1 B2 s, i O1 C1 = O1 C2 . (1)
−−→ −−→
Din ipoteză, avem 4HA1 A2 dreptunghic ı̂n H. Rezultă HA1 · HA2 = 0 ⇒

(c − b − m)~i − 2bc~j (c − b + m)~i − 2bc~j = 0


î óî ó

⇒ (c − b − m) (c − b + m) + 4b2 c2 = 0 ⇒ m2 = 4b2 c2 + b2 + c2 − 2bc.


1
Profesor, Colegiul Nat, ional,,Traian”, Drobeta Turnu Severin, leonardgiugiuc@yahoo.com
2
Profesor, Liceul Tehnologic Hălânga, dianaveronica80@yahoo.com

27
28 L.M. Giugiuc, D. Trăilescu Gogan

−−→ −−→
4HB1 B2 dreptunghic ı̂n H ⇒ HB1 · HB2 = 0 ⇒

c (1 − n)~i + (1 + n − 2bc) · ~j · c (1 + n) · ~i + (1 − n − 2bc) ~j = 0


î ó î ó

4b2 c2 + c2 + 1 − 4bc
⇒ n2 = .
1 + c2
−−→ −−→
4HC1 C2 dreptunghic ı̂n H ⇒ HC1 · HC2 = 0 ⇒

−b (1 − p)~i + (1 + p − 2bc) · ~j · −b (1 + p)~i + (1 − p − 2bc) · ~j = 0


î ó î ó

4b2 c2 + b2 + 1 − 4bc
⇒ p2 = .
1 + b2
O1 A1 2 = m2 + (1 − bc)2 = 5b2 c2 + b2 + c2 + 1 − 4bc.
O1 B1 2 = (b − cn)2 + (bc + n)2 = 5b2 c2 + b2 + c2 + 1 − 4bc.
O1 C1 2 = (c + bp)2 + (bc − p)2 = 5b2 c2 + b2 + c2 + 1 − 4bc.
Deci
O1 A1 = O1 B1 = O1 C1 . (2)
Din relat, iile (1) s, i (2) obt, inem că punctele A1 , A2 , B1 , B2 , C1 s, i C2 sunt conciclice.

Aplicat, ia 2. Considerăm 4ABC ascutitunghic. Fie P piciorul ı̂nălt, imii dinA, O centrul
cercului circumscris 4ABC. Presupunem că B +30◦ ≤ C. Demonstrat, i că A+m COP ÷ < 90◦ .

I.M.O. 2001

Solut, ie. Considerăm in s.c.c.o. xP y, punctele A (0, 2), B (−2b, 0), C (2c, 0). Cum 4ABC este
ascutitunghic ⇒ b, c > 0 s, i b · c < 1. De asemenea, cum B < C ⇒ b > c.

    
÷ < 90◦ ⇔ ctg A + m COP
A + m COP ÷ >0
 
⇔ ctg A · ctg COP
÷ > 1.

Avem:
1 − bc
ctg A = −ctg (B + C) = ,
b+c
   
ctg COP ◊−P
÷ = ctg COM OM , unde M este mijlocul lui (BC).

Notăm c.c. al 4ABC cu w. Avem:

w : [x − (c − b)]2 + [y − (1 − bc)]2 = 1 + b2
Ä äÄ ä
1 + c2 .

◊ = OM = 1 − bc .
 
În 4COM, ctg COM
CM b+c
  OM 1 − bc
În 4P OM, ctg P◊OM = = .
OP b−c
Coordonate analitice practice 29

1 − bc 1 − bc
· +1 (1 − bc)2 + b2 − c2
b + c b − c
 
Deci, ctg COP =
÷ = .
1 − bc 1 − bc 2c · (1 − bc)

b−c b+c
  1 − bc (1 − bc)2 + b2 − c2
ctg A · ctg COP
÷ >1 ⇔ · >1
b+c 2c (1 − bc)
(1 − bc)2 + b2 − c2 Ä äÄ ä
⇔ > 1 ⇔ 1 + b2 1 + c2 > 4c (b + c) ⇔ R2 > 4c (b + c) ,
2c (b + c)
unde R este raza lui w.
În 4P AC, P C = AC cos C = 2R sin B cos C ⇒ 2c = 2R sin B cos C ⇒ c = R sin B cos C.
Aplicând teorema sinusurilor in 4ABC, avem BC = 2R sin A ⇒ 2 (b + c) = 2R sin A
⇒ b + c = R sin A. Deci avem de arătat că
1
R2 > 4R2 sin A sin B cos C ⇔ sin A sin B cos C <
4
1 1
⇔ sin A (sin A − sin (C − B)) < .
⇔ sin A [sin (B + C) − sin (C − B)] <
2 2
1 1
Cum C − B este unghi ascutit si C − B ≥ 30◦ ⇒ sin (C − B) ≥ ⇒ − sin (C − B) ≤ − .
2 2
Este suficient să arătăm că
Ç å
1 1
sin A sin A − < ⇔ (sin A − 1) (2 sin A + 1) < 0. (3)
2 2

Cum A < 90◦ ⇒ sin A − 1 < 0 ⇒ inegalitatea (3) este adevarată, deci concluzia problemei
este adevărată.
BC CA AB √
Aplicat, ia 3. Un 4ABC si un punct D din planul său satisfac relat, ia = = = 3.
AD BD CD
Demonstrat, i că 4ABC este echilateral s, i D este centrul său.
I.M.O. 1980-Luxembourg

Solut, ie. Alegem A (0, 1), B (−b, 0), C (c, 0) s, i D (x, y) .


Avem: BC 2 = (b + c)2 , CA2 = 1+c2 , AB 2 = 1+b2 , AD2 = x2 +(y − 1)2 , BD2 = (x + b)2 +y 2
s, i CD2 = (x − c)2 + y 2 . Deci condit, iile din ipoteză devin:
b2 + 2bc + c2 = 3x2 + 3y 2 + 3 − 6y, (4)

1 + c2 = 3x2 + 3y 2 + 3b2 + 6bx (5)


s, i
1 + b2 = 3x2 + 3y 2 + 3c2 − 6cx. (6)
2 (c − b)
Scăzând din relat, ia (5) relat, ia (6) obt, inem 4 (c2 − b2 ) = 6x (c + b) ⇒ x = . Scăzând
3
2 − 3bc
din relat, ia (5) relat, ia (4), obt, inem y = . Înlocuind valorile x s, i y găsite ı̂n relat, ia (4),
3
avem:
4b2 + 4c2 + 9b2 c2 + 4 − 20bc
b2 + c2 + 2bc = + 3 − 4 + 6bc
3
30 L.M. Giugiuc, D. Trăilescu Gogan

⇒ 3b2 + 3c2 + 6bc = 4b2 + 4c2 + 9b2 c2 + 4 − 20bc − 3 + 18bc


⇒ b2 + c2 − 2bc + 9b2 c2 − 6bc + 1 = 0 ⇒ (b − c)2 + (3bc − 1)2 = 0 ⇒ b − c = 0
1
s, i 3bc − 1 = 0 ⇒ ctg B = ctg C ⇒ B = C s, i 3ctg 2 B = 1 ⇒ ctg B = √ (deoarece triunghiul
3 Ç å
π 1
nu poate avea două unghiuri obtuze) ⇒ B = C = ⇒ 4ABC este echilateral. D 0, ⇒D
3 3
este centrul 4ABC.
În ı̂ncheiere vom propune spre rezolvare trei probleme de geometria triunghiului prin metoda
de mai sus, ale căror enunt, uri pot fi găsite la adresele web următoare:

static.olimpiade.ro/uploads/attach data/32/51/5/2013 matematica internationala subiecte.pdf

ssmr.ro/files/onm2013/faza nationala/subiecte/subiecte juniori.pdf


Asupra unei probleme propuse la Olimpiada Nat, ională
de Matematică

1
Marin Ionescu

La Olimpiada Nat, ională de Matematică din 2017, domnul profesor Lucian Dragomir din
Ot, elu Ros, u a propus elevilor de clasa a X-a următoarea problemă:
Problema 1. Demonstrat, i inegalitatea:

π 2
sin ≥ ,
4n 2n
unde n este un număr natural nenul [1].
În cele ce urmează vom da o solut, ie diferită de cea propusă de Comisia Centrală.
Inegalitatea din enunt, este o consecint, ă a următorului rezultat.
Propozit, ie. Pentru orice n ∈ N? s, i x ∈ R, există inegalitatea

| sin nx| ≤ n| sin x|.

Demonstraţie. Pentru n = 1, inegalitatea este verificată. Presupunem inegalitatea adevărată


pentru n s, i demonstrăm că este adevărată pentru n + 1.
Avem:

| sin(n + 1)x| = | sin nx cos x + sin x cos nx| ≤ | sin nx cos x| + | sin x cos nx|

= | sin nx| · | cos x| + | sin x| · | cos nx| ≤ | sin nx| + | sin x| ≤ n| sin x| + | sin x| = (n + 1) · | sin x|.

Cu propozit, ia de mai sus, inegalitatea dată la concurs rezultă astfel:



2 π Å π ã π π


= sin = sin n · ≤ n · sin
= n · sin
2 4 4n
4n 4n

2 π
s, i de aici obt, inem ≤ sin .
2n 4n
În G.M. 10/2017 domnul profesor Traian Preda din Bucures, ti a propus la clasa a X-a
următoarea problemă:
π 1 π
Problema 2. (27437) Să se demonstreze inegalitatea sin ≥ sin , unde m s, i n sunt
mn m n
numere naturale nenule.
1
Profesor, Colegiul Nat, ional ,,Ion C. Brătianu”, Pites, ti, marin.ionescu61@gmail.com

31
32 M. Ionescu

Cu propozit, ia de mai sus, solut, ia este imediată:


π π ã Å π ã π π
Å

sin = sin m · = sin m · ≤ m · sin

= m · sin
n mn mn mn mn
s, i de aici obt, inem
π 1 π
sin ≥ sin .
mn m n

Bibliografie

[1] A 68-a Olimpiadă Nat, ională de Matematică, 19-23 aprilie 2017, Timis, oara.

[2] Colect, ia Gazetei Matematice, Seria B.


Probleme de calcul integral. Inegalităt, i integrale I

1
Florin Stănescu

Scopul acestei note este familiarizarea cititorului cu problemele de calcul integral, urmărindu-
se ı̂n special rezolvarea de aplicat, ii din sfera inegalităt, ilor integrale. În final, pentru pasionat, i,
am propus un test.
Funct, ii pozitive. Funct, ii monotone. Funct, ii convexe

Rb
• Fie f : [a, b] → R integrabilă s, i f (x) ≥ 0, x ∈ [a, b] . Atunci a f (x) dx ≥ 0.

• Fie f, gR: [a, b] → R două funct, ii integrabile cu proprietatea că f (x) ≤ g (x) x ∈ [a, b] .
b Rb
Atunci a f (x) dx ≤ a g (x) dx.

• Dacă f : [a, b] → R este o funct, ie integrabilă, atunci |f | este integrabilă s, i


Z
b Z b


a
f (x) dx ≤ |f (x)| dx.
a

• Inegalitatea lui Cebâs, ev :


Fie f, g : [a, b] → R două funct, ii având aceeas, i monotonie, iar p : [a, b] → [0, ∞) o funct, ie
integrabilă. Atunci are loc inegalitatea:
b b b b
ÇZ å ÇZ å ÇZ å ÇZ å
p(x)dx p(x)f (x)g(x)dx ≥ p(x)f (x)dx · p(x)g(x)dx .
a a a a

În cazul când f s, i g sunt de monotonii diferite, avem:


b b b b
ÇZ å ÇZ å ÇZ å ÇZ å
p(x)dx p(x)f (x)g(x)dx ≤ p(x)f (x)dx · p(x)g(x)dx .
a a a a

Punând p(x) = 1, (∀)x ∈ [a, b], obt, inem:


Rb Rb Rb
a) (b − a) a f (x)g(x)dx ≥ a f (x)dx a g(x)dx, dacă f s, i g au aceeas, i monotonie;
Rb Rb Rb
b) (b − a) a f (x)g(x)dx ≤ a f (x)dx a g(x)dx, dacă f s, i g sunt de monotonii diferite.
Egalitatea are loc dacă una din funct, iile f, g este constantă (cu except, ia, eventual, a unei
mult, imi numărabile).

• Dacă f : I → R este o funct, ie convexă s, i x, y, z ∈ I astfel ı̂ncât x < y < z, atunci are loc
inegalitatea:
f (y) − f (x) f (z) − f (y)
≤ .
y−x z−y
1
Profesor, S, coala Gimnazială ,,S, erban Cioculescu”, Găes, ti, florin.florinstanescu@yahoo.com

33
34 F. Stănescu

Aplicat, ii

1. Se consideră f : [a, b] → R o funct, ie derivabilă, cu derivata continuă, astfel ı̂ncât f (a) =


f (b) = 0. Arătat, i că are loc inegalitatea:
Z b
1
max f (t) ≤ · |f 0 (x)| dx.
a≤t≤b 2 a

Rx 0 R
b Rx 0
Solut, ie. Avem |f (x)| = | a f (t) dt| s, i |f (x)| = x f 0 (t) dt , deci 2 |f (x)| = | a f (t) dt| +
R
b 0 Rx Rb 0 Rb
x f (t) dt

≤ a |f 0 (t)| dt + x |f (t)| dt =
0
a |f (x)| dx, (∀) x ∈ [a, b] . Astfel, max f (t) ≤
a≤t≤b
1 Rb 0
2
· a |f (x)| dx.
2. Dacă f : [a, b] → R este o funct, ie derivabilă cu f 0 continuă pe [a, b], demonstrat, i inegalita-
tea:
b−a
Z b Z b
1 1
0≤ · |f (x)| dx −

· f (x) dx ≤ · max |f 0 (x)| .
b−a a b−a a 3 a≤x≤b
R
Solut, ie. Avem ab f (x) dx ≤ ab |f (x)| dx, deci 0 ≤
R 1 Rb 1 Rb
b−a
· a |f (x)| dx − b−a · a f (x) dx .

Mai departe, vom calcula:


Z b Z x Z b
0
f (x) dx + (t − a) f (t) dt − (b − t) f 0 (x) dt
a a x
Z b Z x Z b
= f (x) dx + (x − a) f (x) − f (x) dx + (b − x) f (x) − f (x) dx = (b − a) f (x) ,
a a x
de unde obt, inem că
Z
b Z x Z b
0 0
|(b − a) f (x)| =

a
f (x) dx + (t − a) f (t) dt − (b − t) f (x) dt
a x
Z Z
b x Z b
0


a
f (x) dx +

(t − a) |f (t)| dt + (b − t) |f 0 (t)| dt.
a x

Rezultă Z b Z
b
(b − a) · |f (x)| dx − (b − a) f (x) dx
a a
Z b ÅZ x ã Z Ç Z b b
å
0 0
≤ (t − a) |f (t)| dt dx + (b − t) |f (t)| dt dx
a a a x

(b − a)3
≤ · max |f 0 (x)| ,
3 a≤x≤b

ceea ce ı̂ncheie demonstrat, ia.


3. Fie f : [a, b] → R derivabilă cu derivata continuă. Să se arate că:

(b − a)2
Z a+b Z b
2


f (x) dx − f (x) dx ≤ · max |f 0 (x)| .
a a+b
2
4 x∈[0,1]

0
Solut, ie. Avem: ab f (x) dx = ab (x − a) f (x) dx = (b −Ra) f (b) − ab (x − a) f 0 (x) dx,
R R R

iar din teorema de medie există c1 ∈ (a, b) astfel ı̂ncât ab (x − a) f 0 (x) dx = f 0 (c1 ) ·
Rb 0 (b−a)2 Rb 0 (b−a)2
a (x − a) dx = f (c 1 ) 2
, de unde a f (x) dx = (b − a) f (b) − f (c 1 ) 2
. În mod
2
f (x) dx =(b − a) f (a) + f 0 (c2 ) (b−a)
Rb
analog, există c2 ∈ (a, b) astfel ı̂ncât a 2
.
Inegalităt, i integrale 35

Astfel, inegalitatea de demonstrat rezultă din:


Ç
(b − a)2
å Ç å Ç å Ç å
a+b a+b a+b a+b

−a f − f 0 (c1 ) − b− f

2 2 8 2 2

(b − a)2 (b − a)2 0 (b − a)2

−f 0 (c2 ) = |f (c 1 ) + f 0
(c 2 )| ≤ · max |f 0 (x)| .
8
8 4 x∈[0,1]

4. Fie f : [0, ∞) → [0, ∞) o funct, ie integrabilă. Arătat, i că:


Z 1 Z 1 Z 1
f (x) dx · f 2 (x) dx ≤ f 3 (x) dx.
0 0 0

Solut, ie. Din inegalitatea lui Schur de gradul al III-lea, avem:


f 3 (x) + f 3 (y) + f 3 (z) + 3f (x) f (y) f (z) ≥ f 2 (x) f (y) + f 2 (y) f (x)
+f 2 (y) f (z) + f 2 (z) f (y) + f 2 (z) f (x) + f 2 (z) f (y) ,
(∀) x, y, z ∈ [0, ∞) , de unde prin integrare ı̂n funct, ie de cele trei variabile, obt, inem
Z 1 å3
1 Z 1 Z 1
ÇZ
3 2
3 f (x) dx + 3 f (x) dx ≥6 f (x) dx · f (x) dx
0 0 0 0
Z 1 Z 1 Z 1
2
⇒ 6 f (x) dx · f (x) dx ≤ 6 f 3 (x) dx,
Jensen 0 0 0
ceea ce ı̂ncheie demonstrat, ia.
5. Fie f : [0, ∞)R →î [0,
Ä ∞)
ä o functó , ie integrabilă pe orice interval [0, a] , a > 0, astfel incât
f (x) ≤ x12 s, i 0x f 2t − f (t) dt ≤ 1, (∀) x ∈ (0, ∞) . Să se arate că
Z 1
f (x) dx ≤ 1.
0

Solut, ie. Relat, ia din enunt, poate fi scrisă sub forma


x
Z
2 1Å x ã Z
f (t) dt ≤ 1+ f (t) dt , (∀) x > 0
0 2 0
Z x Ç Z 2x å
1
⇒ f (t) dt ≤ 1+ f (t) dt ,
0 2 0
iar prin induct, ie obt, inem că
Z x Ç Z 2n x å
1
f (t) dt ≤ n 2n − 1 + f (t) dt , (∀) n ≥ 1.
0 2 0

Acum, pentru a > 1, putem scrie:


Z a Z 1 Z a Z 1 Z a
1
f (x) dx = f (x) dx + f (x) dx ≤ f (x) dx + dx
0 0 1 0 1 x2
Z 1 Z 1
1
= f (x) dx + 1 − <1+ f (x) dx := A.
0 a 0

În continuare, Z x 2n x
Ç å
1
Z
f (t) dt ≤ n 2n − 1 + f (t) dt
0 2 0
A + 2n − 1
Z 1
≤ → 1, (∀) x > 0 ⇒ f (t) dt ≤ 1.
2n n→∞ 0
36 F. Stănescu

6. Fie
R1
a, b ∈ R Rcu a > 0, 2a + 3b = 6 s, i f : [0, 1] → R descrescătoare. Să se arate că:
1 2
0 f (x) dx ≥ 0 (ax + bx) f (x) dx.

Solut, ie. Notăm g : [0, 1] → R, g(x) = ax2 + bx − 1. Ecuat, ia g (x) = 0 are o unică solut, ie α
in intervalul (0, 1).
Rezultă g (x) ≤ 0, (∀) x ∈ [0, α] s, i g (x) ≥ 0, (∀) x ∈ [α, 1] . Cum f este descrescătoare,
obt, inem
g (x) (f (x) − f (α)) ≤ 0, (∀) x ∈ [0, α]
s, i
g (x) (f (α) − f (x)) ≥ 0, (∀) x ∈ [α, 1]
Z 1
⇒ g (x) (f (x) − f (α)) ≤ 0, (∀) x ∈ [0, 1] ⇒ g (x) f (x) dx ≤ 0.
0

7. Dacă f : [0, 2π] → R este o funct, ie convexă s, i integrabilă, atunci are loc inegalitatea:
Z 2π
f (x) · cos xdx ≥ 0.
0

Solut, ie. Putem scrie:


Z 2π Z π Z 2π
f (x) · cos xdx = f (x) cos xdx + f (x) cos xdx.
0 0 π

Pentru ultima integrală făcând substitut, ia x = t + π, rezultă că


Z 2π Z π
f (x) cos xdx = − f (x + π) cos xdx,
π 0

de unde Z 2π Z π
f (x) · cos xdx = cos x (f (x) − f (x + π)) dx.
0 0

Acum, pentru x, y ∈ [0, π] , x < y, deci x < y < x + π < y + π, obt, inem că

f (y + π) − f (x + π) f (y) − f (x)

y−x y−x

⇒ f (y) − f (y + π) ≤ f (x) − f (x + π) .
Astfel, funct, ia f (x) − f (x + π) , x ∈ [0, π] este descrescătoare.
În final, cum s, i cos x este descrescătoare pe [0, π], aplicând inegalitatea lui Cebâs, ev, avem
Z 2π Z π
f (x) · cos xdx = cos x (f (x) − f (x + π)) dx
0 0

1 ÅZ π ã ÅZ π ã
≥ cos xdx · (f (x) − f (x + π)) dx = 0.
π 0 0

8. Presupunem
R1
că f : [0, 1] → R este o funct, ie derivabilă, cu f 0 continuă, astfel ı̂ncât
0 f (x) dx = 0. Pentru α ∈ (0, 1) demonstrat, i inegalitatea:
Z α
1

· max |f 0 (x)| .

f (x) dx ≤

0 8 x∈[0,1]
Inegalităt, i integrale 37

Solut, ie. Considerăm funct, ia derivabilă de două ori


x2
Z x
g : [0, 1] → R, g (x) = , f (t) dt + M ·
0 2
unde M = max |f 0 (x)| . Avem g 00 (x) = f 0 (x) + M ≥ 0, (∀) x ∈ [0, 1] , deci g este convexă
x∈[0,1]
pe [0, 1] . Folosind că g este convexă, putem scrie:
g (α) = g ((1 − α) · 0 + α · 1) ≤ (1 − α) g (0) + αg (1)
å2
α2 1−α+α
Z α Z α Ç
α M M 1
⇒ f (x) dx + M ≤ M ⇒ f (x) dx = α (1 − α) ≤ = M.
0 2 2 0 2 2 2 8

În final, ı̂nlocuind pe f cu −f , obt, inem că − 0 f (x) dx ≤ 81 M , deci
Z α
1

· max |f 0 (x)| .

f (x) dx ≤

0 8 x∈[0,1]
9. Dacă h este o funct, ie pozitivă s, i descrescătoare pe [0, 1], atunci:
R1 1 2
xh2 (x) dx
R
0 0 h (x) dx
R1 ≤ R1 .
0 xh (x) dx 0 h (x) dx
1
Solut, ie. În inegalitatea lui Cebâs, ev, pentru f (x) = x, g (x) = h(x)
, p (x) = h2 (x) , obt, inem:
1 1 b b
ÇZ å ÇZ å ÇZ å ÇZ å
2 2 1 2 2 1
h (x)dx · h (x) · x · dx ≥ h (x) · xdx h (x) dx
0 0 h (x) a a h (x)
1 1 b b
ÇZ å Z ÇZ å ÇZ å
2 2
⇒ h (x)dx · x · h (x) dx ≥ h (x) · xdx h(x)dx .
0 0 a a
10. Considerăm P ∈ R[X] un polinom de grad n ≥ 2 cu coeficient, i strict pozitivi s, i 0 ≤ a < b.
Dacă P are numai rădăcini reale, demonstrat, i că:
P 00 (x) P 0 (a)
Z b
P (a)
(b − a) dx > ln · ln 0 .
a P (x) P (b) P (b)
Solut, ie. Dacă x1 , x2 , ..., xn sunt rădăcinile polinomului P , din enunt, deducem că ele sunt
strict negative. Mai avem că
P 0 (x) 1 1 1
= + + ... + , (∀)x ≥ 0.
P (x) x − x1 x − x2 x − xn
Derivând, obt, inem:
å0
P 0 (x)
Ç !
1 1 1
=− 2 + 2 + ... + < 0.
P (x) (x − x1 ) (x − x2 ) (x − xn )2
0
În continuare, construind funct, ia φ : [0, ∞) → R, φ(x) = PP (x) (x)
, rezultă că φ este strict
0
descrescătoare. Deoarece P are numai rădăcini reale strict negative, analog, obt, inem că
00 (x)
funct, ia ψ : [0, ∞) → R, ψ(x) = PP 0 (x) este strict descrescătoare, deci funct, iile φ s, i ψ au
aceeas, i monotonie si nu sunt constante.
Dacă 0 ≤ a < b, putem scrie:
P 00 (x)
Z b Z b Z b Z b
1
dx = φ(x) · ψ(x)dx > φ(x)dx · ψ(x)dx
a P (x) a b−a a a

P 0 (x) P 00 (x) P 0 (b)


Z b Z b
1 1 P (b)
= dx · dx = ln · ln .
b−a a P (x) a P 0 (x) b − a P (a) P 0 (a)
38 F. Stănescu

Test
Ra
1. Fie f : [0, 1] → R. Dacă f este derivabilă s, i există a ∈ (0, 1] astfel ı̂ncât 0 f (x) dx = 0, să
se arate că 01 f (x) dx ≤ 1−a sup |f 0 (x)|.
R
2
x∈[0,1]
2. Dacă f : [0, 1] → R este o funct, ie crescătoare, demonstrat, i inegalităt, ile:
Z 1 Z 1 Z 1
n
f (x) dx ≤ xn−1 f (x) dx ≤ 1
f (x) dx, (∀) n ∈ N∗ .
0 0 1− n

R1
3. Dacă f : [0, 1] → R este o funct, ie derivabilă, cu f 0 integrabilă s, i 0 f (x) dx = 0, demonstrat, i
inegalitatea: å2
Z 1Ä Ç Z 1
0 2
ä
2
1−x (f (x)) dx ≥ 24 xf (x) dx .
0 0
Ä ä
4. Dacă f : [0, 1] → R, este o funct, ie integrabilă, cuf 0 integrabilă s, i f 1
2
= 0, arătat, i că are
loc inegalitatea: å2
1 Z 1
ÇZ
1 2
f (x) dx ≤ · (f 0 (x)) dx.
0 12 0

5. Fie a, b > 0. Demonstrat, i că:


√π Ç s
sin x2 cos x2
Ç å å Ç å
1 1 1 π 1 1
Z
2
√ + ≤ + dx ≤ 2
+ 2 .
2π a b 0 a b 2 a b

Bibliografie

[1] M. Andronache, R. Gologan, D. Schwarz, D. S, erbănescu, Olimipiada de matematică 2006-


2010, Ed. Sigma, Bucures, ti, 2010.

[2] M.O. Drimbe, Inegalităt, i, idei s, i metode, Ed. Gil, Zalău, 2003.

[3] L.G. Lăduncă, Borne pentru matematicieni, Algebră-Analiză, clasele IX-XII, Ed. Taida, Ias, i,
2010.

[4] C. Mortici, Bazele Matematicii. Teorie s, i exercit, ii, Ed. Paralela 45, Pites, ti, 2016.

[5] N. Mus, uroaia, Gh. Boroaica, Analiză Matemtică pentru concursuri, olimpiade s, i centre de
excelent, ă, Clasa a XII-a, Ed. Paralela 45, Pites, ti, 2014.

[6] F. Stănescu, Inegalităt, i integrale:de la init, iere la performant, ă, Ed. Paralela 45, Pites, ti, 2015.

[7] R.T. Rockafeller, Analiză convexă, Ed. Theta, Bucures, ti, 2002.

[8] Colect, ia Gazetei Matematice, Seria B.


ARTICOLE S, I NOTE DE INFORMATICĂ

Optimalitatea algoritmului de căutare binară

Costel Bălcău 1

În acest articol vom demonstra că algoritmul de căutare binară este optim. Pentru aceasta
vom prezenta s, i utiliza câteva not, iuni s, i rezultate de bază din teoria algoritmilor: ordine de
complexitate, Teorema Master, proprietăt, i ale arborilor binari strict, i.

Ordine de complexitate

Timpul de execut, ie al unui algoritm depinde, ı̂n general, de setul datelor de intrare, iar pentru
fiecare astfel de set el este bine determinat de numărul de operat, ii executate s, i de tipul acestora.
Astfel timpul de execut, ie al unui algoritm poate fi interpretat s, i analizat drept o funct, ie pozitivă
ce are ca argument dimensiunea datelor de intrare.
Definit, ia 1. Pentru orice algoritm A, notăm cu TA (n) timpul de execut, ie pentru algoritmul
A corespunzător unui set de date de intrare având dimensiunea totală n.
Definit, ia 2. Un algoritm A este considerat optim dacă (se demonstrează că) nu există un
algoritm având un timp de execut, ie mai bun pentru rezolvarea problemei date, adică pentru orice
algoritm A0 care rezolvă problema dată avem TA (n) ≤ TA0 (n), pentru orice n.

Obt, inerea de algoritmi pur optimi - ı̂n sensul definit, iei anterioare - este posibilă ı̂n put, ine
situat, ii, iar demonstrarea optimalităt, ii acestora este de obicei dificilă. Mult mai des se ı̂ntâlnesc
algoritmi cu o comportare apropiată de cea optimă, pentru valori suficient de mari ale dimensiunii
setului datelor de intrare. Prezentăm ı̂n continuare câteva not, iuni prin care se cuantifică această
apropiere.
Definit, ia 3. Fie f, g : N \ A → R+ două funct, ii, unde A este o mult, ime finită, A ⊂ N.
f (n)
a) f s, i g se numesc asimptotic echivalente s, i notăm f (n) ∼ g(n) dacă ∃ lim = 1.
n→∞ g(n)

b) Spunem că f este asimptotic mărginită superior de g, iar g este asimptotic


mărginită inferior de f s, i notăm f (n) = O (g(n)) s, i g(n) = Ω(f (n)) dacă ∃ c > 0, ∃ n0 ∈ N
astfel ı̂ncât f (n) ≤ c · g(n), ∀ n ≥ n0 .
c) Spunem că f s, i g au acelas, i ordin de cres, tere s, i notăm f (n) = Θ(g(n)) dacă
f (n) = O (g(n)) s, i f (n) = Ω(g(n)).

Următorul rezultat este o consecint, ă imediată a definit, iei anterioare.


1
Conf. univ. dr., Universitatea din Pites, ti, cbalcau@yahoo.com

39
40 C. Bălcău

Propozit, ia 1. Fie f, g : N \ A → R+ două funct, ii, unde A este o mult, ime finită, A ⊂ N.
f (n)
Presupunem că există n→∞
lim = L. Atunci:
g(n)
a) f (n) = O (g(n)) dacă s, i numai dacă L ∈ [0, +∞);
b) f (n) = Ω(g(n)) dacă s, i numai dacă L ∈ (0, +∞];
c) f (n) = Θ(g(n)) dacă s, i numai dacă L ∈ (0, +∞).

Corolarul 1. Fie f, g : N \ A → R+ două funct, ii, unde A este o mult, ime finită, A ⊂ N. Dacă
f (n) ∼ g(n), atunci f (n) = Θ(g(n)).

Definit, ia 4. Fie A un algoritm s, i f : N\A → R+ o funct, ie, unde A este o mult, ime finită, A ⊂ N.
Spunem că algoritmul A are ordinul de complexitate (complexitatea) O (f (n)), Ω(f (n)),
respectiv Θ(f (n)) dacă TA (n) = O (f (n)), TA (n) = Ω(f (n)), respectiv TA (n) = Θ(f (n)).

Observat, ia 1. Notat, ia O se utilizează pentru a exprima complexitatea unui algoritm cores-


punzătoare timpului de execut, ie ı̂n cazul cel mai defavorabil, fiind astfel cea mai adecvată analizei
algoritmilor. Notat, ia Ω este corespunzătoare timpului de execut, ie ı̂n cazul cel mai favorabil, caz
practic irelevant, fiind astfel mai put, in utilizată. Notat, iile ∼ s, i Θ se utilizează atunci când se
constată că timpii de execut, ie corespunzători cazurilor cel mai defavorabil s, i cel mai favorabil
fie sunt chiar asimptotic echivalent, i (notat, ia ∼, deci s, i notat, ia Θ), cazul cel mai simplu fiind
acela al algoritmilor a căror executare depinde doar de dimensiunea setului de date de intrare,
nu s, i de valorile acestor date, fie au măcar acelas, i ordin de cres, tere (notat, ia Θ). Tot aceste
notat, ii se utilizează s, i atunci când se poate determina timpul mediu de execut, ie al algoritmului,
calculat ca medie aritmetică ponderată a timpilor de execut, ie pentru toate seturile de date de
intrare posibile, ponderile fiind frecvent, ele de aparit, ie ale acestor seturi.

Definit, ia 5. Un algoritm A este considerat asimptotic-optim dacă (se demonstrează că) nu


există un algoritm având un ordin de complexitate mai bun pentru rezolvarea problemei date,
adică pentru orice algoritm A0 care rezolvă problema dată avem TA (n) = O (TA0 (n)).

Evident, orice algoritm optim este asimptotic-optim. Reciproca acestei afirmat, ii nu este
adevărată, ı̂n continuare fiind prezentat un exemplu ı̂n acest sens.
Exemplul 1. Considerăm problema determinării maximului s, i minimului dintre elementele unui
vector dat A = (a1 , a2 , . . . , an ), n ≥ 1, adică se cere să se determine perechea (M, m), unde
M = max{ai | 1 ≤ i ≤ n}, m = min{ai | 1 ≤ i ≤ n}.
Un algoritm uzual de rezolvare este următorul.
MAX-MIN(A, n, M, m) :
M ← a1 ; m ← a1 ;
for i = 2, n do
if ai > M then
M ← ai ;
else
if ai < m then
m ← ai ;

Pentru evaluarea complexităt, ii algoritmilor care rezolvă problema dată, vom analiza numai com-
parat, iile ı̂n care intervin elemente ale vectorului sau valorile M s, i m, numite comparat, ii de chei.
Optimalitatea algoritmului de căutare binară 41

Evident, algoritmul MAX-MIN efectuează cel put, in n − 1 s, i cel mult 2n − 2 astfel de comparat, ii
(iar celelalte operat, ii nu depăs, esc ordinul de cres, tere al acestora), deci are complexitatea Θ(n).
Se poate demonstra us, or prin induct, ie că orice algoritm A care calculează maximul dintre n
elemente, bazat pe comparat, ii de chei, necesită cel put, in n − 1 astfel de comparat, ii, deci are
complexitatea Ω(n). Astfel TA (n) = Ω(TMAX-MIN (n)), sau, echivalent, TMAX-MIN (n) = O (TA (n)),
deci algoritmul MAX-MIN este asimptotic-optim (ı̂n clasa algoritmilor bazat, i pe comparat, ii de
chei). Pe de altă parte, el nu este optim, din punct de vedere al timpului de execut, ie ı̂n cazul
cel mai defavorabil, deoarece există algoritmi (bazat, i pe comparat, ii de chei) care ı̂n cazul cel
mai defavorabil efectuează mai put, in de 2n − 2 comparat, ii de chei, cât efectuează algoritmul
MAX-MIN. Un astfel de exemplu este următorul algoritm, ce compară a1 cu a2 , a3 cu a4 , . . . ,
s, i calculează maximul dintre maximele acestor perechi s, i minimul dintre minimele perechilor.
MAX-MIN-PER(A, n, M, m) :
M ← an ; m ← an ;
for i = 1, bn/2c do
if a2i−1 > a2i then
if a2i−1 > M then
M ← a2i−1 ;
if a2i < m then
m ← a2i ;
else
if a2i > M then
M ← a2i ;
if a2i−1 < m then
m ← a2i−1 ;

(bxc reprezintă parteaõ û ı̂ntreagă a numărului real x). Evident, algoritmul MAX-MIN-PER
n
efectuează exact 3 · comparat, ii de chei, indiferent de ordinea dintre elementele vectorului A
2
(iar celelalte operat, ii nu depăs, esc ordinul de cres, tere al acestora), deci are complexitatea Θ(n).

Teorema Master

Prezentăm un rezultat celebru, deosebit de util ı̂n determinarea complexităt, ii algoritmilor


specifici metodei de programare Divide et Impera s, i nu numai.
Considerăm o relat, ie de recurent, ă de forma
T (n) = T (n/b) + . . . + T (n/b) +f (n), ∀ n > n0 ,
| {z }
a termeni

unde T, f : Nõ →û R+ , °n0§∈ N, a ∈ N∗ , b ∈ R, b > 1 s, i, prin convent, ie, fiecare termen n/b


n n
reprezintă fie , fie .
b b
Ment, ionăm că dxe = min{k | k ∈ Z, k ≥ x} reprezintă aproximarea ı̂ntreagă prin adaos a
numărului real x, numită s, i partea ı̂ntreagă superioară a lui x. În acest context, partea ı̂ntreagă
uzuală, bxc (notată de obicei cu [x]), se numes, te s, i partea ı̂ntreagă inferioară a lui x.
Prin abuz de notat, ia de ı̂nmult, ire, această relat, ie de recurent, ă este convent, ional rescrisă
prescurtat sub forma
T (n) = aT (n/b) + f (n), ∀ n > n0 . (1)
42 C. Bălcău

Următorul rezultat prezintă o metodă generală pentru rezolvarea acestei relat, ii de recurent, ă.
Teorema 1 (Teorema Master [3]). Fie T : N → R+ o funct, ie ce verifică relat, ia de recurent, ă
(1), sub ipotezele s, i convent, iile de mai sus.
Cazul 1. Dacă f (n) = O (nc ), cu c < logb a, atunci T (n) = Θ(nlogb a ).
Cazul 2. Dacă f (n) = Θ(nc logk2 n), cu c = logb a s, i k ≥ 0, atunci T (n) = Θ(nlogb a logk+1
2 n).
Cazul 3. Dacă f (n) = Ω(nc ), cu c > logb a, s, i există k < 1 s, i n1 ∈ N astfel ı̂ncât
af (n/b) ≤ kf (n) ∀ n ≥ n1 , atunci T (n) = Θ(f (n)).

Proprietăt, i ale arborilor binari strict, i

Deoarece terminologia ı̂n tematica arborilor nu este unificată, definim not, iunile pe care le vom
utiliza ı̂n continuare.
Definit, ia 6. Un arbore binar este un arbore cu rădăcină, reprezentat pe nivele, ı̂n care fiecare
nod x are cel mult doi descendent, i (fii, succesori direct, i), situat, i pe nivelul următor, s, i anume
(cel mult) un descendent stâng, situat ı̂n stânga lui x, s, i (cel mult) un descendent drept,
situat ı̂n dreapta lui x, contând ordinea dintre aces, tia. Nodurile cu cel put, in un descendent se
numesc interne (interioare), iar nodurile fără descendent, i se numesc externe (exterioare,
frunze).
Definit, ia 7. Fie T = (V, F ) un arbore binar având rădăcina r, r ∈ V .
a) Notăm cu I(T ) mult, imea nodurilor interne s, i cu E(T ) mult, imea nodurilor ex-
terne ale arborelui binar T .
b) Pentru orice nod x ∈ V , distant, a de la nodul rădăcină r la nodul x, notată cu
DT (x) = D(x), reprezintă lungimea lant, ului elementar unic de la r la x.
c) Numărul h(T ) = max{DT (x) | x ∈ V } se numes, te ı̂nălt, imea arborelui binar T .
Observat, ia 2. Fie T = (V, F ) un arbore binar. Evident, avem V = I(T )∪E(T ), I(T )∩E(T ) = Ø,
E(T ) 6= Ø s, i h(T ) = max{DT (x) | x ∈ E(T )}. Deci h(T ) = k − 1, unde k reprezintă numărul
de nivele ale arborelui T .
Dacă nivelul pe care este reprezentată rădăcina arborelui este numerotat cu 0, atunci pentru
orice nod x ∈ V avem DT (x) = numărul nivelului pe care este situat nodul x, iar ı̂nălt, imea
arborelui T este egală cu numărul ultimului nivel al arborelui.
Definit, ia 8. Un arbore binar strict este un arbore binar pentru care orice nod intern are
exact doi descendent, i (s, i anume un descendent stâng s, i un descendent drept).
Propozit, ia 2. Orice arbore binar strict cu n noduri interne, n ∈ N, are n + 1 noduri externe.

Demonstraţie. Demonstrăm afirmat, ia din enunt, prin induct, ie după n.


Pentru n = 0 este evident că arborele este format doar din nodul rădăcină, deci are un singur
nod s, i acesta este extern.
Presupunem afirmat, ia adevărată pentru n − 1 s, i o demonstrăm pentru n, n ≥ 1. Fie
T = (V, F ) un arbore binar strict cu n noduri interne. Fie x un nod intern al lui T situat pe
Optimalitatea algoritmului de căutare binară 43

penultimul nivel, deci x are ca descendent, i două noduri externe y s, i z, situate pe ultimul nivel.
Fie
T 0 = T \ {y, z} = (V \ {y, z}, F \ {[x, y], [x, z]})
arborele obt, inut din T prin eliminarea nodurilor externe y s, i z. Evident, T 0 este arbore binar
strict s, i I(T 0 ) = I(T ) \ {x}, E(T 0 ) = E(T ) \ {y, z} ∪ {x}, deci card(I(T 0 )) = card(I(T )) − 1 =
n − 1, card(E(T 0 )) = card(E(T )) − 1. Conform ipotezei de induct, ie, aplicată pentru T 0 , avem
card(E(T 0 )) = card(I(T 0 )) + 1 = n, deci card(E(T )) = card(E(T 0 )) + 1 = n + 1, adică arborele
T are n + 1 noduri externe. Demonstrat, ia prin induct, ie este ı̂ncheiată.
Propozit, ia 3. Pentru orice arbore binar strict T cu n noduri interne, n ∈ N, avem
X X
DT (v) = DT (v) + 2n.
v∈E(T ) v∈I(T )

Demonstraţie. Demonstrăm egalitatea din enunt, prin induct, ie după n.


Pentru n = 0 este evident că arborele T este format doar din nodul rădăcină r, E(T ) = {r},
I(T ) = Ø, DT (v) = DT (r) = 0, DT (v) + 2n = 0 + 2 · 0 = 0, deci egalitatea din
P P
v∈E(T ) v∈I(T )
enunt, este verificată.
Presupunem egalitatea adevărată pentru n − 1 s, i o demonstrăm pentru n, n ≥ 1. Fie
T = (V, F ) un arbore binar strict cu n noduri interne. Fie x un nod intern al lui T situat pe
penultimul nivel, deci x are ca descendent, i două noduri externe y s, i z, situate pe ultimul nivel.
Evident, DT (y) = DT (z) = 1+DT (x). Fie T 0 = T \{y, z} = (V \{y, z}, F \{[x, y], [x, z]}) arborele
obt, inut din T prin eliminarea nodurilor externe y s, i z. Evident, T 0 rămâne tot arbore binar
strict s, i I(T 0 ) = I(T ) \ {x}, E(T 0 ) = E(T ) \ {y, z} ∪ {x}, DT 0 (v) = DT (v), ∀ v ∈ I(T 0 ) ∪ E(T 0 ).
Utilizând s, i ipoteza de induct, ie, aplicată pentru T 0 , avem DT (v) = DT 0 (v)−DT 0 (x)+
P P
v∈E(T ) v∈E(T 0 )
DT (y)+DT (z) = DT 0 (v)+2(n−1)−DT (x)+2(1+DT (x)) = DT (v)+DT (x)+2n =
P P
v∈I(T 0 ) v∈I(T )\{x}
DT (v) + 2n. Demonstrat, ia prin induct, ie este ı̂ncheiată.
P
v∈I(T )

Propozit, ia 4. Pentru orice arbore binar strict T cu n noduri interne, n ∈ N, avem


h(T ) ≥ dlog2 (n + 1)e .

Demonstraţie. Considerăm că nivelul pe care este reprezentată rădăcina arborelui este numerotat
cu 0. T este un arbore binar strict, cu 2n + 1 noduri (n interne s, i n + 1 externe), iar pe fiecare
nivel i ∈ {0, 1, . . . , h(T )} el are cel mult 2i noduri (induct, ie!), deci 2n+1 ≤ 1+2+22 +. . .+2h(T ) .
Obt, inem, succesiv: 2n + 1 ≤ 2h(T )+1 − 1, 2h(T ) ≥ n + 1, h(T ) ≥ log2 (n + 1). Cum h(T ) ∈ N,
deducem că h(T ) ≥ dlog2 (n + 1)e .
Observat, ia 3. Pentru orice n ∈ N∗ avem dlog2 (n + 1)e = 1 + blog2 nc .
Într-adevăr, fie dlog2 (n + 1)e = k, k ∈ N∗ . Avem, succesiv: k − 1 < log2 (n + 1) ≤ k,
2k−1 < n + 1 ≤ 2k , 2k−1 ≤ n < 2k , k − 1 ≤ log2 n < k. Deci blog2 nc = k − 1 = dlog2 (n + 1)e − 1.
Propozit, ia 5. Fie T un arbore binar strict cu n noduri interne, n ∈ N. Dacă toate nodurile
externe sunt situate pe ultimul s, i, eventual, pe penultimul nivel, atunci avem
h(T ) = dlog2 (n + 1)e ,
DT (v) = (n + 1) dlog2 (n + 1)e + n + 1 − 2dlog2 (n+1)e .
X

v∈E(T )
44 C. Bălcău

Demonstraţie. Considerăm din nou că nivelul pe care este reprezentată rădăcina arborelui este
numerotat cu 0. Rezultă că:

• arborele are h(T ) + 1 nivele, numerotate cu 0, 1, . . . , h(T );

• pe nivelele 0, 1, . . . , h(T ) − 2 avem doar noduri interne, fiecare având câte doi descendent, i;

• pe penultimul nivel, h(T ) − 1, putem avea s, i noduri externe, iar nodurile interne de pe
acest nivel au de asemenea câte doi descendent, i;

• pe ultimul nivel, h(T ), avem doar noduri externe;

• pentru orice i ∈ {0, 1, . . . , h(T ) − 1}, pe nivelul i avem exact 2i noduri.

Notând cu a numărul de noduri interne de pe penultimul nivel s, i cu b numărul de noduri externe


de pe acelas, i nivel, obt, inem că
a + b = 2h(T )−1 (2)
(numărul total de noduri de pe penultimul nivel, nivelul h(T ) − 1), iar a ≥ 1 (pe penultimul
nivel există cel put, in un nod intern).
Pe nivelele 0, 1, . . . , h(T ) − 2 nu avem noduri externe, pe penultimul nivel, h(T ) − 1, avem b
noduri externe, iar pe ultimul nivel, h(T ), avem 2a noduri externe (s, i anume descendent, ii celor
a noduri interne de pe penultimul nivel), deci numărul total de noduri externe ale arborelui este
egal cu b + 2a. Rezultă că
b + 2a = n + 1 (3)
s, i X
DT (v) = b(h(T ) − 1) + 2a · h(T ). (4)
v∈E(T )

Din relat, iile (2) s, i (3) obt, inem că

a = n + 1 − 2h(T )−1 . (5)

Dar 1 ≤ a ≤ 2h(T )−1 (a doua inegalitate rezultă din (2)), deci 1 ≤ n + 1 − 2h(T )−1 ≤ 2h(T )−1 .
Rezultă că 2h(T )−1 ≤ n < n + 1 s, i 2h(T ) ≥ n + 1, deci h(T ) − 1 < log2 (n + 1) ≤ h(T ) s, i astfel
h(T ) = dlog2 (n + 1)e .
Conform (4), (3) s, i (5) obt, inem că
X
DT (v) = (n + 1 − 2a)(h(T ) − 1) + 2a · h(T ) = (n + 1)h(T ) + 2a − n − 1
v∈E(T )
Ä ä
= (n + 1)h(T ) + 2 n + 1 − 2h(T )−1 − n − 1 = (n + 1)h(T ) + n + 1 − 2h(T ) ,

de unde rezultă a doua egalitate din enunt, .

Propozit, ia 6. Pentru orice arbore binar strict T cu n noduri interne, n ∈ N, avem

DT (v) ≥ (n + 1) dlog2 (n + 1)e + n + 1 − 2dlog2 (n+1)e .


X

v∈E(T )

Mai mult, egalitatea are loc dacă s, i numai dacă toate nodurile externe sunt situate pe ultimul s, i,
eventual, pe penultimul nivel.
Optimalitatea algoritmului de căutare binară 45

Demonstraţie. Fie M = {T | T = arbore binar strict, card(I(T )) = n} s, i S(T ) = DT (v),


P
v∈E(T )
∀ T ∈ M. Fie T ∗ ∈ M, T ∗ = (V, F ), astfel ı̂ncât

S(T ∗ ) = min{S(T ) | T ∈ M}.

Considerăm din nou că nivelul pe care este reprezentată rădăcina arborelui este numerotat cu 0.
Demonstrăm că arborele T ∗ nu are noduri externe pe nivelele 0,1, . . ., h(T ∗ ) − 2 prin reducere la
absurd. Să presupunem că arborele T ∗ ar avea un nod extern x pe un nivel i, cu i ≤ h(T ∗ ) − 2.
Fie y un nod intern al lui T ∗ situat pe penultimul nivel, nivelul h(T ∗ ) − 1, s, i fie y1 s, i y2 cei doi
descendent, i ai lui y, deci y1 s, i y2 sunt noduri externe situate pe ultimul nivel, nivelul h(T ∗ ).
Evident, DT ∗ (x) = i, DT ∗ (y) = h(T ∗ ) − 1, DT ∗ (y1 ) = DT ∗ (y2 ) = h(T ∗ ).
Fie T 0 arborele obt, inut din T ∗ prin mutarea nodurilor y1 s, i y2 de la descendent, i ai nodului y
la descendent, i ai nodului x, adică

T 0 = (V, F \ {[y, y1 ], [y, y2 ]} ∪ {[x, y1 ], [x, y2 ]}).

Evident, T 0 rămâne un arbore binar strict, T 0 ∈ M, E(T 0 ) = E(T ∗ ) \ {x} ∪ {y},


DT 0 (y1 ) = DT 0 (y2 ) = i + 1, DT 0 (v) = DT ∗ (v), ∀ v ∈ E(T 0 ) \ {y1 , y2 }. Avem S(T 0 ) − S(T ∗ ) =
DT 0 (v) − DT ∗ (v) = DT 0 (y) + DT 0 (y1 ) + DT 0 (y2 ) − DT ∗ (x) − DT ∗ (y1 ) − DT ∗ (y2 ) =
P P
v∈E(T 0 ) v∈E(T ∗ )
h(T ) − 1 + 2(i + 1) − i − 2h(T ∗ ) = i + 1 − h(T ∗ ) < 0, deci S(T 0 ) < S(T ∗ ), ceea ce contrazice

alegerea lui T ∗ . Demonstrat, ia prin reducere la absurd este ı̂ncheiată.


Deoarece arborele T ∗ nu are noduri externe pe nivelele 0,1, . . ., h(T ∗ ) − 2, rezultă că
toate nodurile sale externe sunt situate pe ultimul s, i, eventual, pe penultimul nivel. Aplicând
Propozit, ia 5 rezultă că

min{S(T ) | T ∈ M} = s(T ∗ ) = (n + 1) dlog2 (n + 1)e + n + 1 − 2dlog2 (n+1)e ,

de unde rezultă inegalitatea din enunt, . Mai mult, dacă toate nodurile externe sunt situate pe
ultimul s, i, eventual, pe penultimul nivel atunci, conform Propozit, iei 5, inegalitatea din enunt,
devine egalitate, iar ı̂n caz contrar, conform demonstrat, iei prin reducere la absurd de mai sus,
inegalitatea este strictă.

Căutarea binară

Considerăm problema căutării unei valori date x printre elementele unui vector sortat crescător
dat A = (a1 , a2 , . . . , an ), a1 ≤ a2 ≤ . . . ≤ an , n ∈ N∗ .
Algoritmul de căutare binară rezolvă această problemă prin următoarea strategie de tip
Divide et Impera:
ö ù
n+1
• Se compară x cu elementul median am , unde m = 2
;

• Avem trei variante posibile:

– Dacă x = am , atunci căutarea se ı̂ncheie cu succes;


– Dacă x < am , atunci algoritmul continuă cu căutarea lui x ı̂n subvectorul (a1 , . . . , am−1 );
– Dacă x > am , atunci algoritmul continuă cu căutarea lui x ı̂n subvectorul (am+1 , . . . , an ).
46 C. Bălcău

Obt, inem astfel următoarea variantă iterativă a algoritmului:

CAUT-BIN(A, n, x) :
p ← 1; u ← n; // (ap , . . . , au ) reprezintă subvectorul curent
// la care s-a restrâns căutarea
while p ≤õ u do û
p+u
m← ;
2
if x = am then
return m; // căutarea se ı̂ncheie cu succes
else
if x < am then
u ← m − 1;
else
p ← m + 1;

return −1; // căutarea se ı̂ncheie fără succes

Observat, ia 4. Pentru evaluarea complexităt, ii algoritmilor care rezolvă problema considerată,


vom analiza numai comparat, iile ı̂n care intervin valoarea căutată x s, i un element al vectorului
A, numite comparat, ii de chei. Astfel considerăm că aceste comparat, ii se efectuează succesiv
(comparat, iile simultane pot fi separate).

Prezentăm câteva not, iuni ajutătoare.

Definit, ia 9. Un arbore binar de căutare (BST - Binary Search Tree) este un arbore
binar cu proprietatea că orice nod intern are valoarea (eticheta) mai mare decât orice nod din
subarborele său stâng si mai mică decât orice nod din subarborele său drept.

Observat, ia 5. Fie A un algoritm de căutare, bazat pe comparat, ii de chei, ce rezolvă problema


considerată. Presupunem că algoritmul nu cont, ine instruct, iuni redundante s, i este aplicabil
pentru orice pozit, ionare posibilă a valorii x ı̂n raport cu componentele vectorului A, deci valoarea
x este comparată exact câte o dată cu fiecare componemtă ai , i = 1, n.
Atunci algoritmului A i se poate asocia un arbore binar de căutare T (A) având n noduri,
etichetate cu numerele 1, 2, . . . , n, construit recursiv astfel: dacă prima comparat, ie efectuată de
algoritm este ı̂ntre valoarea căutată x s, i un element ai al vectorului, atunci:

• rădăcina arborelui este etichetată cu indicele i;

• subarborele stâng corespunde continuării algoritmului ı̂n cazul x < ai , dacă acest caz este
posibil după prima comparat, ie (altfel este vid, adică nu există descendent stâng);

• subarborele drept corespunde continuării algoritmului ı̂n cazul x > ai , dacă acest caz este
posibil după prima comparat, ie (altfel este vid, adică nu există descendent drept).

Reciproc, oricărui arbore binar de căutare T având n noduri, etichetate cu numerele


1, 2, . . . , n, ı̂i corespunde un algoritm de căutare A(T ) a valorii x ı̂n vectorul sortat
Optimalitatea algoritmului de căutare binară 47

A = (a1 , a2 , . . . , an ), descris recursiv astfel:


A(Tî ): ó
if T este vid then
return −1; // căutare fără succes
else
i ← eticheta rădăcinii lui T ;
if x = ai then
return i; // căutare cu succes
else
if x < ai then
Ts ← subarborele stâng al lui T ;
return A(Ts );
else
Td ← subarborele drept al lui T ;
return A(Td );

Corespondent, ele A → T (A) s, i T → A(T ) de mai sus sunt bine definite s, i inverse una
celeilalte, deci sunt biject, ii ı̂ntre mult, imea algoritmilor de căutare (bazat, i pe comparat, ii de
chei) a unei valori x ı̂ntr-un vector sortat cu n componente (a1 , a2 , . . . , an ) s, i mult, imea arborilor
binari de căutare având n noduri etichetate cu numerele 1, 2, . . . , n.

Definit, ia 10. Arbore binar de căutare T (A) construit ı̂n observat, ia anterioară se numes, te s, i
arborele de decizie al algoritmului de căutare A.

Exemplul 2. Arborele binar de căutare asociat algoritmului căutării binare pentru n = 8 este
reprezentat ı̂n figura următoare.

4
2 6
1 3 5 7
8

Observat, ia 6. Arborele de căutare asociat algoritmului de căutare binară este un arbore binar,
nu neapărat strict.

Definit, ia 11. Fie T un arbore binar de căutare având n noduri, etichetate cu numerele
1, 2, . . . , n. Extindem arborele binar T la un arbore binar strict T astfel: pentru orice
k ∈ {1, 2, . . . , n}, nodul lui T având eticheta k este reprezentat ı̂ntr-un cerc s, i devine nod
intern ı̂n T prin aplicarea următoarelor două reguli:

• dacă nodul nu are descendent stâng, atunci i se adaugă un descendent stâng etichetat cu
k − 1 s, i reprezentat ı̂ntr-un pătrat;

• dacă nodul nu are descendent drept, atunci i se adaugă un descendent drept etichetat cu k
s, i reprezentat ı̂ntr-un pătrat.
48 C. Bălcău

Arborele T se numes, te arbore binar extins de căutare.

Exemplul 3. Arborele binar extins de căutare asociat algoritmului căutării binare pentru n = 8
este reprezentat ı̂n figura următoare.

4
2 6
1 3 5 7
0 1 2 3 4 5 6 8
7 8

Observat, ia 7. Pentru orice algoritm de căutare A a valorii x ı̂n vectorul sortat A = (a1 , a2 , . . . , an ),
arborele binar extins de căutare asociat T = T (A) este un arbore binar strict ce are n noduri
interne, etichetate cu numerele 1, 2, . . . , n reprezentate ı̂n cercuri s, i n + 1 noduri externe,
etichetate cu numerele 0, 1, 2, . . . , n reprezentate ı̂n pătrate, având semnificat, ia următoare:

• pentru orice i ∈ {1, 2, . . . , n}, nodul intern etichetat cu i reprezintă comparat, ia valorii
căutate, x, cu elementul ai din vectorul sortat;

• pentru orice i ∈ {0, 1, 2, . . . , n}, nodul extern etichetat cu i reprezintă ı̂ncheierea fără
succes a căutării, corespunzătoare situat, iei:

x < a1 , dacă i = 0,
ai < x < ai+1 , dacă 1 ≤ i ≤ n − 1,
x > an , dacă i = n.

De asemenea, avem:

• la o căutare cu succes, x = ai , numărul de comparat, ii de chei este egal cu numărul de


noduri (interne) situate pe lant, ul elementar dintre nodul rădăcină s, i nodul intern i, adică
este mai mare cu 1 decât lungimea acestui lant, ;

• la o căutare fără succes numărul de comparat, ii de chei este egal cu numărul de noduri
interne situate pe lant, ul elementar dintre nodul rădăcină s, i nodul extern corespunzător
ı̂ncheierii fără succes a căutării, adică cu lungimea acestui lant, .

Teorema 2 (complexitatea algoritmilor de căutare). Pentru orice algoritm de căutare A,


bazat pe comparat, ii de chei, a valorii x ı̂n vectorul sortat A = (a1 , a2 , . . . , an ) avem:
1) numărul de comparat, ii de chei efectuate ı̂n cazul cel mai defavorabil este mai mare sau
egal cu dlog2 (n + 1)e ;
2) numărul mediu de comparat, ii de chei efectuate este mai mare sau egal cu

2n + 2 n + 2 − 2 · 2dlog2 (n+1)e
· dlog2 (n + 1)e + .
2n + 1 2n + 1
Optimalitatea algoritmului de căutare binară 49

Demonstraţie. 1) Analizăm cazul cel mai defavorabil. Conform Observat, iei 7, numărul de
comparat, ii de chei efectuate ı̂n cazul cel mai defavorabil este egal cu ı̂nălt, imea h a arborelui
binar extins de căutare asociat T = T (A). Arborele T este un arbore binar strict, cu n noduri
interne (s, i n + 1 noduri externe), deci conform Propozit, iei 4 rezultă că h ≥ dlog2 (n + 1)e .
2) Analizăm acum cazul mediu (cazul timpului mediu de execut, ie). Conform Observat, iei 7,
numărul mediu de comparat, ii de chei are valoarea

(1 + D(k)) + D(k) n+ D(k) + D(k)


P P P P
k∈I k∈E k∈I k∈E
Nmed (n) = = ,
2n + 1 2n + 1
unde I este mult, imea nodurilor interne, E este mult, imea nodurilor externe, iar, pentru fiecare
nod k, D(k) reprezintă distant, a de la rădăcina arborelui T = T (A) la nodul k. Conform
Propozit, iilor 3 s, i 6 rezultă că

2 D(k) − n
P Ä ä
k∈E 2 (n + 1) dlog2 (n + 1)e + n + 1 − 2dlog2 (n+1)e − n
Nmed (n) = ≥ ,
2n + 1 2n + 1
adică inegalitatea din enunt, .

Propozit, ia 7. Pentru algoritmul de căutare binară ı̂ntr-un vector sortat cu n componente, o


căutare cu succes necesită cel mult h comparat, ii de chei, iar o căutare fără succes necesită h − 1
sau h comparat, ii de chei, unde h = dlog2 (n + 1)e = blog2 nc + 1.

Demonstraţie. Fie T (n) arbore binar de căutare asociat algoritmului de căutare binară ı̂n-
tr-un vector sortat cu n componente. Conform descrierii algoritmului de căutare binară
s, i a construirii arborelui binar de căutare asociat, rezultă că subarborele ö ùstâng al lui T (n)
corespunde căutării binare ı̂n subvectorul (a1 , . . . , am−1 ), unde m = n+1 2
, deci este chiar
Äö ùä
n−1
T 2
, iar subarborele drept corespunde căutării binare ı̂n subvectorul (am+1 , . . . , an ) de
ö ù † £ Ć £ä
lungime n − m = n − n+1 2
= n−1 2
, deci are aceeas, i structură cu T n−1 2
(doi arbori
binari au aceeas, i structură dacă sunt identici, cu except, ia etichetelor, adică prin suprapunerea
rădăcinilor ei se suprapun perfect). Aplicând metoda induct, iei matematice, rezultă că arborele
binar T (n + 1) are aceeas, i structură cu arborele binar T (n) plus un nod extern, situat pe
ultimul nivel (la pasul inductiv se analizează separat cazurile † £ nö par,ù respectiv impar s, i se aplică
ipoteza de induct, ie pentru cei doi subarbori). Cum n−1 2
= n−1 2
pentru n impar, respectiv
† £ ö ù
n−1 n−1
2 Ć
= 2 + 1 pentru n par, rezultă că subarborele drept al lui T (n), având aceeas, i structură
£ä Äö ùä
cu T n−1 2
, are aceeas
, i structură cu subarborele stâng T n−1
2
plus un eventual nod extern
(când n este par). Aplicând din nou metoda induct, iei matematice, rezultă că ı̂n arborele binar
T (n) toate nodurile care nu au doi descendent, i (fii) sunt situate pe ultimul s, i, eventual, pe
penultimul nivel (la pasul inductiv se foloses, te relat, ia de structură ment, ionată dintre cei doi
subarbori s, i se aplică ipoteza de induct, ie pentru subarborele drept).
Rezultă că arborele binar extins de căutare asociat T (n) are nodurile externe situate pe
cel mult două nivele. Notând cu h ı̂nălt, imea arborelui extins (adică lungimea maximă de la
rădăcină la un nod extern) s, i considerând că rădăcina arborelui este reprezentată pe nivelul 0,
rezultă că: arborele extins are h + 1 nivele, n noduri interne s, i n + 1 noduri externe; pe nivelele
0, 1, . . . , h − 2 avem doar noduri interne, fiecare având câte doi descendent, i; pe penultimul nivel,
h − 1, putem avea s, i noduri externe, iar nodurile interne de pe acest nivel au de asemenea câte
doi descendent, i; pe ultimul nivel, h, avem doar noduri externe.
50 C. Bălcău

Conform Propozit, iei 5 s, i Observat, iei 3 rezultă că h = dlog2 (n + 1)e = blog2 nc + 1. Conform
Observat, iei 7 rezultă că o căutare cu succes necesită cel mult h comparat, ii de chei, iar o căutare
fără succes necesită h − 1 sau h comparat, ii de chei.

Observat, ia 8. Conform propozit, iei anterioare, algoritmul de căutare binară are timpul de execut, ie
ı̂n cazul cel mai defavorabil de ordinul Θ(log2 n) (celelalte operat, ii nu depăs, esc ordinul de cres, tere
al comparat, iilor de chei), deci timpul (mediu) de execut, ie are ordinul O (log2 n). Teorema
următoare ı̂mbunătăt, es, te acest rezultat.

Teorema 3. Fie T (n) timpul mediu de execut, ie al algoritmului de căutare binară ı̂ntr-un vector
sortat cu n componente. Avem T (n) = Θ(log2 n).

Demonstraţie. Considerăm că timpul mediu de execut, ie T (n) al algoritmului de căutare binară
este dat de numărul de comparat, ii de chei efectuate (celelalte operat, ii nu depăs, esc ordinul de
cres, tere al acestora). Vom demonstra că T (n) = Θ(log2 n) prin două metode.
Metoda 1) Vom aplica Teorema Master. Conform descrierii recursive a algoritmului de
căutare binară, timpul de execut, ie T (n) verifică relat, ia de recurent, ă

T (n) = T (n/2) + O (1), ∀ n > 0,

cu convent, iile de notat, ie din Teorema Master. Conform Cazului 2 al acelei teoreme, pentru
a = 1, b = 2, c = 0 = logb a s, i k = 0, rezultă că T (n) = Θ(log2 n).
Observat, ia 9. Ca o consecint, ă imediată a Metodei 1, Propozit, iei 7 s, i Teoremei 2 rezultă că
algoritmul de căutare binară este asimptotic optim (ı̂n clasa algoritmilor bazat, i pe comparat, ii
de chei, atât ı̂n raport cu timpul de execut, ie ı̂n cazul cel mai defavorabil, cât s, i ı̂n raport cu
timpul mediu de execut, ie). Pentru a justifica faptul că algoritmul este chiar optim, nu este
suficientă evaluarea complexităt, ii cu Teorema Master. Aici intervine metoda următoare, prin
care se numără comparat, iile efectuate de algoritm.

Metoda 2) Vom calcula efectiv timpul mediu de execut, ie T (n). Conform demonstrat, iei
Propozit, iei 7, arborele binar extins de căutare asociat algoritmului căutării binare este un arbore
binar strict cu n noduri interne s, i n + 1 noduri externe, iar toate nodurile externe sunt situate
pe ultimul s, i, eventual, pe penultimul nivel. Notăm cu I mult, imea nodurilor interne, cu E
mult, imea nodurilor externe, iar pentru fiecare nod k notăm cu D(k) distant, a de la rădăcina
arborelui la nodul k. Conform Observat, iei 7, timpul mediu de execut, ie are valoarea

(1 + D(k)) + D(k) n+ D(k) + D(k)


P P P P
k∈I k∈E k∈I k∈E
T (n) = = .
2n + 1 2n + 1
Conform Propozit, iilor 3 s, i 5 rezultă că

2 D(k) − n
P
k∈E 2n + 2 n + 2 − 2 · 2dlog2 (n+1)e
T (n) = = · dlog2 (n + 1)e + . (6)
2n + 1 2n + 1 2n + 1

Dar, conform Observat, iei 3, dlog2 (n + 1)e = 1 + blog2 nc , deci

2n + 2 3n + 4 − 4 · 2blog2 nc
T (n) = · blog2 nc + .
2n + 1 2n + 1
Optimalitatea algoritmului de căutare binară 51

n
Evident, log2 n − 1 < blog2 nc ≤ log2 n, deci < 2blog2 nb ≤ n. Rezultă că
2
3n − 2 − log2 n n + 4 + log2 n
log2 n − < T (n) < log2 n + , deci
2n + 1 2n + 1
3n − 2 − log2 n T (n) n + 4 + log2 n
1− < <1+ , ∀ n ≥ 2.
(2n + 1) log2 n log2 n (2n + 1) log2 n

Aplicând Criteriul cles, telui rezultă că

T (n)
lim = 1, (7)
n→∞ log2 n

deci conform Propozit, iei 1 obt, inem că T (n) = Θ(log2 n).

Mai mult, conform relat, iei (7) am obt, inut următorul rezultat.

Corolarul 2. Numărul mediu de comparat, ii de chei efectuate de algoritmul de căutare binară


ı̂ntr-un vector sortat cu n componente este asimptotic echivalent cu log2 n.

Teorema 4 (optimalitatea algoritmului de căutare binară). Algoritmul de căutare binară


este optim (ı̂n clasa algoritmilor bazat, i pe comparat, ii de chei), atât ı̂n raport cu timpul de execut, ie
ı̂n cazul cel mai defavorabil, cât s, i ı̂n raport cu timpul mediu de execut, ie.

Demonstraţie. Optimalitatea ı̂n raport cu timpul de execut, ie ı̂n cazul cel mai defavorabil este o
consecint, ă directă a Propozit, iei 7 s, i a primei părt, i din Teorema 2. Optimalitatea ı̂n raport cu
timpul mediu de execut, ie este o consecint, ă directă a relat, iei (6) din demonstrat, ia Teoremei 3 s, i
a părt, ii secunde din Teorema 2.

Bibliografie

[1] Gh. Barbu, I. Văduva, M. Bolos, teanu, Bazele informaticii, Editura Tehnică, Bucures, ti, 1997.

[2] A. Carabineanu, Structuri de date, http://ebooks.unibuc.ro/informatica/carabineanu/CARA STR.pdf.

[3] T.H. Cormen, C.E. Leiserson, R.L. Rivest, C. Stein, Introduction to Algorithms, MIT Press,
Cambridge, 2009.

[4] H. Georgescu, Tehnici de programare, Editura Universităt, ii din Bucures, ti, Bucures, ti, 2005.

[5] D.E. Knuth, The Art Of Computer Programming. Vol. 4A: Combinatorial Algorithms,
Addison-Wesley, Massachusetts, 2011.

[6] R. Sedgewick, P. Flajolet, An Introduction to the Analysis of Algorithms, Addison-Wesley,


New Jersey, 2013.
Perspective profesionale pentru pasionat, ii de programare

1
ROWEB Development

În contextul actual al ı̂nvăt, ământului superior din România numărul de absolvent, i de
Informatică este de aproximativ 7000 pe an, iar necesarul de pe piat, a muncii este mult mai
mare. Comisia Europeană estimează că, până ı̂n anul 2020, Europa s-ar putea confrunta cu
un deficit de până la 825.000 de profesionis, ti ı̂n IT&C, dintre care aproximativ 300.000 vor fi
necesari pentru a acoperi cres, terea prognozată a domeniului de software din România.
Discrepant, a dintre cerere s, i ofertă reprezintă deja o provocare, pe care atât companiile
producătoare de software, cât s, i universităt, ile ı̂ncearcă să o rezolve, adesea ı̂n colaborare. Unele
facultăt, i de informatică ı̂s, i dublează deja de la un an la altul numărul de locuri, iar unele
companii oferă training sau colaborează cu universităt, ile pentru a-s, i asigura viitori angajat, i.
O altă provocare o reprezintă potrivirea dintre limbajele de programare predate la nivelul
liceelor s, i facultăt, ilor, s, i tehnologiile cerute pe piat, ă. Stagiile de vară, internship-urile ı̂n cadrul
companiilor cu experient, ă, precum s, i concursurile de programare urmăresc ı̂mbunătăt, irea acestui
aspect. Infoarena reunes, te informat, ii despre cele mai importante concursuri, inclusiv:
a) .campion - un program de pregătire de performant, ă ı̂n informatică sust, inut de profesori de
prestigiu din t, ară, membri ai Comisiei Nat, ionale de Informatică, s, i de student, i - fos, ti olimpici
internat, ionali la informatică;
b) TopCoder - un concurs ce pune accent ı̂n primul rând pe viteză s, i acuratet, e, apoi pe
dificultate;
c) IPSC - Internet Problem Solving Contest - cel mai prestigios concurs online pe echipe
s, i totodată primul concurs care nu pune accentul pe surse de solut, ii, ci pe output-uri pentru
testele publicate;
d) Waterloo Programming Contests - concursuri de pregătire pentru ACM organizate de
către Universitatea Waterloo din Canada;
e) ACM - University of Vallaloid, ACM - Peking University s, i ACM - Zhejiang University
organizează concursuri lunare;
f) ACM ICPC International Collegiate Programming Contest - cel mai prestigios concurs de
programare/algoritmică pentru universităt, i;
g) CodeCup - Concurs anual organizat de către The Netherlands Olympiad in Informatics;
h) Bubble Cup - Concurs organizat ı̂n Serbia, cu sprijinul Microsoft Development Center
Serbia.
Concursurile stimulează student, ii să ı̂s, i dezvolte un nivel ı̂nalt al cunos, tint, elor s, i abilităt, ilor
ı̂n programare, ı̂nsă pentru a-i ajuta să-s, i facă o idee asupra piet, ei muncii ı̂n care urmează să
lucreze, este nevoie de implicarea companiilor.
1
vcostea@roweb.ro, catalina.radu@roweb.ro

52
Perspective profesionale... 53

ROWEB este un exemplu de companie care investes, te ı̂n astfel de eforturi, remarcându-se ı̂n
Pites, ti printr-o colaborare cu Universitatea ce durează de peste 12 ani. Fondatorii ROWEB au
absolvit Universitatea din Pites, ti s, i ı̂ncă din primii ani de la ı̂nfiint, are au atras tineri student, i
s, i absolvent, i talentat, i, care s-au bucurat de aparit, ia unei firme care le permitea să-s, i urmeze
pasiunea pentru programare fără să fie nevoie să părăsească Pites, ti-ul ı̂n favoarea unui centru
de software mai mare din t, ară.
Colaborarea dintre ROWEB s, i UPIT a adus beneficii comunităt, ii universitare locale s, i
a ajutat multe tinere talente să aibă mai us, or acces la o carieră ca programatori. Această
colaborare s-a concretizat prin workshop-uri gratuite de Android, HTML s, i PHP, sust, inere ı̂n
jurizarea unor concursuri de programare, precum s, i stagii de vară de care au beneficiat sute de
student, i. Stagiile se desfăs, oară cu succes ı̂ncă de acum 9 ani s, i ı̂n fiecare an ajută câteva zeci
de student, i sa se familiarizeze cu dezvoltarea de aplicat, ii ı̂n .NET, PHP sau cu dezvoltarea de
aplicat, ii mobile. Mai mult, i stagiari au rămas ulterior ca angajat, i full-time, aproximativ 90%
din angajat, ii firmei fiind fos, ti stagiari.
Stagiile de vară permit student, ilor să lucreze ı̂mpreună cu dezvoltatorii seniori, foarte
prietenos, i, entuzias, ti să-s, i ı̂mpărtăs, ească experient, a s, i cunos, tint, ele, s, i să pună ı̂n practică
cunos, tint, ele acumulate la facultate, căpătând nu doar experient, ă profesională, ci s, i training
intensiv din partea unor specialis, ti care au o reputat, ie foarte bună pentru expertiza lor ı̂n cele
mai recente tehnologii s, i pentru aderarea la cele mai bune practici de lucru.
Un stagiu de practică la ROWEB reprezintă o oportunitate de pregătire a student, ilor pentru
intrarea pe piat, a muncii, dar ı̂n acelas, i timp le oferă acestora s, ansa de a pune ı̂n practică toate
cunos, tint, ele dobândite ı̂n anii de studiu, ajutându-i să se familiarizeze cu cerint, ele concrete din
piat, a de IT.
Însă ı̂n IT nu este suficient să dezvolt, i noi talente s, i să atragi proaspet, ii absolvent, i. Retent, ia
angajat, ilor a devenit o provocare ı̂n ultimii ani, ı̂n condit, iile disparităt, ii dintre cerere s, i oferta
de muncă. Salariul nu mai este de mult principalul diferent, iator pe care ı̂l are ı̂n vedere un
candidat bun când alege compania pentru care să lucreze. Acesta ajunge acum să se uite la
alte aspecte, cum sunt cele legate de cultura organizat, ională, dezvoltare de carieră s, i personală,
de posibilitatea unui program de lucru flexibil, posibilitatea de a lucra de acasă etc. Tinerii
programatori ı̂s, i doresc s, i au posibilitatea să aleagă o companie care să le ı̂mpărtăs, ească valorile
s, i să le asigure transparent, a s, i un mediu de lucru plăcut, ı̂n care ideile să le fie ascultate s, i
apreciate.
ROWEB are ı̂n vedere aceste aspecte prin posibilitatea de a se realoca dintr-o linie de business
ı̂n alta prin intermediul programului de mobilitate internă, acces la cursuri de programare,
de limba engleză, la o bibliotecă de specialitate s, i prin dezvoltarea s, i păstrarea unei culturi
organizat, ionale etice s, i incluzive, care asigură angajat, ilor un echilibru ı̂ntre viat, a personală s, i
profesională s, i beneficii care le permit să progreseze profesional s, i să ı̂s, i ı̂ntemeieze s, i sust, ină
familii ı̂n acelas, i timp, ceea ce reprezintă o diferent, ă importantă mai ales pentru programatoare,
care ı̂n alte companii sunt adesea nevoite sa aleagă ı̂ntre evolut, ia profesională s, i decizia de a
avea copii.
Alte criterii importante sunt amploarea s, i complexitatea proiectelor, accentul pus mai
degrabă pe proiecte de dezvoltare, care implică tehnologii noi, decât pe cele de mentenant, ă, ceea
ce permite angajat, ilor să ı̂nvet, e ı̂n permanent, ă s, i să aibă de-a face cu provocări noi ı̂n domeniu.
În condit, iile ı̂n care, conform estimărilor ANIS, anul acesta industria de software s, i servicii
IT din România va reprezenta 3% din PIB, iar nevoia de a forma specialis, ti ı̂ntr-un ritm care să
54 ROWEB Development

t, ină pasul cu cel al cererii exprimate este din ce ı̂n ce mai evidentă ı̂n piat, ă, tinerii care aleg
specializarea informatică au provocări tot mai mari ı̂n fat, ă, trebuie să se adapteze rapid s, i să fie
mereu la curent cu cele mai noi tehnologii pentru a fi competitivi, dar ı̂n acelas, i timp beneficiază
de posibilităt, i tot mai mari de dezvoltare, de compensare salarială s, i de a alege angajatorul care
le ı̂ndeplines, te cel mai bine cerint, ele.
MATEMATICĂ PENTRU PROGRAMATORI S, I
PROGRAMARE PENTRU MATEMATICIENI

Rezolvarea sistemelor de ecuat, ii liniare

1
Laurent, iu Deaconu

Ne propunem să realizăm o aplicat, ie (un program) care să rezolve un sistem de ecuat, ii liniare.
Nu insistăm asupra sistemelor de ecuat, ii liniare compatibile determinate (pentru care rangul
matricei sistemului este egal cu rangul matricei extinse s, i cu numărul de necunoscute) pentru
care există multe metode simple de rezolvare (inclusiv metoda Cramer) care se pot transfera ı̂n
algoritmi informatici, ci prezentăm o metodă care permite atât stabilirea compatibilităt, ii cât s, i
rezolvarea sistemelor compatibile nedeterminate.

Argumentare matematică

Considerăm sistemul de ecuat, ii liniare


a11 x1 + a12 x2 +· · · + a1n xn =b1



 a21 x1 + a22 x2 +· · · + a2n xn =b2


.. , (1)




.
am1 x1 +am2 x2 +· · · +amn xn =bm

cu aij , bi ∈ R, i ∈ 1, m, j ∈ 1, n, m, n ∈ N? .
Algoritmul folosit pentru rezolvarea sistemului are la bază lema substitut, iei. Pentru simplifi-
carea calculelor vom organiza rezolvarea transferând sistemul ı̂ntr-un tabel. În mod normal,
dacă sistemul este compatibil, algoritmul are m pas, i. La pasul k (0 ≤ k ≤ m) tabelul are
următoarea structură:
x1 x2 ... xl ... xn
[k] [k] [k]
j1 j2 ... jl ... jn[k]
[k] [k] [k] [k] [k] [k]
i1 a11 a12 ... a1l ... a1n b1
[k] [k] [k] [k] [k] [k]
i2 a21 a22 ... a2l ... a2n b2
.. .. .. .. .. .. .. ..
. . . . . . . .
[k] [k] [k] [k] [k] [k]
ik ak1 ak2 ... akl ... akn bk
[k] [k] [k] [k] [k]
0 ak+1,1 ak+1,2 ... ak+1,l ... ak+1,n bk+1
.. .. .. .. .. .. .. ..
. . . . . . . .
[k] [k] [k] [k] [k]
0 am 0 1 am 0 2 ... am 0 l ... am 0 n bm 0
1
Lect. univ. dr., Universitatea din Piteşti, laurentiu.deaconu@upit.ro

55
56 L. Deaconu

Elementele tabelului vor fi precizate ı̂n cadrul algoritmului.


Pasul 1◦ Generarea tabelului. Primul tabel se completează astfel:
1. Prima linie x1 , x2 , . . . , xn este doar o notat, ie pentru coloanele asociate necunoscutelor
sistemului. Ea nu se modifică pe parcursul algoritmului.
2. A doua linie j1 , j2 , . . . , jn se completează cu 0, jl = 0, ∀l ∈ 1, n. Ea se completează cu
elemente nenule la fiecare pas.
3. Coloana din stânga cu elementele i1 , i2 , . . . , im0 are init, ial toate elementele nule, ik = 0,
∀k ∈ 1, m0 . La primul pas avem m0 = m. Se completează cu elemente nenule la fiecare
pas al algoritmului.
4. Zona centrală cont, ine elementele matricei sistemului. La primul tabel (k = 0) avem
[0]
aij = aij , ∀i ∈ 1, m, ∀j ∈ 1, n.
[0]
5. Coloana din dreapta se completează cu termenii liberi ai sistemului. Avem bi = bi ,
∀i ∈ 1, m.
Pasul 2◦ Prelucrarea tabelului. La pasul k, (k ∈ 1, m0 ), parcurgem următoarele etape:
[k−1] [k−1] [k−1] [k−1]
1. Căutăm, ı̂n linia ak1 , ak2 , . . . , akl , . . . , akn din zona centrală, un element nenul.
Dacă găsim, ı̂l numim pivot, identificăm coloana ı̂n care se află (notată xl ı̂n tabelul de
[k−1]
mai sus) s, i trecem la pasul următor. Dacă avem akq = 0, ∀q ∈ 1, n, atunci:
[k−1]
(a) dacă bk = 0 eliminăm linia k deoarece corespunde unei ecuat, ii care este o
combinat, ie liniară a ecuat, iilor precedente s, i care nu contează ı̂n rezolvarea sistemului.
În acest caz numărul ecuat, iilor se mics, orează cu 1, m0 ← m0 − 1. Dacă avem k ≤ m0 ,
reluăm etapa 1, altfel trecem la Pasul 3◦ .
[k−1]
(b) dacă bk 6= 0 algoritmul se opres, te. Sistemul este incompatibil.
[k]
2. În coloana din stânga, punem ik = l, unde xl este coloana ı̂n care a fost identificat pivotul.
Celelalte elemente rămân ca la pasul precedent, i[k] p = ip
[k−1]
, ∀p ∈ {1, 2, . . . , m0 } \ {k}.
[k]
3. În linia a doua, punem jl = k s, i jq[k] = jq[k−1] , ∀q ∈ {1, 2, . . . , n} \ {l}.
4. Elementele din linia care cont, ine pivotul se ı̂mpart la pivot:
[k−1] [k−1]
[k] akq [k] bk
akq = [k−1]
, ∀q ∈ 1, n, bk = [k−1]
.
akl akl
[k]
Evident akl = 1.
[k]
5. Celelalte elemente din coloana xl , ı̂n care se află pivotul, vor fi egale cu 0, apl = 0,
∀p ∈ {1, 2, . . . , m0 } \ {k}.
6. Toate elementele care nu au fost ı̂nlocuite ı̂n etapele 2-5 se determină folosind regula
dreptunghiului:
[k−1] [k−1] [k−1] [k−1] [k−1]
a[k−1]
pq akl − akq apl akq apl
a[k]
pq = [k−1]
= a[k−1]
pq − [k−1]
,
akl akl

∀p ∈ {1, 2, . . . , m0 } \ {k}, ∀q ∈ {1, 2, . . . , n} \ {l}.


7. Dacă k < m0 , cres, tem valoarea lui k cu 1 s, i reluăm Pasul 2◦ , altfel trecem la Pasul 3◦ .
Pasul 3◦ Recuperarea solut, iei. Sistemul este compatibil:
[m0 ]
1. determinat, dacă m0 = n. Solut, ia este xl = b [m0 ] , 1 ≤ l ≤ n.
jl
Rezolvarea sistemelor de ecuat, ii liniare 57

[m0 ]
2. nedeterminat, dacă m0 < n. Construim mult, imea S = {l ∈ N | jl = 0, 1 ≤ l ≤ n}.
[m0 ] [m0 ] 0
Parcurgem a doua linie, j1 , j2 , . . . , jn[m ] :
[m0 ]
(a) dacă jl = 0, 1 ≤ l ≤ n, atunci xl este necunoscută secundară s, i avem xl = αl , unde
αl ∈ R este un parametru.
[m0 ] [m0 ] X [m0 ]
(b) dacă jl 6= 0, 1 ≤ l ≤ n, atunci xl = b [m0 ] − a [m0 ] αq .
jl jl q
q∈S

Exemplu de calcul

Considerăm sistemul 


 x1 +2x2 +3x3 + 4x4 + 5x5 = 6


2x1 + x2 +3x3 + 2x4 + x5 = 8


, (2)



 x1 +5x2 +6x3 +10x4 +14x5 =10



2x1 + x2 +6x3 + 5x4 + 4x5 = 3

Pentru rezolvare utilizăm algoritmul prezentat mai sus. Mai ı̂ntâi scriem tabelul init, ial.
x1 x2 x3 x4 x5
0 0 0 0 0
0 1 2 3 4 5 6
0 2 1 3 2 1 8
0 1 5 6 10 14 10
0 2 1 6 5 4 3

În linia corespunzătoare primei ecuat, ii toate elementele sunt nenule, deci oricare element
poate fi ales ca pivot. Pentru a evita calculele complicate, alegem ca pivot primul element, pe 1,
elementul ı̂ncadrat ı̂ntr-un pătrat ı̂n tabelul anterior. Prelucrăm primul tabel:
– ı̂n coloana din stânga, ı̂n linia pivotului, punem 1 deoarece am ales pivotul ı̂n coloana lui x1 ;
restul elementelor se transcriu din primul tabel;
– ı̂n linia de deasupra zonei centrale, ı̂n coloana pivotului, sub x1 , punem 1 deoarece pivotul se
găses, te ı̂n linia 1 din zona centrală, linia corespunzătoare primei ecuat, ii; restul elementelor se
transcriu din primul tabel;
– linia pivotului se ı̂mparte la pivot, adică se transcrie deoarece pivotul este egal cu 1;
– coloana pivotului se completează cu zero-uri;
– restul elementelor se calculează cu regula dreptunghiului; de exemplu, elementul din linia 4 s, i
coloana 3 din zona centrală (care este 6 ı̂n primul tabel) se calculează astfel:
[1] 6·1−2·3
a43 = = 0,
1
6 este elementul corespunzător din tabelul anterior, 1 este pivotul, 2 s, i 3 sunt ”proiect, iile”
elementului ı̂nlocuit pe coloana, respectiv linia, pivotului. Termenii liberi se calculează ı̂n acelas, i
mod: elementul din linia 3 din coloana termenilor liberi se calculează astfel
[1] 10 · 1 − (1) · 6
b3 = = 4.
1

Obt, inem tabelul următor:


58 L. Deaconu

x1 x2 x3 x4 x5
1 0 0 0 0
1 1 2 3 4 5 6
0 0 −3 −3 −6 −9 −4
0 0 3 3 6 9 4
0 0 −3 0 −3 −6 −9

În linia corespunzătoare ecuat, iei 2 alegem pivotul −3 din coloana lui x2 . În coloana din
stânga, ı̂n linia pivotului punem 2 deoarece pivotul este ı̂n coloana lui x2 , iar ı̂n linia de deasupra
zonei centrale, sub x2 , punem 2 deoarece pivotul se găses, te ı̂n linia 2 din zona centrală. Restul
elementelor se completează ca la pasul precedent. Obt, inem tabelul:
x1 x2 x3 x4 x 5
1 2 0 0 0
1 1 0 1 0 −1 10 3
4
2 0 1 1 2 3 3
0 0 0 0 0 0 0
0 0 0 3 3 3 −5
Observăm că ı̂n linia 3 din zona centrală, ı̂n coloanele corepunzătoare necunoscutelor toate
elementele sunt egale cu 0, deci nu putem alege un pivot. Elementul corespunzător din coloana
termenilor liberi este tot 0, deci putem renunt, a la ecuat, ia 3. Eliminăm această linie s, i obt, inem
tabelul:
x1 x2 x3 x4 x5
1 2 0 0 0
1 1 0 1 0 −1 10 3
4
2 0 1 1 2 3 3
0 0 0 3 3 3 −5

În tabelul anterior alegem pivotul 3 din ultima linie s, i coloana lui x3 .
x1 x2 x3 x4 x5
1 2 3 0 0
1 1 0 0 −1 −2 5
2 0 1 0 1 2 3
3 0 0 1 1 1 − 53
Analizând ultimul tabel deducem că sistemul este compatibil nedeterminat. Necunoscu-
tele principale sunt x1 , x2 , x3 , iar necunoscutele secundare x4 s, i x5 . Asociem paramatri reali
necunoscutelor secundare: x4 = α, x5 = β.
Pentru simplificarea identificării solut, iei putem completa tabelul anterior astfel:
α β
x1 x2 x3 x4 x5
1 2 3 0 0
1 1 0 0 −1 −2 5
2 0 1 0 1 2 3
3 0 0 1 1 1 − 53
Rezolvarea sistemelor de ecuat, ii liniare 59

Pentru x1 , ı̂n linia de deasupra zonei centrale găsim 1. Din linia corespunzătoare, adică
linia 1 recuperăm x1 : din elementul din coloana termenilor liberi, 5, se scad produsele dintre
elementele din coloanele corespunzătoare parametrilor s, i respectivii parametri, adică −1 · α s, i
−2 · β. Procedăm similar pentru necunoscutele x2 s, i x3 . Obt, inem astfel solut, ia:




 x1 = 5 +α +2β

x = 3−α −2β


 2


x =− 35 −α −β ,
 3
(3)




 x4 = α



x5 = β

Implementare ı̂n limbajul C++

Algorimul prezentat ı̂n acest articol permite realizarea unui program care

• determină dacă un sistem de ecuat, ii liniare este compatibil sau incompatibil

• identifică dacă un sistem compatibil este compatibil determinat sau compatibil nedetermi-
nat

• pentru sistemele compatibile determinate calculează solut, ia

• pentru sistemele compatibile nedeterminate determină solut, ia generală, ı̂n funct, ie de


parametri.

Programul prezentat mai jos, realizat ı̂n Code::Blocks, cites, te datele din fis, ierul text
sistem.txt s, i scrie rezultatele ı̂n fis, ierul text solutie.txt.
Fis, ierul prezentat mai jos (sistem.txt) cont, ine datele corespunzătoare sistemului (2). Prima
linie cont, ine m, n, numărul ecuat, iilor, respectiv numărul necunoscutelor. În continuare se scriu
elementele matricei extinse de dimensiune m × (n + 1), ultima coloană cont, inând termenii liberi.

4 5
1 2 3 4 5 6
2 1 3 2 1 8
1 5 6 10 14 10
2 1 6 5 4 3

Programul respectă algoritmul prezentat, cu următoarele precizări:


– Pentru alegerea pivotului se determină elementul din linia curentă care are valoarea absolută
maximă. Dacă acesta este nul, deducem că toate elementele liniei sunt nule. Se analizează Pasul 2◦ ,
etapa 1 (a) s, i (b). Dacă elementul determinat este nenul, acesta va fi pivotul (se alege elementul cu
valoare absolută maximă pentru reducerea erorilor de rotunjire).
– Pentru a verifica dacă un număr real este nul, nu ı̂l comparăm cu 0 ci verificăm dacă valoarea
absolută a lui este mai mică decât ε, unde ε este un număr real pozitiv foarte mic. În program
ε = 10−20 .
– Prelucrarea tabelului din Pasul 2◦ se face parcurgând etapele ı̂n ordinea 6, 4, 5, 3, 2.
60 L. Deaconu

#include <iostream>
#include <fstream>
#include <iomanip>
#include <math.h>
using namespace std;
ifstream f1("sistem.txt");
ofstream f2("solutie.txt");
int m,n,io,jo;
double a[20][40],eps=1.0e-20;
int o[40],v[20];
void citire()
{
int i,j;
f1>>m>>n;
for(i=1;i<=m;i++)
for(j=1;j<=n+1;j++)
f1>>a[i][j];
f1.close();
}
void generare_tabel()
{
int i,j;
for(i=1;i<=m;i++)
v[i]=0;
for(j=1;j<=n+1;j++)
o[j]=0;
}
void prelucrare()
{
int i,j;
for(i=1;i<=m;i++)
if(i!=io)
for(j=1;j<=n+1;j++)
if(j!=jo)
a[i][j]=(a[i][j]*a[io][jo]-a[i][jo]*a[io][j])/a[io][jo];
for(j=1;j<=n+1;j++)
if(j!=jo)
a[io][j]/=a[io][jo];
for(i=1;i<=m;i++)
a[i][jo]=0.0;
a[io][jo]=1.0;
o[jo]=io;
v[io]=jo;
}
int pivot()
{
int i,j;
double max;
max=fabs(a[io][1]);
jo=1;
for(j=2;j<=n;j++)
if(max<=fabs(a[io][j]))
{
max=fabs(a[io][j]);
jo=j;
}
if(max<eps)
{
if(fabs(a[io][n+1])<eps)
Rezolvarea sistemelor de ecuat, ii liniare 61

{
for(i=io;i<m;i++)
for(j=1;j<=n+1;j++)
a[i][j]=a[i+1][j];
m--;
return -1;
}
else
{
f2<<"Sistem incompatibil!";
io=m+2;
return 0;
}
}
else
{
return 1;
}
}
void recuperare_solutie()
{
int j;
char c[n];
if(io==m+1)
{
if(m==n)
f2<<"Sistemul este compatibil determinat."<<endl;
else
{
f2<<"Sistemul este compatibil nedeterminat."<<endl;
int k=96;
for (j=1;j<=n;j++)
if(o[j]==0)
c[j]=char(++k);
}
f2<<"Solutia sistemului este:"<<endl;
for(jo=1;jo<=n;jo++)
{
f2<<"x"<<noshowpos<<jo<<"="<<showpos;
if(o[jo]==0)
{
f2<<c[jo]<<endl;
}
else
{
io=o[jo];
f2<<setw(8)<<fixed<<setprecision(4)<<a[io][n+1];
for(j=1;j<=n;j++)
if((o[j]==0)&&(fabs(a[io][j]))>eps)
{
f2<<setw(8)<<fixed<<setprecision(4)<<-a[io][j]<<"*"<<c[j];
}
f2<<endl;
}
}
}
}
void scrie_tabel()
{
int i,j;
f2<<"======";
for(j=1;j<=n+1;j++)
62 L. Deaconu

f2<<"========";
f2<<endl;
f2<<" |";
for(j=1;j<=n;j++)
f2<<setw(5)<<o[j]<<" ";
f2<<"|"<<endl;
f2<<"------";
for(j=1;j<=n+1;j++)
f2<<"--------";
f2<<endl;
for(i=1;i<=m;i++)
{
f2<<setw(4)<<v[i]<<"|";
for(j=1;j<=n;j++)
f2<<setw(8)<<fixed<<setprecision(3)<<a[i][j];
f2<<"|"<<setw(8)<<fixed<<setprecision(3)<<a[i][n+1]<<endl;
}
f2<<"======";
for(j=1;j<=n+1;j++)
f2<<"========";
f2<<endl;
}
int main(void)
{
int p;
citire();
if (m==0||n==0)
{
cout<<"Sistemul nu exista!";
return 1;
}
f2<<"Tabelul initial:"<<endl;
generare_tabel();
scrie_tabel();
io=1;
do
{
p=pivot();
if(p==1)
{
prelucrare();
f2<<"Pozitia pivotului: linia "<<io<<" coloana "<<jo<<endl;
scrie_tabel();
io++;
}
else
{
if(p==-1)
{
f2<<"Eliminare ecuatie - linia "<<io<<endl;
scrie_tabel();
}
}
}
while(io<=m);
recuperare_solutie();
f2.close();
return 0;
}

Pentru sistemul (2) se obt, ine rezultatul din fis, ierul solutie.txt:

Tabelul initial:
Rezolvarea sistemelor de ecuat, ii liniare 63

======================================================
| 0 0 0 0 0 |
------------------------------------------------------
0| 1.000 2.000 3.000 4.000 5.000| 6.000
0| 2.000 1.000 3.000 2.000 1.000| 8.000
0| 1.000 5.000 6.000 10.000 14.000| 10.000
0| 2.000 1.000 6.000 5.000 4.000| 3.000
======================================================
Pozitia pivotului: linia 1 coloana 5
======================================================
| 0 0 0 0 1 |
------------------------------------------------------
5| 0.200 0.400 0.600 0.800 1.000| 1.200
0| 1.800 0.600 2.400 1.200 0.000| 6.800
0| -1.800 -0.600 -2.400 -1.200 0.000| -6.800
0| 1.200 -0.600 3.600 1.800 0.000| -1.800
======================================================
Pozitia pivotului: linia 2 coloana 3
======================================================
| 0 0 2 0 1 |
------------------------------------------------------
5| -0.250 0.250 0.000 0.500 1.000| -0.500
3| 0.750 0.250 1.000 0.500 0.000| 2.833
0| 0.000 0.000 0.000 0.000 0.000| 0.000
0| -1.500 -1.500 0.000 0.000 0.000| -12.000
======================================================
Eliminare ecuatie - linia 3
======================================================
| 0 0 2 0 1 |
------------------------------------------------------
5| -0.250 0.250 0.000 0.500 1.000| -0.500
3| 0.750 0.250 1.000 0.500 0.000| 2.833
0| -1.500 -1.500 0.000 0.000 0.000| -12.000
======================================================
Pozitia pivotului: linia 3 coloana 1
======================================================
| 3 0 2 0 1 |
------------------------------------------------------
5| 0.000 0.500 -0.000 0.500 1.000| 1.500
3| 0.000 -0.500 1.000 0.500 0.000| -3.167
1| 1.000 1.000 -0.000 -0.000 -0.000| 8.000
======================================================
Sistemul este compatibil nedeterminat.
Solutia sistemului este:
x1= +8.0000 -1.0000*a
x2=a
x3= -3.1667 +0.5000*a -0.5000*b
x4=b
x5= +1.5000 -0.5000*a -0.5000*b

As, a cum se poate observa din fis, ierul cu rezultate, solut, ia generală obt, inută prin program
pentru sistemul (2) este




 x1 = 8 −a

x = a


 2


x3 =− 19
6
+ 12 a− 12 b , (4)




 x4 = b

x5 = 32 − 21 a− 12 b


64 L. Deaconu

În această formă ea diferă de solut, ia generală (3) obt, inută ı̂n exemplul de mai sus. Cele două
solut, ii generale generează aceeas, i mult, ime de solut, ii.
Dacă identificăm necunoscutele x4 s, i x5 din cele două solut, ii
3 1 1
x4 = α = b s, i x5 = β = − a − b,
2 2 2
adică, dacă folosim substitut, ia
3 1 1
α = b s, i β = − a− b
2 2 2
ı̂n solut, ia (3), regăsim solut, ia (4):




 x1 =5 + b + 3 − a − b = 8 − a

 x2 =3 − b − 3 + a + b = a




x =− 55 − b − 32 + 12 a + 21 b = − 19
 3 6
+ 12 a − 12 b .




 x4 =b

x5 = 32 − 21 a − 12 b


Determinarea pozit, iilor relative a mai multor drepte ı̂n
plan cu ajutorul mediului Maple

1
Raluca Mihaela Georgescu

Stabilirea pozit, iilor relative a dreptelor determinate de perechi de puncte din plan este o
problemă frecvent ı̂ntâlnită ı̂n geometria analitică plană.
Rezolvarea acestei probleme necesită destul de mult timp, deoarece presupune aplicarea
metodelor de determinare a pozit, iilor relative a două drepte pentru fiecare pereche de drepte.
Pentru n puncte distincte din plan avem N = Cn2 perechi de puncte care identifică acelas, i număr
de drepte. Stabilirea pozit, iilor relative presupune astfel analizarea unui număr destul de mare,
CN2 , de perechi de drepte.
Mediul de programare Maple oferă posibilitatea realizării unui program care, pentru fiecare
pereche ı̂n parte, determină pozit, ia relativă a dreptelor componente s, i, pentru dreptele concurente,
precizează măsura unghiului dintre ele.
Consideram punctele Pi (xi , yi ), cu i ∈ 1, n. Pentru a defini aceste puncte folosim comanda
point(Pi, xi, yi), (1)
pentru fiecare punct Pi cu i ∈ 1, n.
Folosind instruct, iunea for s, i obiectul geometric line construim toate dreptele dij ce trec
prin punctele Pi s, i Pj , pentru orice i ∈ 1, n − 1 s, i j ∈ 2, n, j > i:

for i to n-1 do
for j from i+1 to n do
line(d||i||j, [P||i, P||j]) (2)
end do
end do;

Pentru a verifica dacă trei puncte sunt coliniare, sau dacă două drepte sunt paralele sau
perpendiculare, se foloses, te una din funct, iile:

• AreCollinear, pentru a verifica dacă trei puncte sunt coliniare


• AreParallel, pentru a verifica dacă două drepte sunt paralele
• ArePerpendicular, pentru a verifica dacă două drepte sunt perpendiculare

Aceste trei funct, ii au ca rezultat ”adevărat” sau ”fals”.


Măsura unghiului dintre două drepte neparalele se determină cu ajutorul funct, iei FindAngle.
Dacă unghiul dintre cele două drepte nu este nul sau drept, atunci rezultatul va fi de forma
”arctan(x)”. În această situat, ie, pentru a converti ı̂n grade valoarea obt, inută se folosesc
comenzile:
1
Lect. univ. dr., Universitatea din Pites, ti, gemiral@yahoo.com

65
66 R.M. Georgescu

Round := proc (x, n) -> parse(sprintf("%.*f", n, x));


u := FindAngle(d||i||j, d||k||l);
a := 180*convert(u, float, 6)/(3.1415926535); (3)
b := Round(a, 4); c := trunc(b); d := trunc(60*(b-c));
e := Round(3600*(b-c-(1/60)*d), 0)

unde c reprezintă gradele, d minutele, iar e secundele corespunzătoare măsurii unghiului u dintre
dreptele dij s, i dkl .
Pentru determinarea s, i afis, area perechilor de drepte paralele folosim secvent, a

print(Dreptele paralele sunt):


for i to n-1 do
for j from i+1 to n do
for k from i to n-1 do
for l from k+1 to n do
if (i <> k or j < l) then
if AreParallel(d||i||j, d||k||l) then (4)
print(d||i||j si d||k||l)
end if
end if
end do
end do
end do
end do;

Pentru determinarea s, i afis, area perechilor de drepte confundate folosim secvent, a

print(Dreptele confundate sunt):


for i to n-1 do
for j from i+1 to n do
for k from i to n-1 do
for l from k+1 to n do
if (i <> k or j < l) then
if (AreCollinear(P||i, P||j, P||k) and
AreCollinear(P||i, P||j, P||l)) then (5)
print(d||i||j si d||k||l)
end if
end if
end do
end do
end do
end do

Pentru determinarea s, i afis, area perechilor de drepte perpendiculare folosim secvent, a

print(Dreptele perpendiculare sunt):


for i to n-1 do
for j from i+1 to n do
for k from i to n-1 do
for l from k+1 to n do
if (i <> k or j < l) then
if ArePerpendicular(d||i||j, d||k||l) then (6)
print(d||i||j si d||k||l)
end if
Determinarea pozit, iilor relative ... 67

end if
end do
end do
end do
end do

Pentru a determina s, i afis, a măsurile unghiurilor dintre perechile de drepte secante neperpen-
diculare folosim secvent, a

print(Masurile unghiurilor dintre dreptele secante neperpendiculare sunt):


for i to n-1 do
for j from i+1 to n do
for k from i to n-1 do
for l from k+1 to n do
if (i <> k or j < l) then
if ((((AreCollinear(P||i, P||j, P||k) and
AreCollinear(P||i, P||j, P||l)) or AreParallel(d||i||j, d||k||l)) or
ArePerpendicular(d||i||j, d||k||l)) = false then
Round := proc (x, n) -> parse(sprintf("%.*f", n, x));
u := FindAngle(d||i||j, d||k||l);
a := 180*convert(u, float, 6)/(3.1415926535); (7)
b := Round(a, 4); c := trunc(b); d := trunc(60*(b-c));
e := Round(3600*(b-c-(1/60)*d), 0);
printf("Masura unghiului dintre dreptele %3.3s si %3.3s este
%d grade %d minute %d secunde \n", d||i||j, d||k||l, c, d, e)
end if
end if
end do
end do
end do
end do

Observat, ie Pentru stabilirea si afis, area pozit, iilor relative a perechilor de drepte determinate
de cele n puncte, secvent, ele (4)-(7) pot fi combinate ı̂ntr-una singură, iar programul ı̂n Maple
este

for i to n-1 do
for j from i+1 to n do
for k from i to n-1 do
for l from k+1 to n do
if (i <> k or j <> l) then
if AreCollinear(P||i, P||j, P||k)
and AreCollinear(P||i, P||j, P||l) } then
print(Dreptele d||i||j si d||k||l sunt confundate)}
elif AreParallel(d||i||j, d||k||l) then
print(Dreptele d||i||j si d||k||l sunt paralele) (8)
elif ArePerpendicular(d||i||j, d||k||l) then
print(Dreptele d||i||j si d||k||l sunt perpendiculare)
else Round := proc (x, n) -> parse(sprintf("%.*f", n, x));
u := FindAngle(d||i||j, d||k||l);
a := 180*convert(u, float, 6)/(3.1415926535);
b := Round(a, 4); c := trunc(b); d := trunc(60*(b-c));
e := Round(3600*(b-c-(1/60)*d), 0);
print(Dreptele d||i||j si d||k||l sunt secante neperpendiculare)
printf("Masura unghiului dintre cele doua drepte este
68 R.M. Georgescu

%d grade %d minute %d secunde", c, d, e)


end if
end if
end do
end do
end do
end do

În continuare vom exemplifica not, iunile prezentate pentru n = 5 puncte din plan.

Exemplu. Se dau punctele P1 (1, 1), P2 (4, −2), P3 (5, 5), P4 (6, 4), P5 (0, 2).
a) Să se precizeze perechile de drepte paralele;
b) Să se precizeze perechile de drepte confundate;
c) Să se precizeze perechile de drepte perpendiculare;
d) Să se calculeze măsurile unghiurilor dintre perechile de drepte secante neperpendiculare.
2
Cele 5 puncte determină N = 10 drepte, deci trebuie analizate C10 = 45 perechi de drepte.
Mai ı̂ntâi definim punctele Pi , i ∈ 1, 5, folosind comanda (1) pentru fiecare punct ı̂n parte, adică

point(P1, 1, 1), point(P2, 4, -2), point(P3, 5, 5),


point(P4, 6, 4), point(P5, 0, 2)

Apoi construim dreptele dij , i, j ∈ 1, 5, i 6= j, folosindu-ne de (2), considerând n = 5.


Putem reprezenta ı̂n sistemul de axe ortogonale punctele (Fig. 1 a)) s, i dreptele (Fig. 1 b))
obt, inute, cu ajutorul comenzilor

draw({P1,P2,P3,P4,P5})
draw({d12,d13,d14,d15,d23,d24,d25,d34,d35,d45})

a) b)
Fig. 1

a) Pentru determinarea perechilor de drepte paralele folosim secvent, a (4) pentru n = 5 s, i


obt, inem rezultatul

Dreptele paralele sunt


d12 si d15
Determinarea pozit, iilor relative ... 69

d12 si d25
d12 si d34
d14 si d35
d15 si d25
d15 si d34
d25 si d34

b) Pentru determinarea perechilor de drepte confundate folosim secvent, a (5) pentru n = 5 s, i


obt, inem rezultatul

Dreptele confundate sunt


d12 si d15
d12 si d25
d15 si d25

c) Pentru determinarea perechilor de drepte perpendiculare folosim secvent, a (6) pentru


n = 5 s, i obt, inem rezultatul

Dreptele perpendiculare sunt


d12 si d13
d13 si d15
d13 si d25
d13 si d34

d) Pentru a determina măsurile unghiurilor dintre perechile de drepte secante neperpendicu-


lare folosim secvent, a (7) pentru n = 5 s, i obt, inem valorile pentru măsurile unghiurilor dintre
drepte.

Dacă unim cele patru cerint, e ı̂ntr-o singură problemă, folosindu-ne de (8) pentru n = 5,
obt, inem pozit, iile relative pentru fiecare din cele 45 de perechi. Folosind notat, iile
= pentru dreptele confundate,
⊥ pentru dreptele perpendiculare,
|| pentru dreptele paralele,
iar pentru dreptele neperpendiculare măsura unghiului dintre ele, putem sintetiza rezultatele
obt, inute ı̂n următorul tabel:

d13 d14 d15 d23 d24 d25 d34 d35 d45


d12 ⊥ 75 570 5000

= 53 7 4800
◦ 0
63 260 600

= || 75 570 5000

63 260 600

d13 14◦ 20 1100 ⊥ 36◦ 520 1200 26◦ 330 5400 ⊥ ⊥ 14◦ 20 1100 26◦ 330 5400
d14 75 570 5000

50◦ 540 2200 40◦ 360 500 75 570 5000

75◦ 570 5000 || 12◦ 310 4400
d15 53◦ 70 4800 63◦ 260 600 = || 75 570 5000

63◦ 260 600
d23 10◦ 180 1700 53◦ 70 4800 53◦ 70 4800 50◦ 540 2200 63◦ 260 600
d24 63◦ 3000 63◦ 260 600 40◦ 360 500 53◦ 70 4800
d25 || 75◦ 570 5000 63◦ 260 600
d34 75◦ 570 5000 63◦ 260 600
d35 12◦ 310 4400
RUBRICA DE ROBOTICĂ

Programarea robot, ilor LEGO folosind mediul


Mindstorms EV3

1
Doru Anastasiu Popescu

Robot, ii sunt folosit, i pentru a realiza anumite activităt, i. Pentru a crea s, i programa robot, i
s-au realizat kit-uri speciale s, i medii de programare. Un asemenea robot este cel obt, inut din
piese LEGO care pentru deplasare foloses, te mediul interactiv de programare Mindstorms EV3.
Kit-ul de bază numit ı̂n engleză LEGO Mindstorm Education EV3 Core Set cont, ine piesele
necesare pentru realizarea unui robot, care să se deplaseze pe o suprafat, ă plană (acesta cont, ine
două motoare pentru deplasare, senzori, unitate de comandă – numită cărămidă inteligentă,
rot, i s, i alte componente LEGO necesare construct, iei robotului).
În Figura 1 este prezentat un robot simplu construit cu aceste piese. Pachetul de bază
cont, ine descrierea necesară construirii unui robot, cu difert, i senzori, astfel ı̂ncât să poată fi
programat pentru deplasarea cu anumite restrict, ii. Un senzor poate fi de mai multe feluri:
de culoare – pentru determinarea unei culori, tactil – pentru a detecta aparit, ia unui obiect,
infraros, u – pentru detectarea obiectelor din apropierea robotului, etc. Robotul pe care ı̂l vom
folosi (Figura 1) pentru verificarea programelor va avea două motoare mari, câte unul pentru
fiecare roată laterală s, i o cărămidă inteligentă (intelligent brick) pentru a controla act, iunile sale.
Această cărămidă are rolul de a memora proiectele s, i de a lansa ı̂n execut, ie programele din ele.

Fig. 1: Elementele constructive ale unui robot LEGO Minstorms EV3

Pentru a crea programe ce deplasează robotul avem nevoie de instalarea mediului Mindstorms
Education EV3, care nu necesită costuri de licent, ă. Kit-ul de instalare se găses, te la adresa
1
Conf. univ. dr., Universitatea din Pites, ti, dopopan@yahoo.com

70
Programarea robot, ilor LEGO 71

https://education.lego.com/en-us/downloads. După lansarea ı̂n execut, ie a acestui mediu grafic


interactiv de programare se obt, ine o fereastră simplă ı̂n care apare un buton cu semnul plus. La
activarea acestui buton se va crea un proiect nou, ı̂n care se vor scrie programele necesare pentru
deplasarea robotului. Un proiect se salvează ı̂ntr-un fis, ier cu extensia ev3, iar un program
este o succesiune de blocuri ce ı̂ncepe cu blocul de Start s, i se termină cu blocul Stop Program.
Blocurile sunt grupate pe categorii s, i se află ı̂n partea de jos a ferestrei zonei de lucru (Figura
2). Un bloc este adus ı̂n program prin select, ie s, i glisat la locul lui cu mouse-ul. Fiecare bloc
trebuie configurat cu parametrii necesari.

Fig. 2: Interfat, a mediului de programare Minstorms EV3

Primul program!
Programul din Figura 3 deplasează robotul cu trei rotat, ii complete ale unei rot, i (aproximativ 55
cm pentru robotul din imagine). Acesta este alcătuit din patru blocuri: blocul de start, blocul
Move Stering – configurat cu On for Rotations, direct, ia ı̂nainte, viteza 50 s, i 3 rotat, ii pentru
fiecare roată, blocul Move Stering – configurat cu Off pentru oprirea motoarelor, respectiv
blocul de oprire Stop Program.

Fig. 3: Deplasarea robotului in linie dreaptă cu 3 rotat, ii

După scrierea programului se conectează robotul la calculator s, i se ı̂ncarcă proiectul cu programul


ı̂n cărămida robotului, apoi se poate lansa ı̂n execut, ie (Figura 4).
72 D.A. Popescu

Fig. 4: Lansarea ı̂n execut, ie a unui program

Din acest exemplu se observă că programul este ca o schemă logică, cu specificarea faptului că
blocurile sunt specializate pe o anumită operat, ie s, i conectate ı̂ntre ele.
Pentru programele pe care le vom scrie avem nevoie de două blocuri motor din Grupul Action.

Fig. 5: Blocurile Move Steering s, i Move Tank

Blocurile Move Steering s, i Move Tank sunt folosite pentru cele două motoare (ı̂n Figura 5
motoarele sunt conectate la porturile B s, i C). Primul buton din fiecare bloc permite setarea
unităt, ii de măsurare a deplasării: secunde, grade, număr de rotat, ii (On for Rotations - pe care
o vom utiliza ı̂n continuare). Pentru oprirea motoarelor se foloses, te Off, iar pentru funct, ionarea
fără oprire On. La blocul Move Steering celelalte butoane sunt pentru configurarea direct, iei,
vitezei s, i numărului de rotat, ii la ambele motoare. La blocul Move Tank apar două butoane
pentru configurarea separată a vitezei la fiecare motor, celelalte butoane au aceeas, i semnificat, ie.
Blocul Move Tank este folosit atunci când vrem să schimbăm direct, ia de deplasare (de exemplu
o ı̂ntoarcere).

Fig. 6: Blocul Sound

În grupul de blocuri Action mai apar blocurile Sound pentru sunet (Figura 6) s, i Display pentru
afis, area datelor pe ecranul cărămizii (Figura 7). Blocul Sound permite alegerea fis, ierului ce
cont, ine sunetul dorit. Mindstorm Education EV3 cont, ine o bibliotecă cu fis, iere sunet, acestea
fiind grupate pe tipuri de sunete (mecanice, animale, culori, numere, etc.).

Fig. 7: Blocul Display


Programarea robot, ilor LEGO 73

Blocul Display permite afis, area textelor, figurilor geometrice, imaginilor din fis, iere, cu s, tergerea
sau nu a ecranului. Afis, area se va realiza pornind din pozit, ia specificată prin coordonatele x s, i y.
Sistemul de coordonate are originea ı̂n colt, ul din stânga - sus al ecranului, axa Ox pe orizontală
cu sensul pozitiv spre dreapta, iar axa Oy pe verticală cu sensul pozitiv ı̂n jos.
Problema deplasării cu sunete
Dorim să deplasăm robotul cu două rotat, ii s, i după fiecare rotat, ie completă a rot, ilor acesta să
redea lătratul unui câine s, i apoi să pronunt, e cuvântul Mindstorms. Creat, i un proiect s, i salvat, i-l
cu numele p2.ev3.
Solut, ie
Pentru realizarea programului se creează un proiect nou, apoi ı̂n fereastra programului se
aduc din zona blocurilor Action: două blocuri Move Steering, două blocuri Sound s, i un bloc
Stop Program. Configurarea s, i ordinea acestor blocuri este cea din Figura 8.

Fig. 8: Program pentru deplasarea robotului cu redarea unor sunete

Problema deplasării cu afis, are pe ecran


Deplasat, i robotul pe o linie dus-ı̂ntors, redat, i repetat cuvântul Hello, iar pe ecranul cărămizii
scriet, i cuvintele Dus-Intors. Distant, a parcursă pe linie va fi de aproximativ 55 cm. Creat, i un
proiect s, i salvat, i-l cu numele p3.ev3.
Solut, ie
Blocul Start se pune ı̂n legătură cu un bloc Display, care se configurează cu Text – Pixels,
s, tergere ecran, cordonatele x=10, y=20 (x este pe orizontală – spre dreapta, y este pe verticală –
ı̂n jos) s, i textul de afis, at Dus-Intors. Următorul bloc care trebuie introdus este Sound configurat
cu sunetul MINDSTORMS din fis, ierul aflat ı̂n grupul Communication, pentru redare repetată.
Urmează apoi trei blocuri Move pentru deplasarea ı̂nainte (Move Steering), rotire (Move Tank )
s, i iar ı̂nainte (Move Steering). Înainte de blocul Stop Program se reface ecranul cărămizii
folosind blocul Display cu opt, iunea Reset Screen. În Figura 9 este programul descris anterior.

Fig. 9: Program pentru deplasarea robotului cu sunete s, i afis, ări pe ecran

Observat, ii
În mediul grafic interactiv Mindstorms Education EV3 proiectele pot cont, ine unul sau mai
multe programe, care pot fi lansate ı̂n execut, ie pe rând. Proiectele ce cont, in programele sunt
scrise pe calculator s, i apoi copiate pe cărămidă.
74 D.A. Popescu

Programe care folosesc senzori s, i variabile

În continuare vom prezenta modul de folosire a senzorilor s, i a variabilelor pentru a realiza
programe cât mai diverse.

Fig. 10: Senzori pentru robot, i LEGO Minstorms EV3

Problema detectării culorii unui obiect

În multe situat, ii este nevoie să cunoas, tem ce culoare are un obiect. Spre exemplu, dacă
robotul se deplasează pe o suprafat, ă cu diverse culori este util să s, tim la fiecare moment pe ce
culoare se află, pentru a corecta deplasarea.

Programul din Figura 11 determină afis, area pe ecranul cărămizii a valorii asociate culorii pe
care se află senzorul de culoare (Colour Sensor ) al robotului.

Fig. 11: Program cu senzor de culoare

Problema detectării distant, ei până la un obiect

Senzorul care ne permite să determinăm distant, a de la robot până la un obiect este cel
infraros, u (Infrared Sensor ). Pentru măsurarea distant, ei foloses, te percent-ul notat cu pct, 100
pct = 70 cm. Programul din Figura 12 afis, ează pe ecranul cărămizii distant, a de la senzor la un
cub.
Programarea robot, ilor LEGO 75

Fig. 12: Program cu senzor de distant, ă

Problema detectării obiectelor prin atingere


Senzorul tactil (Touch Sensor ) ne permite detectarea obiectelor atinse de vârful acestuia. În
funct, ie de modul de atingere acesta transmite una din valorile 0, 1, 2. Programul din Figura 13
afis, ează una dintre aceste valori.

Fig. 13: Program cu senzor tactil

Grupul Sensor cont, ine blocuri pentru mai mult, i senzori, trei dintre ele fiind folosite mai
frecvent (aces, tia sunt ı̂n kit-ul de bază):

1. Touch Sensor (senzor tactil – folosit la măsurarea gradului de atingere a obiectelor: 1, 2,


3)
2. Infrared Sensor (senzor infraros, u – folosind la identificarea obiectelor de la o distant, ă
precizată, unitatea de măsură este pct, 100pct=70cm)
3. Colour Sensor (senzor de culoare – folosit la identificarea culorilor, de cele mai multe
ori pe suprafat, a pe care se deplasează robotul, culorile au asociate numere 0 - culoare
nedefinită, 1 - negru, 2 – albastru, 3 – verde, 4 – galben, 5 – ros, u, 6 – alb, 7 - maro).

La tot, i senzorii, ı̂n căsut, a din dreapta-sus trebuie introdus numărul portului unde este
conectat la cărămidă. În cazul ı̂n care cărămida este conectată la calculator, atunci numărul
portului este completat automat.
76 D.A. Popescu

Problema numărului aleator


Folosind generatorul de numere aleatoare vrem să obt, inem un număr de două cifre s, i să
afis, ăm pe ecranul cărămizii, la un interval de 10 secunde, numărul urmat de pătratul său. În
programul următor sunt prezentate blocurile utilizate.

Fig. 14: Program cu variabile init, ializate cu valori generate aleator

Programul cont, ine câteva blocuri noi: Random – pentru generarea unui număr natural
cuprins ı̂ntre două valori date, Variable – pentru a realiza operat, ia de creare, atribuire s, i folosire
a valorii unei variabile, Math – pentru operat, ii aritmetice, din grupul Data Operations.

Grupul Data Operations cont, ine blocuri pentru lucrul cu variabile, operatori s, i expresii.

Fig. 15: Crearea unei variabile

Blocurile cele mai utilizate din grupul Data Operations sunt:

1. Variable – cu două stări: Read, pentru folosirea valorii s, i Write, pentru modificarea valorii
(Figura 15).
2. Constant – pentru a crea date constante, ce pot fi folosite ı̂n program numai cu valoarea
dată la definirea lor.
3. Random – permite generarea unui număr aleator dintr-un interval de valori specificat sau
a valorilor true sau false (de tip logic).
4. Logic Operations – pentru operat, iile logice (s, i, sau, negat, ie). Valorile pentru care se
realizează operat, ia se vor introduce ı̂n bloc.
5. Math – pentru operat, ii aritmetice (adunare, scădere, ı̂nmult, ire, ı̂mpărt, ire, etc.).
6. Compare – pentru operat, iile de comparare (folosind operatorii relat, ionali). Rezultatul
acestui bloc este o valoare logică.

Atât pentru variabile, cât s, i pentru constante, se utilizează trei tipuri de date: numerice,
text s, i logic.
Programarea robot, ilor LEGO 77

Problema deplasării pe un traseu ı̂n formă de dreptunghi


Generat, i aleator numărul de rotat, ii pe care să le parcurgă un robot pe laturile unui drept-
unghi (adică lungimile laturilor măsurate ı̂n rotat, ii de motor), pentru lungime din mult, imea
{4,5,6}, iar pentru lăt, ime din mult, imea {2,3,4}, apoi deplasat, i robotul pornind dintr-un colt, s, i
la final revenit, i ı̂n aceeas, i pozit, ie. Toate operat, iile specificate se vor realiza cu acelas, i program.

Solut, ie
Pentru rezolvarea problemei vom folosi două variabile numerice, cu numele a – pentru
lungime, b - pentru lăt, ime. Crearea lor se va realiza cu două blocuri Variable, iar pentru
init, ializarea lor se vor utiliza numerele generate de blocul Random folosit de două ori, cu limitele
de valori 4 s, i 6 pentru a, respectiv 2 s, i 4 pentru b. Apoi sunt folosite blocurile Move Stering s, i
Variable, cu opt, iunea Read, pentru numărul de rotat, ii, ca să se deplaseze robotul. În Figura 16
este prezentat programul.

Fig. 16: Program pentru deplasarea robotului pe un contur dreptunghiular

Probleme propuse
1. Se consideră programul următor:

Fig. 17: Program pentru deplasarea unui robot LEGO Minstorms EV3

Cum se deplasează robotul, dacă se foloses, te acest program? Explicat, i care este rolul fiecărui
bloc.
2. Se consideră programul din Figura 18. Care este efectul acestui program, dacă senzorul
de culoare se află deasupra unei plans, e de culoare albă?
78 D.A. Popescu

Fig. 18: Program cu senzor de culoare

Pentru fiecare din problemele următoare scriet, i câte un program folosind mediul interactiv
de programare Mindstorm Education EV3.
3. Generat, i aleator trei numere naturale din mult, imea {1, 2, ..., 10} s, i apoi afis, at, i pe ecranul
cărămizii, pentru 5 secunde, suma lor.
4. Modificat, i programul de la Problema deplasării pe un traseu ı̂n formă de drept-
unghi astfel ı̂ncât deplasarea să se realizeze pe conturul unui pătrat.

Bibliografie

[1] D.A. Popescu, S. Profeanu, S. Dobrescu, Manual de informatică pentru clasa a V-a, Editura
CD-Press, 2017.

[2] Minstorms EV3 – Ghid de Utilizare, LEGO Group, 2013.

[3] L. Negrescu, L. Negrescu, Construirea s, i programarea robot, ilor LEGO Minstorms EV3,
Editura Albastră, 2015.

[4] J. Olayvar, E. Lindberg, LEGO Mindstorms EV3 Programming Basics, Washington State
Library, 2016.
PROBLEME DE MATEMATICĂ PENTRU EXAMENE 79

PROBLEME DE MATEMATICĂ PENTRU


EXAMENE

Teste pentru examenul de Evaluare Nat, ională

Testul 1

1 2
Ana Maria Iordache s, i Mădălina Enache

SUBIECTUL I

1. Rezultatul calculului 2−1 · 0, (3) − 1 : 6 este . . .


2. Dacă 2x
7
10
= 4y , atunci 10% din xy este . . .
3. Un unghi al unui triunghi are măsura egală cu 33◦ 12”. Suma celorlalte unghiuri ale
triunghiului este egală cu . . .
4. Cardinalul mult, imii Z ∩ [−2, 3) este . . .
5. Cubul PROBLEMA are muchia de 6 cm. Aria patrulaterului POML este . . . cm2 .
6. Un elev are evident, a mesajelor primite pe telefon de la prietenii săi ı̂ntr-o săptămână,
redată ı̂n tabelul de mai jos:
Ziua Luni Mart, i Miercuri Joi Vineri Sâmbătă Duminică
Numărul mesajelor 17 12 18 6 14 20 18
primite
Numărul mediu de apeluri primite pe zi este . . .

SUBIECTUL al II-lea

1. Desenat, i o piramidă triunghiulară regulată SABC.


2. a) Demonstrat, i că numărul A = (x2 + x)(x2 + x − 20) + 100 este pătrat perfect, oricare
ar fi x ∈ N.
b) Determinat, i o valoare naturală
√ a lui x pentru care A este cub perfect.
3. Calculat, i opusul numărului 2 3 − 1 − 1−4√3 .

| |
4. Ana are o sumă de bani. După ce cheltuies, te o zecime din sumă, restul ı̂l ı̂mparte ı̂n mod
egal cu fratele ei, Dan. În vacant, ă, Ana ı̂s, i triplează suma, iar Dan s, i-o ı̂njumătăt, es, te pe
a sa, ajungând√să aibă ı̂mpreună 315 lei. Cât, i bani a avut Ana init, ial?
5. Dacă x > 0 s, i x − √1x = 2, calculat, i x + x1 .

SUBIECTUL al III-lea

1. Figura următoare reprezintă curtea unei pensiuni formată din parcul ABCD s, i rondurile cu
flori ı̂n formă de semicerc. Ştim că AB = 20 m, DC = 10 m s, i AD = BC = 16 m, ABkCD.

1
Profesor, Liceul tehnologic ,,Virgil Madgearu”, Ros, iorii de Vede, anamariayord@yahoo.com
2
Profesor, Liceul tehnologic ,,Virgil Madgearu”, Ros, iorii de Vede, emadimate@yahoo.com
80 PROBLEME DE MATEMATICĂ PENTRU EXAMENE

D C

A B
Figura 1
a) Aflat, i suprafat, a rondurilor cu flori.
b) Determinat, i perimetrul curt, ii.
c) Aflat, i cu cât la sută este mai mare aria triunghiului AOB fat, ă de aria triunghiului
DOC, unde {O} = AC ∩ BD.
2. O cutie sub forma unui paralelipiped dreptunghic ABCDA0 B 0 C 0 D0 are dimensiunile
proport, ionale cu primele trei numere naturale prime s, i suma tuturor muchiilor 120 cm.
a) Aflat, i cât, i cm2 de carton au fost necesari pentru confect, ionarea cutiei.
b) Determinat, i o funct, ie trigonometrică a unghiului format de dreptele D0 B s, i AD.
c) Este posibilă introducerea ı̂n cutie a unei rigle având lungimea de 20 cm?

Testul 2

3
Florin Stănescu

SUBIECTUL I
1. Rezultatul calculului 5 − 22 · (42 − 15) este . . .
2. Dacă 4n = 64, atunci n este egal cu . . .
3. La aruncarea unui zar probabilitatea de a obt, ine un număr prim este egală cu . . .
4. Diagonala unui pătrat cu aria de 16 cm2 este egală cu . . . cm
5. Se consideră cubul ABCDM N P Q. Măsura unghiului format de dreptele AB s, i DQ este
egală cu . . ..
6. În tabelul de mai jos este reprezentată situat, ia notelor obt, inute de elevii unei clase la un
test de evaluare:

Nota 3 4 5 6 7 8 9 10
Număr elevi 2 5 7 4 5 4 2 1

Media clasei este egală cu . . .


SUBIECTUL al II-lea
1. Desenat, i pe foaia de examen piramida patrulateră regulată V ABCD, cu vârful V s, i baza
ABCD.
2. Fie a, b, c trei numere astfel ı̂ncât 2a + b + 2c = 11 s, i a + 2b + c = 4. Calculat, i (a + c)2 − b2 .
3. Un casier, fiind ı̂ntrebat cât a ı̂ncasat ı̂ntr-o zi, a răspuns: dacă as, fi ı̂ncasat ı̂ncă 25%
din cât am ı̂ncasat s, i ı̂ncă 500 lei, as, fi obt, inut 5500 lei. Cât a ı̂ncasat ı̂n ziua respectivă
casierul?
3
Profesor, S, coala Gimnazială ,,S, erban Cioculescu”, Găes, ti, florin.florinstanescu@yahoo.com
PROBLEME DE MATEMATICĂ PENTRU EXAMENE 81

4. Se consideră funct, ia f : R → R, f (x) = ax + 2, unde a ∈ R.


a) Determinat, i a, astfel ı̂ncât punctul A(1, 3) apart, ine graficului funct, iei f .
b) Pentru a = 1, determinat, i sinusul unghiului format de axa Ox cu graficul funct, iei f .
5. Se consideră expresia

x−2
Ç å
3 1 4
E(x) = − − 2 : , x ∈ R \ {−1, 1, 2}.
2(x − 1) 2(x + 1) x − 1 x2 − 2x + 1

Determinat, i a ∈ Z \ {−1, 1, 2} pentru care E(a) ∈ Z.

SUBIECTUL al III-lea

1. În Figura 1 este ilustrat schematic un teren de fotbal. Cele două careuri sunt reprezen-
tate prin două dreptunghiuri EF IJ, respectiv GHKL, care au lungimea dublul lăt, imii,
suprafat, a din mijlocul terenului este un cerc de centrul O s, i diametrul de 15 m, iar ABN M
este dreptunghi. Avem: P Q = EF = GH = AE = F B = DH = GC = N P = M Q s, i
AM = M D = 50 m.
a) Stabilit, i dacă suprafat, a terenului de fotbal, fără cele două careuri s, i cercul de la
mijlocul terenului, este mai mare decât 3390 m2 . (3, 14 < π < 3, 15)
b) Doi jucători, plecând ı̂n acelas, i timp, aleargă ı̂n linie dreaptă cu aceeas, i viteză pe
următoarele trasee: primul jucător: D − N − A, al doilea jucător: F − N − M − H.
Care dintre cei doi jucători termină primul alergarea?
c) Aflat, i sinusul unghiului EOF .

B N C

I P L G
F
O
K
E H
J Q

A M D
Figura 1

2. În Figura 2, este reprezentată o cutie sub forma unei prisme drepte ABCDA0 B 0 C 0 D0 cu
baza pătrat. Prisma are aria bazei egală cu 100 dm2 , iar suma ariilor fet, elor laterale ale
prismei este de 200 dm2 .
a) Arătat, i că ı̂nălt, imea cutiei este BB 0 = 5 dm.
b) Aflat, i distant, a de la punctul A0 la dreapta AC 0 .
c) Aflat, i sinusul unghiului format de diagonalele prismei AC 0 s, i A0 C.
82 PROBLEME DE MATEMATICĂ PENTRU EXAMENE

A B
C
D

A’ B’

D’ C’
Figura 2

Testul 3
4
Adina Florina Militaru

SUBIECTUL I
1. Rezultatul calculului 20180 + 02018 + 2018 : 2 este . . .
2. Dacă ab = 32 , atunci 2a este egal cu . . .
3b √
3. Lungimea razei cercului circumscris unui pătrat este egală cu 4 2 cm. Aria pătratului
este . . . cm2 .
4. Două unghiuri complementare congruente au măsurile egale cu . . .
5. Aria laterală a unui con circular drept cu raza de 3 cm s, i ı̂nălt, imea de 4 cm este egală cu
. . . π cm2 .
6. În tabelul de mai jos sunt reprezentate notele obt, inute de elevii unei clase la teza de la
matematică:

Note sub 5 5 − 5,99 6 − 6,99 7 − 7,99 8 − 8,99 9 − 9,99 10


Număr elevi 1 3 4 3 5 7 2

Numărul elevilor care au obt, inut cel put, in nota 7 este egal cu . . .
SUBIECTUL al II-lea
1. Desenat, i pe foaia de examen un paralelipiped dreptunghic
√ 2 P ROM√ QV AT .
2. Calculat, i media aritmetică a numerelor a = (3 + 2) s, i b = (3 − 2)2 .
3. Alina ı̂s, i propune să citească o carte ı̂n trei zile. În prima zi vrea să citească jumătate
din ea, a doua zi un sfert s, i ı̂ncă 20 de pagini, iar a treia zi cu 5 pagini mai put, in decât
jumătate din a doua zi. Câte pagini are cartea?
4. Se consideră funct, ia f : R → R, f (x) = 2x + 3.
a) Reprezentat, i grafic funct, ia ı̂n sistemul de coordonate xOy.
b) Determinat, i aria triunghiului format de graficul funct, iei s, i axele de coordonate.
5. Se consideră expresia
x−3 x−1 x−1
ñ ô
1
E(x) = + − · , x ∈ R \ {−1, 0, 1, 2}.
x (x − 1)(x + 1) x(x + 1) x − 2

Arătat, i că E(x) = x1 .


4
Profesor, Liceul Tehnologic Construct, ii de Mas, ini, Mioveni, popescuadina73@yahoo.com
PROBLEME DE MATEMATICĂ PENTRU EXAMENE 83

SUBIECTUL al III-lea
1. Fie ABCD un trapez dreptunghic ı̂n A, având bazele AB = 14 cm s, i CD = 10 cm, iar
m(^B) = 30◦ .
a) Determinat, i lungimea liniei mijlocii a trapezului.
b) Determinat, i aria trapezului.
c) Calculat, i distant, a de la vârful C la diagonala BD.
2. Se consideră un paralelipiped dreptunghic ABCDA0 B 0 C 0 D0 cu dimensiunile de 6 cm, 4
cm, respectiv 3 cm.
a) Calculat, i lungimea diagonalei paralelipipedului.
b) Ştiind că pentru acoperirea unui cm2 se folosesc 10 g vopsea, calculat, i câte kg de
vopsea sunt necesare pentru acoperirea ı̂ntregului corp.
c) Cât, i litri de apă ı̂ncap ı̂n paralelipiped?

Testul 4

5
Mugurel Simion

SUBIECTUL I
1. Rezultatul calculului 18 · 6 − 18 : 6 este egal cu . . . .
2. Numerele ı̂ntregi din intervalul (−2, 6] sunt ı̂n număr de . . . .
3. Şase robinete de acelas, i debit pot umple un bazin ı̂n 8 ore. Două robinete vor umple
acelas, i bazin ı̂n . . . ore.
4. Aria unui pătrat este 81 cm2 . Perimetrul pătratului este egal cu . . . cm.
5. Lungimea ı̂nălt, imii unui con circular drept cu raza bazei egală cu 8 cm s, i generatoarea de
16 cm este egală cu . . . .
6. Numărul elevilor dintr-un lot de handbal s, i vârstele lor sunt reprezentate ı̂n tabelul de
mai jos:

Vârsta (ani) 11 12 13
Numărul elevilor 5 10 5

Media de vârstă a lotului de elevi este egală cu . . . ani.


SUBIECTUL al II-lea
1. Desenat, i un cilindru circular drept cu sect, iunea axială dreptunghiul ABB 0 A0 .
2. Aflat, i cel mai mic număr natural de trei cifre care prin ı̂mpărt, ire la 14 s, i 21 dă de fiecare
dată restul 3.
2
3. Un turist a parcurs un traseu ı̂n trei zile. În prima zi a parcurs din lungimea traseului.
7
A doua zi a parcurs 20% din rest, iar a treia zi a parcurs ultimii 24 de km. Aflat, i lungimea
traseului.
4. Se consideră funct, ia f : R → R, f (x) = −2x + 6.
√ √
a) Arătat, i că: f ( 3) − f ( 3 − 5) = −10.
b) Determinat, i punctele M (x, y) situate pe graficul funct, iei f cu proprietatea x = y .
5
Profesor, S, coala Gimnazială ,,Constantin Brâncoveanu”, S, tefănes, ti, simion 34@yahoo.com
84 PROBLEME DE MATEMATICĂ PENTRU EXAMENE

5. Se consideră expresia:
E(x) = (x + 1)(x2 + 2x − 24).
Arătat, i că 6|E(x), pentru oricare x număr ı̂ntreg.
SUBIECTUL al III-lea
1. Figura următoare reprezintă schit, a unui teren. Triunghiul ABC este isoscel cu AB =
AC = 20 cm√s, i m(^BAC) = 120◦ . Triunghiul BDC este dreptunghic cu m(^BDC) = 90◦
s, i DC = 10 3 cm.

Q
B C

D

a) Arătat, i că BC = 20 3 cm.

b) Arătat, i că A4 ACD = 100 3 cm2 . √
c) Dacă Q ∈ (BC) astfel ı̂ncât BQ = 12 3 cm, verificat, i dacă punctele A, Q s, i D sunt
coliniare.
2. Fie ABCDA0 B 0 C 0 D0 o prismă patrulateră regulată cu AB = 6 cm, AA0 = 8 cm, BD ∩
AC = {P } s, i BC 0 ∩ B 0 C = {Q}.
a) Arătat, i că volumul prismei este egal cu 288 cm3 . √
0 0 82
b) Arătat, i că sinusul unghiului format de dreptele P Q s, i B D este egal cu .
10
c) Aflat, i distant, a de la punctul A0 la planul (AB 0 D0 ).

Testul 5

6
Adrian Ţurcanu

SUBIECTUL I
1. Inversul numărului rat, ional 0,25 este numărul natural . . .
2. Aria unui romb cu diagonalele de 12 cm s, i 7 cm este . . . cm2 .
3. Solut, ia negativă a ecuat, ie (x + 5)2 = 121 este . . .
4. Cel mai mare număr natural din intervalul [−3, 8) este . . .
5. Fie ABCDA0 B 0 C 0 D0 un cub. Măsura unghiului dintre dreptele B 0 C s, i AD0 este . . .
6. Notele obt, inute de elevii clasei a VIII-a A la teza la matematică din semestrul I sunt date
ı̂n tabelul de mai jos:

Nota 4 5 6 7 8 9 10
Număr elevi 1 2 3 3 5 5 11
6
Lect. univ. dr., Universitatea din Pites, ti, adrianturcanu85@yahoo.com
PROBLEME DE MATEMATICĂ PENTRU EXAMENE 85

Conform tabelului, probabilitatea ca alegând un elev din Clasa a VIII-a A, acesta să fi
obt, inut la teză o notă mai mare sau egală cu 8 este . . .
SUBIECTUL al II-lea
1. Desenat, i prisma patrulateră regulată
√ M√ AT EIN F O.√ √
2. Arătat, i că numărul N = (3 − 2)2 + 2( 2 + 5)(3 − 2) + ( 2 + 5)2 este natural.
3. Un turist a parcurs un traseu ı̂n trei zile astfel: ı̂n prima zi o cincime din drum, a doua zi
jumătate din rest, iar a treia zi ultimii 10 km. Aflat, i lungimea traseului.
4. Fie funct, ia f : R → R, f (x) = 2x − 6.
a) Reprezentat, i grafic funct, ia ı̂n sistemul de coordonate xOy.
b) Determinat, i numărul natural nenul n care verifică egalitatea f (1) + f (2)+. . . +f (n) =
150.
10
5. Se consideră mult, imile A = {x ∈ Z | |2x − 1| ≤ 5} s, i B = {x ∈ Z | x+1 ∈ Z}. Determinat, i
A ∩ B.
SUBIECTUL al III-lea
1. O grădină are forma unui trapez isoscel ABCD cu AB = 100 m, CD = 40 m s, i
BC = AD = 50 m.
a) Calculat, i lungimea gardului care ı̂mprejmuies, te grădina.
b) Calculat, i aria suprafet, ei grădinii.
c) Dacă AD ∩ BC = {M }, determinat, i aria triunghiului M DC.
2. Fie V ABC o piramidă triunghiulară regulată cu latura bazei BC = 12 cm, M mijlocul
lui [BC], V M = 6 cm s, i V O ı̂nălt, imea acesteia.
a) Aflat, i aria laterală s, i volumul piramidei.
b) Dacă N este mijlocul laturii [CV ] arătat, i că M N k (V OB).
c) Arătat, i că V A ⊥ V M .
86 PROBLEME DE MATEMATICĂ PENTRU EXAMENE

Teste pentru examenul de Bacalaureat, specializarea S, tiint, e ale naturii

Testul 1
1
Marius Macarie

SUBIECTUL I
5 − 4i
1. Determinat, i partea imaginară a numărului complex z = , unde i2 = −1.
2 + 3i
2. Să se determine m ∈ R astfel ı̂ncât valoarea minimă a funct, iei f : R → R, f (x) =
1
x2 − 3x + m − 2 să fie − .
4
5
3. Rezolvat, i ı̂n R ecuat, ia: log3 x + logx 3 = .
2
4. Să se calculeze probabilitatea ca alegând o mult, ime din mult, imea submult, imilor nevide
ale mult, imii A = {1, 2, 3, 4, 5}, aceasta să aibă trei elemente.
5. Să se determine ecuat, ia mediatoarei segmentului AB, unde A(2, −1), B(−4, 1).
6. Triunghiul ABC are lungimile laturilor AB = 2, BC = 3 s, i AC = 4. Să se calculeze cos A.
SUBIECTUL al II-lea
Ñ é
1 + 3x 9x
1. Se consideră matricea A(X) = , unde x este număr real.
x 1 + 3x
a) Calculat, i det A(−1).
b) Demonstrat, i că A(x) · A(y) = A(x + y + 6xy), pentru orice x, y ∈ R.
c) Calculat, i A−1 (1). √
2. Pe G = (1, ∞) se defines, te legea de compozit, ie x ◦ y = x2 y 2 − x2 − y 2 + 2.
»
a) Demonstrat, i că x ◦ y = (x2 − 1)(y 2 − 1) + 1.
b) Demonstrat, i că legea ,,◦” este asociativă.
| ◦ x ◦{z. . . ◦ x} = x.
c) Rezolvat, i ı̂n G ecuat, ia x
de 2018 ori x

SUBIECTUL al III-lea
1. Fie f : R → R, f (x) = 2x3 · e−x .
a) Determinat, i ecuat, ia asimptotei la Gf spre +∞.
b) Determinat, i punctele
Ç 00
de extrem
å
ale funct, iei f .
f (x)
c) Calculat, i lim x +1 .
x→∞ f 0 (x)
x−1
2. Fie f : (0, ∞) → R, f (x) = √ .
x

a) Să se determine a, b ∈ R astfel ı̂ncât funct, ia g : (0, ∞) → R, g(x) = (ax + b) · x să
fie o primitivă a lui f .
Z 4
b) Calculat, i f (x)g 2 (x)dx, unde a s, i b au valorile determinate la punctul a).
1
c) Determinat, i volumul corpului obt, inut prin rotat, ia ı̂n jurul axei Ox a graficului funct, iei
1
h : [2, 3] → R, h(x) = .
f (x)
1
Lect. univ. dr., Universitatea din Pites, ti, macariem@yahoo.com
PROBLEME DE MATEMATICĂ PENTRU EXAMENE 87

Testul 2
2
Raluca Mihaela Georgescu

SUBIECTUL I
1. Să se determine x ∈ R astfel ı̂ncât 2, x2 + 3x s, i 8 sunt termeni consecutivi ai unei progresii
geometrice.
2. Să se determine inversa funct, iei bijective f : R √ → (4, ∞), f (x) = 2x + 4.
3. Rezolvat, i ı̂n mult, imea numerelor reale ecuat, ia 5x − 6 − x = 0.
4. Câte numere naturale de 4 cifre distincte se pot forma cu elemente din A = {0, 1, 2, 3, 4, 5, 6}.
−→ −→ −−→
5. Fie punctele A(2, 3), B(−1, 1) s, i C(0, −2). Aflat, i lungimea vectorului AB + AC + BC.
Å
π ã
5
6. Fie x ∈ 0, s, i sin x = . Să se calculeze tg 2x.
2 13
SUBIECTUL al II-lea
Ü ê
1 a 2
1. Se consideră matricea A(a) = 1 1 2a , unde a este număr real.
1 2 a
a) Calculat, i det A(0).
b) Să se afle a ∈ R astfel ı̂ncât A(a) să fie inversabilă.
Ü ê
−2
c) Să se determine X ∈ M3,1 (R) astfel ı̂ncât A(3) · X = 0 .
−1
4x + 4y
2. Pe mult, imea G = (−2, 2) se defines, te legea de compozit, ie x ◦ y = . Fie f : G → R,
4 + xy
2+x
f (x) = log5 .
2−x
a) Demonstrat, i că legea ,,◦” este asociativă.
b) Demonstrat, i că f (x ◦ y) = f (x) + f (y), ∀x, y ∈ G.
c) Să se determine simetricul lui 1 ∈ G fat, ă de legea ,,◦”.
SUBIECTUL al III-lea
1 + ln x
1. Fie f : (0, ∞) → R, f (x) = .
x
a) Determinat, i ecuat, ia tangentei la graficul funct, iei f ı̂n punctul de abscisă x = e, situat
pe graficul funct, iei f .
b) Demonstrat, i că ln x ≤ x − 1, oricare ar fi x ∈ (0, ∞).
c) Determinat, i intervalele de convexitate s, i concavitate ale funct, iei f .
2. Fie f : R → R, f (x) = (x + 2) · e−x .
Z 1
a) Arătat, i că ex · f (x)dx = 4.
Z 1−1
b) Calculat, i f 00 (x)dx.
0
c) Să se determine numărul real a > 1 astfel ı̂ncât aria suprafet, ei plane delimitate de
graficul funct, iei g : (0, ∞) → R, g(x) = f (ln x), axa Ox s, i dreptele de ecuat, ii x = 1
5
s, i x = a să fie egală cu .
2
2
Lect. univ. dr., Universitatea din Pites, ti, gemiral@yahoo.com
88 PROBLEME DE MATEMATICĂ PENTRU EXAMENE

Testul 3

3
Mihai Florea Dumitrescu

SUBIECTUL I

1. Aflat, i suma elementelor mult, imii A = {x ∈ [−1; 1] | [x] · x = 1}, unde [x] reprezintǎ partea
ı̂ntreagǎ a numǎrului real x.
2. Se considerã funct, iile f : R → R, f (x) = x2 − 2x + 1 s, i g : R → R, g(x) = −x2 + 4x − 4.
Aflat, i distant, a dintre vârfurile parabolelor asociate celor douǎ funct, ii.
3. Rezolvat, i ı̂n mult, imea numerelor reale inecuat, ia 9log3 x ≤ 2 · 3log3 x − 1 .
4. Câte numere pare de trei cifre se pot forma cu cifrele 0, 1, 2, 3 ?
5. În reperul cartezian xOy se considerã punctele A(3, −5) s, i B(−3, 1). Determinat, i lungimea
−→ −−→
vectorului AO + OB. Ä ä
6. Se considerǎ expresia E (x) = sin2 2x + cos2 3x + tg 2 4x + ctg 2 6x, x ∈ R. Calculat, i E 12π
.

SUBIECTUL al II-lea
Ü ê
x  
1. Se considerǎ matricele A (x, y) = 1 , B (x, y) = 1 y x , unde x s, i y sunt
y
numere reale.
a) Arǎtat, i cǎ det [A (x, y) · B (x, y)] = 0 pentru orice numere reale x s, i y.
b) Rezolvat, i ı̂n mult, imea numerelor reale ecuat, ia det [A (x, x) · B (x, x) + xI3 ] = 4x.
c) Rezolvat, i ı̂n Z × Z ecuat, ia det [B (x, y) · A (x, y)] = 1.
2. Pe mult, imea numerelor reale se defines, te legea de compozit, ie asociativǎ x∗y = xy−x−y+2.
a) Arǎtat, i cǎ x ∗ y = (x − 1) (y − 1) + 1 pentru orice numere reale x s, i y.
b) Aflat, i numǎrul n ∈ N care verificǎ egalitatea Cn1 ∗ Cn2 = 1.
c) Calculat, i 2 ∗ 3 ∗ 4 ∗ ... ∗ 2018.

SUBIECTUL al III-lea

1. Se considerã funct, ia f : R → R, f (x) = x + cos x.


a) Calculat, i lim f (x).
x→+∞
b) Determinat, i ecuat, ia tangentei la graficul funct, iei f ı̂n punctul de abscisǎ x = 0, situat
pe graficul funct, iei f .
c) Arãtat, i cã 2 cos 1 ≤ π − 2. √
2. Se considerã funct, ia f : R → R, f (x) = x2 + 1.
Z 1√
4
a) Arãtat, i cã x2 + 1 · f (x) dx = .
Z 20 3
b) Calculat, i f (x) dx.
1
c) Aflat, i volumul corpului obt, inut prin rotat, ia ı̂n jurul axei Ox a graficului funct, iei
x
g : [1, 2] → R, g (x) = f (x) .

3
Profesor, Liceul ,,S, tefan Diaconescu”, Potcoava, florin14mihai@yahoo.com
PROBLEME DE MATEMATICĂ PENTRU EXAMENE 89

Testul 4

4
Gheorghe Nistor

SUBIECTUL I

1. Fie (an )n≥1 o progresie aritmetică. S, tiind că a3 = 10 s, i a5 = 16, să se calculeze a100 .
2. Rezolvat, i ı̂n R ecuat, ia: (f ◦ f )(x) = 7, unde f : R → R, f (x) = −4x + 3.
3. Rezolvat, i ı̂n R inecuat, ia: 2 log2 x + 1 ≤ 5.
4. Aflat, i probabilitatea ca alegând o funct, ie f : {1, 2, 3} → {7, 8, 9}, aceasta să fie bijectivă.
5. Să se determine ecuat, ia dreptei care trece prin A(0, 3) s, i e perpendiculară pe dreapta
determinată de punctele B(0, 2) s, i C(−1, 4).
6. Să se afle raza cercului circumscris triunghiului ABC cu lungimile laturilor AB = 7,
BC = 8 s, i AC = 9.

SUBIECTUL al II-lea
 Ü ê
mx + y + 2z = 4



m 1 2
1. Se consideră sistemul de ecuat, ii  2x + my + z = 3 s, i matricea A(m) = 2 m 1 ,


x + 2y + mz = 1 1 2 m
unde m este un număr real.
a) Calculat, i det A(0).
b) Să se afle m ∈ R pentru care matricea A(m) nu este inversabilă.
c) Rezolvat, i sistemul pentru m = 1.
2. Se consideră polinomul f = X 4 − X 3 + mX 2 + 2X + n unde m, n ∈ R.
a) Să se determine m, n ∈ R astfel ı̂ncât f să fie divizibil cu g = X 2 − 3X + 2.
b) Să se determine m, n ∈ R astfel ı̂ncât x1 = 1 + i să fie rădăcină pentru f .
c) Pentru m = n = 0 să se descompună polinomul f ı̂n produs de factori ireductibili ı̂n
R[X].

SUBIECTUL al III-lea
x2 + x + 1
1. Fie f : R \ {1} → R, f (x) = .
x−1
f (x) − f (3)
a) Calculat, i lim .
x→3 x−3
b) Determinat, i ecuat, ia asimptotei oblice spre +∞ la graficul funct, iei f .
c) Determinat, i intervalele de monotonie ale funct, iei f .
2. Fie f : (0, ∞) → R, f (x) = (x + 1) ln x.
3
a) Să se determine o primitivă F a lui f pentru care F (1) = .
Z 3 4
ln x
b) Calculat, i dx.
2 xf (x)
c) Determinat, i volumul corpului obt, inut prin rotat, ia ı̂n jurul axei Ox a graficului funct, iei
f (x)
g : [1, e] → R, g(x) = .
x+1
4
Lect. univ. dr., Universitatea din Pites, ti, ghe.nistor@yahoo.com
90 PROBLEME DE MATEMATICĂ PENTRU EXAMENE

Testul 5
5
Alina S, tefan

SUBIECTUL I
Ä √ ä 8
1. Arătat, i că numărul n = log3 9 3 9 + 8log2 3 − este număr natural.
3
2. Aflat, i distant, a dintre punctele de intersect, ie ale graficului funct, iei f : R → R, f (x) =
x2 + 4x + 3 cu axa absciselor. Ç åx+1 Ç åx
4 27 2
3. Rezolvat, i ı̂n R inecuat, ia: · = .
9 8 3
4. Calculat, i probabilitatea ca, alegând un număr din mult, imea numerelor naturale de 3 cifre,
acesta să fie cub perfect.

− →
− →
− →

5. Fie vectorii → −
u = 2 i − 3 j s, i →
−v = m i − (m − 1) j . Să se determine m ∈ R astfel ı̂ncât

− u ·→−
v = 5. π π
6. Calculat, i lungimea razei cercului circumscris triunghiului M N P , s, tiind că M = , N =
6 3
s, i M N = 4.
SUBIECTUL al II-lea
! !
3 6 1 0
1. Se consideră matricele A = , I2 = s, i X(a) = I2 + a · A, unde a ∈ R.
−1 −2 0 1
a) Calculat, i det(A2 ).
b) Verificat, i dacă X(a) · X(b) = X(a + b + ab) pentru orice a, b ∈ R.
c) Determinat, i X 2018 (1).
2. Se consideră polinomul f = X 3 − 3X 2 + aX − 2, a ∈ R.
a) Să se determine a ∈ R astfel ı̂ncât
Ç restulå ı̂mpărt, irii lui f la X − 2 să fie 4.
9
b) Să se demonstreze că dacă a ∈ , ∞ , atunci f nu are toate rădăcinile reale.
2
x x x
1 2 3

c) Să se determine a ∈ R astfel ı̂ncât x3 x1 x2 = 9, unde x1 , x2 , x3 sunt rădăcinile

x2 x3 x1
polinomului f .
SUBIECTUL al III-lea
1. Fie f : (0, ∞) → R, f (x) = x3 · ln x.
f (x) − f (e)
a) Calculat, i lim .
x→e x−e
b) Arătat, i că graficul funct, iei f nu admite asimptote.
1
c) Demonstrat, i că f (x) ≥ − , pentru orice x ∈ (0, ∞).
3e
? xn
2. Pentru fiecare n ∈ N se consideră funct, ia fn : R → R, fn (x) = .
x2 + 4
Z 2 Ä√ ä 6
a) Arătat, i că x dx = 1 − 4 ln .
f2
1 Z 1 Z 15
1
b) Demonstrat, i că fn+2 (x)dx + 4 fn (x)dx = .
0 0 n+1
c) Să se determine a > 1 astfel ı̂ncât aria suprafet, ei plane delimitate de graficul funct, iei
f1 , axa Ox s, i dreptele de ecuat, ii x = 1 s, i x = a să fie egală cu ln 2.
5
Lect. univ. dr., Universitatea din Pites, ti, alinastanescu2000@yahoo.com
PROBLEME DE MATEMATICĂ PENTRU EXAMENE 91

Teste pentru examenul de Bacalaureat, specializarea


Matematică-Informatică

Testul 1
1
Daniel Jinga

SUBIECTUL I (30p)
1. Determinat, i numerele reale a, b s, tiind că (a + bi)(b + ai) = (a − bi)(b − ai), unde i2 = −1.
(5p)
2. Determinat, i m ∈ R pentru care funct, ia f : R → R, f (x) = x2 − (m − 3)x + m − 4 are
valoarea minimă egală cu 0. (5p)
3. Rezolvat, i ecuat, ia 6x − 2x − 2 · 3x + 2 = 0. (5p)
n 2
4. Aflat, i n ∈ Z cu proprietatea Cn+2 = 1. (5p)
−→ −→
5. În triunghiul ABC, AB = 3, BC = 6, AC = 4. Calculat, i AB · AC. (5p)
1
6. Fie x ∈ R astfel ı̂ncât tg x = . Calculat, i sin 2x. (5p)
2
SUBIECTUL al II-lea (30p)




x + my − z = 1
1. Se consideră sistemul de ecuat, ii liniare  x + y − z = 2 , unde m ∈ R.


−x + y + z = 0
a) Arătat, i că matricea sistemului are rangul 2 pentru orice m ∈ R. (5p)
b) Determinat, i m ∈ R astfel ı̂ncât sistemul să fie compatibil. (5p)
c) Aflat, i solut, iile (x, y, z) ∈ Z × Z × Z cu proprietatea că x2 + y 2 + z 2 = 42. (5p)
x+y
2. Fie G = (−1, 1) s, i x ∗ y = , ∀ x, y ∈ G.
1 − xy
a) Arătat, i că tg 27◦ ∗ tg 18◦ = 1. (5p)
b) Arătat, i că (x ∗ y) ∗ z = x ∗ (y ∗ z), ∀ x, y, z ∈ G. (5p)
c) Rezolvat, i ı̂n G ecuat, ia x ∗ x ∗ x = 2x ∗ 2x ∗ 2x. (5p)
SUBIECTUL al III-lea (30p)
3x2 + 3x + 1
1. Se consideră funct, ia f : R \ {−1, 0} → R, f (x) = .
x3 (x + 1)3
1 1
a) Arătat, i că f (x) = 3
− , ∀ x ∈ R \ {−1, 0}. (5p)
x (x + 1)3
b) Determinat, i coordonatele punctului situat pe graficul funcţiei f , ı̂n care tangenta la
graficul funct, iei f este perpendiculară pe axa ordonatelor. (5p)
1
c) Calculat, i lim (f (1) + f (2) + . . . + f (n)) nf (n) . (5p)
n→∞

1
Profesor, Colegiul Nat, ional ,,Ion C. Brătianu”, Pites, ti, jinga.daniel@yahoo.com
92 PROBLEME DE MATEMATICĂ PENTRU EXAMENE

Z 2
2. Se consideră funct, ia f : R → R, f (x) = 2x − x2 s, i In = f n (x)dx, unde n ∈ N∗ .
0

a) Arătat, i că I1 = 34 . (5p)


b) Arătat, i că s, irul (In )n≥1 este convergent. (5p)
c) Arătat, i că (2n + 1)In = 2nIn−1 , ∀ n ≥ 2. (5p)

Testul 2
2
Sorin Ulmeanu

SUBIECTUL I (30p)
1. Să se rezolve ı̂n mult, imea numerelor reale ecuat, ia |x − 1| = 2x − 5. (5p)
2. Fie ecuat, ia x2 − 2x + m = 0, m ∈ R, care are soluţiile reale x1 s, i x2 . Ştiind că |x1 − x2 | = 1,
să se determine m. (5p)
Å
πã Å
π ã
3. Să se rezolve ecuat, ia sin x + + cos − x = 1, x ∈ R. (5p)
6 3
1−x
4. Să se arate că funct, ia f : (−1, 1) → R, f (x) = ln , ∀ x ∈ (−1, 1) este impară. (5p)
1+x
5. Fie G centrul de greutate al triunghiului ABC. Să se determine a, b ∈ R astfel ı̂ncât să
−→ −−→ −→
aibă loc egalitatea aGA + bGB = GC. (5p)
6. Să se determine ecuat, ia medianei din A a triunghiului ABC, s, tiind că A(2, 2) s, i că
ecuat, iile medianelor duse din B s, i C sunt 2x + y − 2 = 0, respectiv x − y + 2 = 0. (5p)
SUBIECTUL al II-lea (30p)

ax + y + z = 1



1. Se consideră sistemul (S):  x + ay + z = 0 , a ∈ R.


x + y + az = −1
a) Arătat, i că determinantul matricei sistemului este egal cu (a + 2)(a − 1)2 . (5p)
b) Determinat, i a ∈ R astfel ı̂ncât (S) să nu fie compatibil. (5p)
c) Aflat, i a ∈ R astfel ı̂ncât solut, ia unică (x0 , y0 , z0 ) a sistemului (S) să formeze o progresie
2
aritmetică de rat, ie r = − . (5p)
5
2. Se consideră mult, imea de numere complexe G = {cos(qπ) + i sin(qπ) | q ∈ Q}.

1 3
a) Să se arate că z = 0 nu se află ı̂n G s, i că ε = + i se află ı̂n G. (5p)
2 2
b) Să se arate că ı̂nmult, irea numerelor complexe determină pe G o structură algebrică de
grup comutativ. (5p)
c) Să se arate că polinomul f = X 6 − 1 are toate rădăcinile ı̂n G. (5p)
2
Profesor, Colegiul Nat, ional ,,Ion C. Brătianu”, Pites, ti, sorinulm@yahoo.com
PROBLEME DE MATEMATICĂ PENTRU EXAMENE 93

SUBIECTUL al III-lea (30p)


1. Se consderă funct, ia f : R → R, dată prin f (x) = x − sin x, pentru orice x ∈ R.
a) Să se arate că funct, ia f este strict crescătoare. (5p)
b) Să se arate că graficul funct, iei f nu are asimptote. (5p)
»
c) Să se arate că g : R → R, g(x) = 3
f (x), oricare ar fi x ∈ R, este derivabilă pe R. (5p)
Z 1
2. Fie s, irul (In )n∈N∗ , dat prin In = (1 − x2 )n dx, ∀ n ∈ N∗ .
−1

a) Să se calculeze I2 . (5p)


2n + 2
b) Să se demonstreze că In+1 = In , ∀ n ∈ N∗ . (5p)
2n + 3
n
X (−1)k Cnk
c) Să se demonstreze că s, irul (an )n∈N∗ , definit prin an = , ∀ n ∈ N∗ , are limita 0.
k=0 2k + 1
(5p)

Testul 3
3
Ileana Ţârcă

SUBIECTUL I (30p)
1. Calculat, i partea reală a numărului complex (1 − 5i)2 . (5p)
» √
2. Rezolvat, i ecuat, ia 3 15 − x − 4 = 2. (5p)
3. Se consideră mult, imea A = {1, 2, 3, 4, 5, 6}. Determinat, i probabilitatea ca alegând la
ı̂ntâmplare una dintre submult, imile lui A, aceasta să cont, ină exact 3 elemente. (5p)
4. Fie fm , g : R → R, fm (x) = x2 + mx + m s, i g(x) = x + 2. Determinat, i m ∈ R pentru
care Gfm ∩ Gg are un singur element. (5p)
5. Fie dreptele d1 : mx + 2y + 1 = 0, d2 : x + (m + 1)y + 5 = 0. Să se determine m ∈ R astfel
ı̂ncât d1 k d2 . (5p)
6. Să se calculeze sin2 1480◦ + cos2 2200◦ . (5p)
SUBIECTUL al II-lea (30p)
Ü ê Ü ê Ü ê
1 0 0 0 1 0 0 0 0
1. Se consideră matricele A = 0 1 0 ,B= 0 0 0 s, i O3 = 0 0 0 .
0 0 0 1 0 0 0 0 0
a) Să se calculeze A2 . (5p)
b) Să se calculeze rangul matricei A + B. (5p)
c) Să se calculeze m·n, unde m este cel mai mic număr natural nenul pentru care (A·B)m = O3
s, i n este cel mai mic număr natural nenul pentru care (B · A)n = O3 . (5p)
2. Se consideră polinomul f = X 4 − 3X + 1, cu rădăcinile x1 , x2 , x3 , x4 ∈ C.
3
Profesor, Liceul de Arte ,,Dinu Lipatti”, Pites, ti, ileana tirca@yahoo.ro
94 PROBLEME DE MATEMATICĂ PENTRU EXAMENE

a) Să se calculeze x1 · x2 · x3 · x4 . (5p)


b) Să se calculeze x41 + x42 + x43 + x44 . (5p)
c) Să se arate că polinomul f nu are rădăcini raţionale. (5p)
SUBIECTUL al III-lea (30p)
ln(2 − x)
1. Se consideră funct, ia f : (−2, 2) → R, f (x) = .
x+2
a) Să se calculeze f 0 (x), x ∈ (−2, 2). (5p)
b) Verificat, i dacă funct, ia f este descrescătoare pe intervalul (−2, 2). (5p)
c) Determinat, i punctele de inflexiune ale funct, iei f . (5p)
1
2. Se consideră funct, ia f : R → R, f (x) = √ .
x2 + 9
Z 1
a) Calculat, i f (x)dx. (5p)
0
Z 1
b) Calculat, i xf (x)dx. (5p)
0

c) Determinat, i volumul corpului obt, inut prin rotat, ia ı̂n jurul axei Ox a graficului funct, iei
g : [0, 1] → R, g(x) = f (x). (5p)

Testul 4
4
Mihai Florea Dumitrescu

SUBIECTUL I (30p)
®ñ ô ´
1
1. Aflat, i suma elementelor mult, imii A = | x ∈ Z∗ , unde [a] reprezintă partea ı̂ntreagǎ
x
a numǎrului real a. (5p)
2. Se consideră funct, ia f : R → R, f (x) = x2 − ax + 1. Aflat, i a ∈ R pentru care distant, a de
la axa de simetrie a parabolei asociate funct, iei f la axa Oy este egalǎ cu 2. (5p)
3. Rezolvat, i ı̂n mult, imea numerelor reale ecuaţia |x − 1|3 = (1 − x)2 . (5p)
4. Calculat, i probabilitatea ca, alegând la ı̂ntâmplare un număr din mult, imea numerelor
naturale de douǎ cifre, cu cifre nenule s, i diferite, acesta să aibă cifra zecilor pătrat perfect s, i
cifra unităt, ilor cub perfect. (5p)
5. În reperul cartezian xOy se consideră punctele A(1, −5) s, i B(−3, 3). Determinat, i
simetricul punctului O fat, ă de dreapta AB. (5p)
π ã Å Å
πã Å
πã 1
6. Se consideră expresia, E(x) = sin − x + cos x − + tg x − + unde
2 √ 2 2 tg x
Å
πã 1 5
x ∈ 0, . Ştiind că sin 2x = , arătat, i că E(x) = . (5p)
2 4 2
4
Profesor, Liceul ,,S, tefan Diaconescu”, Potcoava, florin14mihai@yahoo.com
PROBLEME DE MATEMATICĂ PENTRU EXAMENE 95

SUBIECTUL al II-lea (30p)


Ü ê
x −1 1
1. Se consideră matricea A(x) = −1 x −1 , unde x este număr real.
1 −1 x
a) Calculat, i detA(0). (5p)
b) Rezolvat, i ı̂n mult, imea numerelor reale ecuat, ia detA(log2 x) = 0. (5p)
c) Arătat, i că A(a) · A(b) = A(ab + 2) + (a + b) · A(0), pentru orice numere reale a s, i b. (5p)
2. Se consideră polinomul f = X 3 + aX 2 + aX + b, unde a, b ∈ R s, i fie x1 , x2 , x3 rădăcinile
polinomului.
a) Aflat, i a s, i b s, tiind că polinomul X 2 + 1 divide polinomul f . (5p)
b) Pentru b = 1, există a ∈ Z pentru care are loc egalitatea (5p)
Ç å
1 1 1 1 1 1 1 1 1
3
+ 3 + 3 =2 2 + 2 + 2 + + + ?
x1 x2 x 3 x1 x2 x3 x1 x2 x3

c) Arătat, i că, pentru b = 1 s, i a ∈ (−∞, −1] ∪ [3, ∞) polinomul f are toate rădăcinile reale.
(5p)
SUBIECTUL al III-lea (30p)
1. Se consideră funct, ia f : R → R, f (x) = sin x − x.
a) Arătat, i că funct, ia f este inversabilă. (5p)

b) Arătat, i că 2(2 − 3)(3 sin 1 + π) > 3. (5p)
x + y ã f (x) + f (y)
Å
c) Arătat, i că f ≥ , pentru orice x, y ∈ [0, π]. (5p)
2 2
xn
Z 1
2. Pentru fiecare număr natural nenul n, se consideră integrala In = dx.
0 x4 + 1
a) Calculat, i I3 . (5p)
8
b) Calculat, i Ik . (5p)
P
k=1

c) Calculat, i lim nIn . (5p)


n→∞

Testul 5
5
Antonio Nuică

SUBIECTUL I (30p)

1 − 3i
1. Să se determine

2+i
.

(5p)

2. Să se rezolve ı̂n R ecuat, ia 4x − 3 · 2x + 2 = 0. (5p)


5
Lect. univ. dr., Universitatea din Pites, ti, antonio 74nm@yahoo.com
96 PROBLEME DE MATEMATICĂ PENTRU EXAMENE

3. Să se determine m ∈ R pentru care mx2 − mx + 1 > 0, ∀ x ∈ R. (5p)


4. Să se determine numărul funct, iilor injective f : {1, 2, 3, 4, 5} → {1, 2, 3, 4, 5} cu proprieta-
tea f (1) = 1. (5p)
5. Să se rezolve ı̂n R ecuat, ia sin 2x = sin x. (5p)
6. Să se calculeze raza cercului circumscris triunghiului ABC, pentru care AB = 3, BC = 5,
CA = 4. (5p)
SUBIECTUL al II-lea (30p)
1. Fie G = (a, ∞), a > 0 s, i x ◦ y = xy − ax − ay + a2 + a, x, y ∈ G.
a) Să se arate că G este parte stabilă relativ la legea de compozit, ie ◦. (5p)
b) Să se arate că (G, ◦) este grup comutativ. (5p)
c) Să se determine inversul elementului a + 1 ı̂n grupul G. (5p)




x−y+z =1
2. Se consideră sistemul (S):  x + y + z = 3 , m ∈ R.


mx + y + z = 3m
a) Să se determine valorile lui m pentru care sistemul are solut, ie unică. (5p)
b) Să se rezolve sistemul pentru m = 2. (5p)
c) Să se rezolve sistemul pentru m = 1. (5p)
SUBIECTUL al III-lea (30p)
x2 − 1
1. Se consideră funct, ia f : R → R, f (x) = .
x2 + 1
a) Să se determine asimptotele la graficul lui f . (5p)
b) Să se studieze monotonia lui f . (5p)
c) Să se determine ecuat, ia tangentei la graficul lui f ı̂n punctul (0, −1). (5p)
xn
Z 1
2. Fie In = dx, n ≥ 1.
0 7x + 8
a) Să se calculeze I1 . (5p)
1
b) Să se arate că 7In+1 + 8In = , pentru orice n ≥ 1. (5p)
n+1
c) Să se calculeze lim In .(5p)
n→∞
PROBLEME DE MATEMATICĂ PENTRU EXAMENE 97

Teste pentru admiterea la facultate

Testul 1

1
Eduard Asadurian

SUBIECTUL I
1. Fie mult, imea
 Ü ê 



x 0 x 


M = A(x) = 0 2 0 | x ∈ R∗  ⊂ M3 (R).
 

x 0 x 

a) Arătat, i că (M, ∗), unde A(x) ∗ A(y) = 12 A(x)A(y), este un grup comutativ, izomorf
cu grupul multiplicativ R∗ .
b) Rezolvat, i ecuat, ia

A(x) ∗ A(x) ∗ · · · ∗ A(x) = A(x) + A(x) + · · · + A(x) ı̂n R∗ × N.


| {z } | {z }
de n ori de n ori

2. Să se rezolve ı̂n C ecuat, ia x4 − x3 + 2x2 + x + 3 = 0, s, tiind că are două rădăcini a căror
sumă este egală cu 2.

SUBIECTUL al II-lea
x2n +2
1. Se consideră s, irul (xn )n≥1 dat prin x1 = 4, xn+1 = x2n −1
, ∀n ≥ 1. Să se studieze convergent, a
s, irului (xn ).
2. Calculat, i
Z2 nx
e +2
lim dx.
n→∞ enx − 1
1

SUBIECTUL al III-lea
Fie M s, i N două puncte pe laturile BC, respectiv AC ale triunghiului ascut, itunghic
−−→ −−→ −−→ −−→
ABC astfel ı̂ncât M C = k1 M B s, i N C = k2 N A. Fie {I} = AM BN s, i {P } = CI AB.
T T

Presupunem cunoscute lungimile laturilor triunghiului ABC.


IC
1. Aflat, i raportul ;
IP
IC
2. Deducet, i valorile raportului ı̂n cazurile ı̂n care AM s, i BN sunt:
IP
a) mediane; b) bisectoare; c) ı̂nălt, imi.

1
Lect. univ. dr., Universitatea din Pites, ti, ediasadurian@yahoo.com
98 PROBLEME DE MATEMATICĂ PENTRU EXAMENE

Testul 2
2
Raluca Mihaela Georgescu

SUBIECTUL I
¶ √ ©
Fie mult, imea A = a + b 2| a, b ∈ Z .
1. Să se arate că (A, +) este grup abelian.
2. Să se arate că polinomul f ∈ R[X], f (X) = X 3 − 3X 2 + X + 1 are o rădăcină x0 ∈ A, cu
x0 < 3.
3. Să se arate că toate rădăcinile ecuat, iei x4 − 6x2 + 1 = 0 se află ı̂n mult, imea A.
SUBIECTUL al II-lea
xn
Fie funct, iile fn : R → R, fn (x) = , n ∈ N? .
x2 + 2x + 5
1. Să se determine ecuat, iile asimptotelor la graficul funct, iei f3 .
2. Să se determine numărul punctelor de extrem ale funct, iei g = f3 − f2 .
R1
lim nIn , unde In = fn (x)dx.
3. Să se calculeze n→∞
0

SUBIECTUL al III-lea
În planul de coordonate xOy se consideră punctele A(1, 0), B(3, 1) s, i C(1, 3).
1. Dacă M este mijlocul segmentului [BC], să se determine coordonatele unui punct D
din plan, D 6= A, astfel ı̂ncât ariile triunghiurilor ABC s, i BCD să fie egale s, i punctele
A, M, D să fie coliniare.
2. Să se determine coordonatele centrului cercului circumscris triunghiului ABC.
3. Să se găsească două puncte E s, i F ı̂n cadranul IV astfel ı̂ncât AEF B să fie pătrat.

Testul 3
3
D.M.I.

Algebră
√ √
2
1. Să se rezolve ecuat, ia 2 x−2 + 5 x −3x+2 = 2.
2. Să se calculeze Cn1 − 2Cn2 + 3Cn3 + . . . + (−1)n−1 nCnn .
3. Fie En o matrice pătratică de ordinul n cu toate elementele egale cu 1. Să se arate că:
a) En2 = nEn ;
b) In − En este inversabilă s, i să se arate că inversa ei este egală cu
1
In − En .
n−1
4. Pe mult, imea G = (0, +∞) − {1} se defines, te legea ,,∗” astfel x ∗ y = xln y . Să se arate că
,,∗” este o lege de compozit, ie pe G s, i să se demonstreze că (G, ∗) este grup comutativ.
5. a) Combinări (definit, ie).
b) Arătat, i că numărul submult, imilor unei mult, imi cu n elemente este 2n .
k−1
c) Arătat, i că: Cnk = Cnn−k ; Cnk = Cn−1
k
+ Cn−1 .
2
Lect. univ. dr., Universitatea din Pites, ti, gemiral@yahoo.com
3
Universitatea din Pites, ti, revista.matinf@upit.ro
PROBLEME DE MATEMATICĂ PENTRU EXAMENE 99

Analiză Matematică
12 + 22 + 32 + . . . + n2
1. Să se calculeze limita s, irului an = .
 n3
 ex + x − α , x ≤ 1
2. Fie funct, ia f : R → R, f (x) =  1 , α ∈ R. Să se studieze continuita-
x x−1 , x>1
tea funct, iei.
3. Fie funct, ia f : R → R, f (x) = A cos ωx + B sin ωx, unde ω ∈ R. Să se demonstreze
egalitatea
f 00 (x) + ω 2 f (x) = 0, ∀ x ∈ R.

ex + a , x≤0
√ √


4. Se dă funct, ia f : R → R, f (x) =  4 + x − b , a, b ∈ R, b > 0.
 , x>0
x
a) Să se determine a s, i b astfel ı̂ncât f să fie continuă.
b) Să se calculeze primitivele funct, iei f pentru a s, i b determinat, i la punctul anterior.
R π/4
5. Să se calculeze 0 sin22tg x+3
x+2 cos2 x
dx.

Geometrie
1. Fie ABC un triunghi s, i D, E, F picioarele ı̂năt, imilor din A, B, respectiv C. Să se arate
că:
a) tiunghiurile AEF , DEC, DBF sunt asemenea cu triunghiul ABC;
b) bisectoarele triunghiului DEF coincid cu ı̂nălt, imile triunghiului ABC.
2. Se dă un cerc s, i o coardă AB. Prin mijlocul C al arcului AB se duc două coarde oarecare
ce intersectează cercul ı̂n D s, i E, iar coarda AB respectiv ı̂n F s, i G. Demonstrat, i că
patrulaterul DEGF este inscriptibil. √ √
3. a) Fie ABC un triunghi cu lungimile laturilor egale cu 2, 2, 1 + 3. Determinat, i
unghiurile triunghiului. √
b) În orice triunghi avem R(b + c) ≥ a bc, notat, iile fiind cele uzuale.
4. Fie ABC un triunghi s, i AD, perpendiculară pe planul său. Ducem AE ⊥ BD (E ∈ BD)
s, i AF ⊥ DC (F ∈ CD). Arătat, i că patrulaterul BCEF este inscriptibil.
5. Într-un triunghi ABC fie D s, i E mijloacele laturilor BC, respectiv CA. Ştiind că A(1, −1),
D(1, 2), E(2, 0), determinat, i:
a) să se scrie ecuat, iile laturilor triunghiului ABC;
b) ecuat, ia cercului circumscris triunghiului ABC.

(Admiterea la Universitatea din Pites, ti, specializările Matematică s, i Matematică-Informatică, 1995)


100 PROBLEME DE MATEMATICĂ PENTRU EXAMENE

Teste grilă pentru admiterea la facultate

Testul 1
1
Sorin Ulmeanu

x2 + x + 1
1. Să se determine mult, imea valorilor funct, iei f : R → R, f (x) = .
x2 − 1
 √ i S h√  h √ √ i
3 3
a) [−3, 3]; b) −∞, − ,c) − 23 , 23 ; d) R; e) (−∞, −1) (1, ∞).
∞ ;
S
2 2
2x π
2. Mult, imea solut, iilor inecuat, iei arccos > este:
x+1 3
a) (-1,1); b)Ø; c) [-1/3,1]; d) [-1/3,1/3); e) (-1,1/3]
3. Determinat , i cardinalul mult , imii
n−1
® ´
A = x ∈ Q| x = 2 ; n ∈ {−3, −2, 1, 2, 3, ..., 12, 13} .
n +n
a) 14; b) 12; c) 10; d) 15; e) 13.
4. Să se determine pentru ce valori reale s, i pozitive x, y pentru care x + y = 50, produsul
P = x3 y 2 ia valoarea maximă.
a) (25,25); b) (10,40); c) (30,20); d) (40,10); e) (20,30). Ñ é
12345678
5. Să se determine cea mai mică valoare n ∈ N∗ pentru care σ n = e, unde σ = ,
61438257
iar e este permutarea identică din S8 .
a) n = 8; b) n =h12; c) i
n = 9; d) n = 8!; e) n = 6.
1
6. Să se calculeze lim n a n − 1 = l, a > 0, a =
6 1.
n→∞

a) l = ln a; b) l = 1; c) l = e; d) l = ∞; e) l = 0.
7. Determinat, i funct, iile f : R → R∗ care verifică relat, iile f (x)
1
+ f 01(x) = e1x , ∀x ∈ R s, i

f (0) = 1 + 2.
√ x √ √
a)f (x) = ex − √ e + 1; b)f (x) = ex + e2x + 1; c)f (x) = −ex + ex + 1;
d)f (x) = −ex − ex + 1; e)f (x) = ex .
Ü ê
2̂ 0̂ 0̂
8. Fie matricea A = 1̂ 1̂ 0̂ cu elemente ı̂n Z3 . Atunci inversa acesteia este:
0̂ 0̂ 1̂
Ü ê Ü ê Ü ê
2̂ 0̂ 0̂ 1̂ 0̂ 0̂ 1̂ 0̂ 0̂
a) A−1 = 2̂ 2̂ 0̂ ; b) A−1 = 1̂ 1̂ 0̂ ; c) A−1 = 2̂ 2̂ 0̂
0̂ 0̂ 2̂ 0̂ 0̂ 1̂ 0̂ 0̂ 2̂
Ü ê
2̂ 0̂ 0̂
d) A−1 = 1̂ 1̂ 0̂ ; e) A nu este inversabilă.
0̂ 0̂ 1̂
R1 dx
9. Determinat, i valoarea lui I = .
−1 (ex + 1)(x2 + 1)
a) I = eπ/4; b) I = π/4; c) I = e2 π/2; d) I = 1; e) I = e2 .
1
Profesor, Colegiul Nat, ional ,,Ion C. Brătianu”, Pites, ti, sorinulm@yahoo.com
PROBLEME DE MATEMATICĂ PENTRU EXAMENE 101

R1
10. Determinat, i o relat, ie de recurent, ă ı̂n cadrul s, irului (In )n∈N∗ , unde In = (1 − x2 )n dx,
−1
n ∈ N∗ .
n 2n 2n n 1
a) In = I ;
n+1 n−1
b) In = I ;
c) In = 2n+1
n+1 n−1
In−1 ; d) In = 2n+1 In−1 ; e) In = I .
n+1 n−1
x+1
11. Mult, imea S a solut, iilor inecuat, iei ≥ 0 este:
x+2
a) (−∞, −2); b) (−∞, −2) (−1, +∞); c) (−∞, −2) [−1, ∞);
S S

d) (−∞, −2] [−1, ∞); e) (−∞, −2] (−1, ∞).


S S

12. Mult, imea valorilor funct, iei f : R → R, f (x) = x2 + x + 1, ∀ x ∈ R este:


î ä Ä ä î ä
3 3
a) R; b) 4
, +∞ ; c) 4
, +∞ ; d) − 43 , +∞ ; e) (0, +∞).
1
13. Valoarea limitei lim (x − 2) · sin este:
x→2 x−2
a) 1; b) 0; c) -1; d)+∞; e) −∞.
14. Suma pătratelor solut, iilor ecuat, iei 4x − 6 · 2x + 8 = 0 este:
a) 20; b)
Ñ 1/4; é c) 3/2; d) 5; e) 5/4.
1 1
15. Dacă A = , atunci A2 + A + I2 este:
0 1
Ñ é Ñ é Ñ é Ñ é Ñ é
1 2 2 3 1 3 3 3 3 2
a) ; b) ; c) ; d) ; e) .
0 1 0 2 0 1 0 3 0 3

Testul 2
2
D.M.I.

1. Fie funcţia f : R → R,

3x − 5



dacă x ∈ (−∞, 1)
f (x) =  (m − 2)x − 5 dacă x ∈ [1, 3]


x+7 dacă x ∈ (3, +∞).

Valoarea parametrului m ∈ R pentru care f este strict crescătoare este


a) m ∈ [5, 7]; b) m ∈ [−5, 5]; c) m ∈ (−∞, 7]; d) m ∈ (−2, 4]; e) m ∈ [5, +∞).

2. Fie sistemul  ñ
2y − 1
ô ñ ô
 3x + 1
+ = x,



2 3


ñ ô ñ ô

 3x 1 2y + 1

 + = y,
3 2

unde [a] reprezintă partea ı̂ntreagă a numărului real a. Dacă


n o
A = x2 + y 2 (x, y) soluţie a sistemului dat ,
2
Universitatea din Pites, ti, revista.matinf@upit.ro
102 PROBLEME DE MATEMATICĂ PENTRU EXAMENE

atunci
a) A = {1, 2, 3}; b) A = {1, 2}; c) A = {0, 3}; d) A = {0, 1}; e) A = {0, 4}.

3. Fie f : R → R, f (x) = mx2 − (m + 1)x + 1, m ∈ R. Dacă f (x) ≥ 0, ∀x ∈ R, atunci


a) m ∈ (−1, ∞); b) m ∈ (0, 1); c) m ∈ (0, ∞); d) m = 1; e) m = −1.
√ √
4. Soluţiile ecuaţiei 3 − x + 2 + x = 3 sunt
    
x1 = −2
  x = −2
1
 x =1
1
 x = −1
1
 x = −1
1
a)  ; b)  ; c)  ; d)  ; e)  .
x2 = 2 x2 = −1 x2 = 2 x2 = 3 x2 = 2

√ √
5. Soluţia inecuaţiei x−5− 9 − x ≥ 1 este
h √ i î ó
14+ 7
a) 2
, 9 ; b) 6, 17
2
; c) Ø; d) [7, 9]; e) [5, 9].

6. Dacă x este soluţia ecuaţiei


1 7
9x − 2x+ 2 = 2x+ 2 − 32x−1 ,

atunci 4x2 + 2x =
a) 9; b) 12; c) 2; d) 0; e) −12.

7. Soluţia ecuaţiei C3n + C4n = n(n − 2) este


a) n = 5; b) n = 4; c) n = 6; d) n = 7; e) n = 1.

8. Fie (bn )n≥1 o progresie geometrică ce satisface relaţiile b5 − b1 = 15 şi b4 − b2 = 6. Dacă


q > 1 este raţia progresiei geometrice, atunci
a) q = 2, 5; b) q = 3; c) q = 8; d) q = 4; e) q = 2.
Ä √ ä
9. Dacă n→∞
lim n an + 2 + bn + cn2 = 1, atunci a + b + c =

a) 1; b) 2; c) −1; d) 0; e) −2.

10.
12 + 22 + 32 + · · · + n2
lim =
n→∞ n3 + n2 + n + 1

1 1
a) ∞; b) 1; c) ; d) ; e) 0.
6 3

11. √
Ç 2 å x2 +1
x +x+1
lim =
x→∞ x2 −x+1

a) e2 ; b) 0; c) 1; d) e−2 ; e) e.
PROBLEME DE MATEMATICĂ PENTRU EXAMENE 103

12. Se consideră funcţia


|x| − 3
f : R → R, f (x) = .
1 + 2|x|

Mulţimea asimptotelor la graficul funcţiei este


a) {y = x + 1}; b) {y = x}; c) {x = − 12 }; d) {y = 12 }; e) {y = 12 , x = − 21 }

13. Dacă F este o primitivă a funcţiei

xearctg x 1 π
f : R → R, f (x) = » , şi lim F (x) = e 2 ,
(1 + x2 )3 x→∞ 2

atunci F (x) =
 x 
x 1 arctg
a) earctg x ; b) earctg x + e 1+x2 ; c) √x−1 earctg x ; d) 2x+1
√ earctg x ;
1−2x 2 2 1+x2 4 1+x2
 x 
x2 arctg
e) 1−2x
earctg x + e 1+x2 .

14. Dacă
Z1 Ä ä
3x2 + 2x − 1 dx = 3,
α

atunci
1
a) α = 0; b) α = ; c) α = −1; d) α = −2; e) α = −3.
2

15. Aria mulţimii mărginită de graficele funcţiilor

f, g : R → R, f (x) = x3 , g(x) = x2

şi dreptele x = 0 şi x = 1 este


x+1
a) 4
; b) 92 ; c) 1
12
; d) 10
3
; e) 32
3
.

Testul 3
3
D.M.I.

1. Suma soluţiilor reale ale ecuaţiei ||x| − 1| = 13 este


a) 6; b) 0; c) 1; d) −1; e) −2.

x − x2
2. Soluţiile inecuaţiei 2 ≥ 0 sunt
x +x−6
a) x ∈ (−1, 4); b) x ∈ [−3, −1) ∪ (1, 2); c) x ∈ (−3, 2); d) x ∈ (−3, 0] ∪ [1, 2);
e) x ∈ (−3, 0] ∪ [2, 3).
3
Universitatea din Pites, ti, revista.matinf@upit.ro
104 PROBLEME DE MATEMATICĂ PENTRU EXAMENE

3. Soluţiile sistemului de inecuaţii



 |x + 1| − x > 2
 x2 + 2x − 80 < 0

sunt
Ç å
3
a) x ∈ (−1, 5]; b) x ∈ (−8, 10); c) x ∈ −10, ; d) x ∈ (−10, 8); e) x ∈ (−10, −4).
2
»
3
√ »
3

4. Valoarea numărului n = 45 + 29 2 + 45 − 29 2 este
a) n = 3; b) n = 4; c) n = 5; d) n = 6; e) n = 2.

5. Numărul soluţiilor ı̂ntregi ale ecuaţiei


√ 3
x2 + 3 − 2x2 − 3x + 2 = (x + 1)
2
este egal cu
a) 1; b) 3; c) 0; d) 4; e) 2.

6. Soluţia inecuaţiei √
log 1 x+3
3
<1
log 1 (x + 1)
3

este
a) (−1, 0] ∪ (1, ∞); b) (−1, 1); c) (−1, 0) ∪ (1, ∞); d) [0, ∞); e) (−1, 0) ∪ (2, ∞).

C y = Cxy+2

7. Soluţia sistemului  x2 este
Cx = 153
    
 x = 12  x = 18  x = 18  x = 18  x = 16
a)  ; b)  ; c)  ; d)  ; e)  .
y=8 y=8 y = 16 y = 10 y = 12

8. Termenii unei progresii geometrice satisfac relaţiile

a1 + a5 = 51 şi a2 + a6 = 102.

Dacă Sn = 3069, atunci n =


a) 8; b) 11; c) 10; d) 12; e) 9.

9. Limita şirului (an )n>0 cu termenul general


√n
n!
an =
n+1
este
PROBLEME DE MATEMATICĂ PENTRU EXAMENE 105

a) ∞; b) 1; c) e; d) 0; e) e−1 .

10. √
12 + 22 + 32 + · · · + n2
lim √ =
n→∞ n 3n

1 1 1
a) ; b) 1; c) ; d) ∞; e) .
2 6 3

11. Numerele reale a şi b pentru care


Ä√ ä
lim x2 + 1 + ax + b = 2
x→−∞

sunt:
    
a=1
  a = −1  a=1  a=0  a=3
a)  ; b)  ; c)  ; d)  ; e)  .
b = −2 b = −2 b=2 b = −1 b = −2

12. Funcţia √ 
2x − a2 x2 + ax + 1 dacă x ≤ 1

f : R → R, f (x) =  √ √
x − 1 + |a| x dacă x > 1
este continuă pe R dacă şi numai dacă
¶ © ¶ © ¶ © ¶ © ¶ ©
a) a ∈ −1, 35 ; b) a ∈ −2, 35 ; c) a ∈ −1, − 35 ; d) a ∈ −2, 52 ; e) a ∈ − 13 , 51 .

13. Primitivele funcţiei


1
f : R → R, f (x) = , (a ∈ R? )
a2 + x2
sunt
Ä √ ä
a) a1 arctg xa + C; b) arctg xa + C; c) arctg x + C; d) tg xa + C; e) ln ax + a2 + x2 + C.

14. Se consideră funcţia f : R\{−1} → R,


1 − x + x2 − x3 + · · · + x2n
f (x) = lim .
n→∞ 1 + x + x2 + x3 + · · · + x2n

Z2
Atunci f (x)dx =
0

e 2 3 4 4
a) 2 ln ; b) 2 ln ; c) 2 ln ; d) 2 ln ; e) ln .
3 3 e 3 3

15. Volumul corpului de rotaţie determinat de funcţia


Ä√ √ ä2
f : [0, a] → R, f (x) = a − x , a > 0,

este egal cu
πa3 πa3 πa3 πa3 πa3
a) 7
; b) 14
; c) 4
; d) 15
; e) 5
.
106 PROBLEME DE MATEMATICĂ PENTRU EXAMENE

Testul 4
4
D.M.I.

1. Se consideră funcţiile

f, g : R → R, f (x) = |x − 1| + 2, g(x) = |x − 2| + 1.

Atunci funcţia f ◦ g are legea de corespondenţa


 
x − 4 dacă x ≥ 4
  −4 dacă x ≥ 2
a) (f ◦ g)(x) =  . b) (f ◦ g)(x) =  .
x + 4 dacă x < 4 x dacă x < 2
 
−x dacă x ≥ 2
  4 dacă x ≥ 2
c) (f ◦ g)(x) =  . d) (f ◦ g)(x) =  .
4 dacă x < 2 x dacă x < 2

 x dacă x ≥ 2
e) (f ◦ g)(x) =  .
4 − x dacă x < 2

2. Se consideră ecuaţia 2x2 − 2(2m − 1)x + 2m2 + 2m + 3 = 0, unde m este un parametru real.
Rădăcinile x1 , x2 sunt reale şi
√ x1 √
2− 3≤ ≤ 2 + 3,
x2
dacă şi numai dacă
ñ ô
5
a) m ∈ [−4, −1],; b) m ∈ (−∞, −4] ∪ −1, − ; c) m ∈ (−∞, −4] ∪ [−1, ∞);
8
Ç ô ñ å
5 5
d) m ∈ −∞, − ; e) m ∈ − , ∞ .
8 8

3. Suma soluţiilor reale ale ecuaţiei x − 3 = x − 3 este
a) 8; b) 6; c) 7; d) 10; e) 9.
»
3
√ » √
3
4. Se consideră numerele reale x = 5 2 + 7, y = 5 2 − 7, A = x + y şi B = x − y. Atunci
a) A ∈ Q, B ∈ R\Q; b) A ∈ R\Q, B ∈ Q; c) A, B ∈ R\Q; d) x, y ∈ Q; e) A, B ∈ Q.

√ √ √
5. Ecuaţia x + 1 + 1 − x2 = 3 x2 + 7 − 2
a) are soluţie unică. b) nu are soluţii. c) are două soluţii in intervalul [−2, 0]. d) are trei
soluţii. e) are două soluţii in intervalul [0, 2].

6. Mulţimea soluţiilor ecuaţiei


log3 x
» = −1
log√3x x
este
4
Universitatea din Pites, ti, revista.matinf@upit.ro
PROBLEME DE MATEMATICĂ PENTRU EXAMENE 107

¶ © n o n o ¶ © n o
1 1 √1 , 1 1 √1 1 √1
a) ,
9 3
; b) 3 9
; c) 3
, 3 ; d) 9
; e) 3
.

Ä√ √ ä8
7. Termenul din mijloc al dezvoltării x+ y este
√ √
a) T5 = C84 x2 · y 2 ; b) T7 = C86 x3 · y 3 ; c) T4 = C83 x2 · y xy; d) T6 = C85 x · y 2 xy;
e) T3 = C82 x3 · y.

8. Fie ecuaţia x3 + x + m = 0, m ∈ R cu rădăcinile x1 , x2 , x3 . Dacă x51 + x52 + x53 = 10, atunci


m=
a) 1; b) 0; c) -1; d) 2; e) −2.

9. Limita şirului (an )n>0 cu termenul general

1! + 2! + 3! + · · · + n!
an =
(n + 1)!
este
a) 1; b) 0; c) ∞; d) e; e) e−1 .

10.
n4
lim =
n→∞ 13 + 23 + 33 + · · · + n3

1
a) 4; b) 0; c) 1; d) ; e) ∞.
2

11. Dacă
sin 5x − sin 3x
L = lim
x→0 5x
atunci
a) L = 25 ; b) L = 15 ; c) L = − 15 ; d) L = − 25 ; e) L = 0.

12. Valoarea parametrului real a, pentru care funcţia


 1
 (sin x + ex ) x dacă x 6= 0
f : R → R, f (x) = 
a dacă x = 0

este continuă pe R, este


a) a = e3 ; b) a = 1; c) a = e−5 ; d) a = e2 ; e) a = 0.

13.
sin x
Z
dx =
1 + cos2 x

√ √
a) ln 1 + cos2 x + C; b) ln(1 + cos2 x) + C; c) arctg (cos x) + C; d) ln 1 − cos2 x + C;
108 PROBLEME DE MATEMATICĂ PENTRU EXAMENE

e) −arctg (cos x) + C.

14. Dacă
f : [0, 2] → R, f (x) = max{1, x2 },
atunci Z 2
ex f (x)dx =
0

a) 4e + 5; b) 1 − 3e2 ; c) 2e2 ; d) 2e2 − 1; e) 2e .

15. Aria domeniului plan mărginit de graficele funcţiilor



f, g : [0, 1] → R, f (x) = 1 − x, g(x) = 1 − x2

este

a) e − 1; b) 21 ; c) 2; d) π−2
4
; e) 2
2
.

Testul 5

5
Raluca Mihaela Georgescu



2

x − 3 −2

1. Să se determine x ∈ R astfel ı̂ncât


= 0.

x+2 1

a) x ∈ {−1, 1}; b) x = −1; c) x = 1; d) x ∈ {−1, 0}; e) x = 0


tg 2 2x
2. Să se calculeze lim .
x→0 sin2 5x

a) 2/5; b) 2/25; c) 4/5; d) 4/25; e) 1


3. Să se determine m ∈ R astfel ı̂ncât parabola asociată funct, iei f (x) = x2 − (m + 3)x + 1 să
fie tangentă axei Ox.

a) m ∈ {−1, −5}; b) m = −1; c) m = 5; d) m ∈ {1, −5}; e) m = 0


(x + 2)2 − (x − 1)2
4. Să se calculeze lim .
x→∞ 2x3 + 3x − 1
a) ∞; b) 0; c) 1/2; d) 1; e) 5/2
5. Să se determine termenul liber al polinomului f = (x + 1)10 + (3x − 1)10 .

a) 0; b) 2;c) -1; d) -2; e) 1


2
ln xRe
6. Să se calculeze dx.
1 x
a) 2/3; b) 1/2; c) 1/3; d) 0; e) 1
5
Lect. univ. dr., Universitatea din Pites, ti, gemiral@yahoo.com
PROBLEME DE MATEMATICĂ PENTRU EXAMENE 109

Ü ê
1 i
7. Fie A = . Să se calculeze A4 .
−i 0
Ü ê Ü ê Ü ê Ü ê Ü ê
3 2i 5 3i −5 3i 5 3i −5 3i
a) ; b) ; c) ; d) ; e)
−2i 1 3i i 1 −3i 2 1 3i 2

8. Fie F primitiva funct, iei f : R → R, f (x) = x2 + 1, cu proprietatea că F (0) = 0. Să se
determine F (1).
Ä√ √ ä √ √ Ä√ √ ä
a) 12 2 + ln(1 + 2) ; b) 3 + ln(1 + 3); c) 1
2
2 + ln 2 ;
Ä √ ä Ä√ √ ä
d) 21 1 + ln(1 + 3) ; e) 12 2 + ln(1 − 2 2)
9. Valorile lui k ∈ R pentru care inecuat, ia x2 − (k − 1)x − k + 1 < 0 nu are solut, ie sunt:

a) [-1,3]; b) (2,4]; c) [-3,1]; d) (-3,1); e) Ø


10. Mult, imea solut, iilor ecuat, iei x2 + 3̂x + 2̂ = 0̂ peste corpul Z5 este

a) {2̂}; b) {3̂}; c) {3̂, 4̂}; d) {4̂}; e) {2̂, 4̂}


11. Aria subgraficului determinat de graficul funct, iei f : R → R, f (x) = x ln x, axa Ox s, i
dreptele de ecuat, ii x = 1, x = e este:
e2 − 1 e+1 e2 + 1 e2 + 1 e+1
a) ; b) ; c) ; d) ; e)
4 4 Ç4 åx Ç å2x 2
2 25 125
12. Dacă x este solut, ia ecuat, iei · = , atunci 3x + 6x este:
5 4 8
a) 9; b) 45; c) 243; d) 241; e) 120
13. Restul ı̂mpărt, irii polinomului f = (x + 1)10 + (x − 1)10 la g = x2 − 3x + 2 este:

a) (310 − 210 + 1)x + 211 − 310 − 1; b) (310 + 210 − 1)x + 211 + 310 − 1;

c) (310 − 210 )x + 211 − 310 ; d)(310 + 1)x + 211 − 1; e) (310 + 210 + 1)x + 211 + 310 + 1
14. Suma solut, iilor reale ale ecuat, iei |||x − 1| − 2| − 1| = 2 este:

a) 3; b) 2; c) 24; d) 6; e) 4
15. Dacă ı̂ntr-o progresie aritmetică a1 = 3, a5 − a3 = 10, atunci suma primilor 20 de termeni
ai progresiei este:

a) 1000; b) 1200; c) 1010; d) 1020; e) 1100


110 PROBLEME DE INFORMATICĂ PENTRU EXAMENE

PROBLEME DE INFORMATICĂ PENTRU


EXAMENE

Teste pentru examenul de Bacalaureat, specializarea S, tiint, e ale naturii

Testul 1

1
Marina Luncan

Limbajul C/C++
SUBIECTUL I (30 de puncte)
Pentru itemul 1, scriet, i pe foaia de examen litera corespunzătoare răspunsului
corect.
1. Care este valoarea expresiei C/C++: (3 + 17/2) ∗ 2? (4p.)
a) 23 b) 22 c) 20 d) 21
2. Se consideră algoritmul de mai jos, reprezentat ı̂n pseudocod. S-a notat cu x%y restul
ı̂mpărt, irii lui x la y.
citeste n , k ( numere naturale )
pentru i←2 , n executa
| daca i % k =0 sau k % i =0 atunci
| _ | _ s←s +1
daca s >0 atunci scrie s
| _ altfel scrie " NU "

a) Scriet, i pe foaia de examen ce se va afis, a dacă se citesc ı̂n această ordine numerele 10
s, i 3. (6p.)
b) Scriet, i o pereche formată din numere de două cifre care să determine afis, area mesajului
NU. (4p.)
c) Scriet, i ı̂n pseudocod un algoritm echivalent cu cel dat care să ı̂nlocuiască structura
pentru cu o structură repetitivă cu test init, ial. (6p.)
d) Scriet, i programul C/C++ corespunzător algoritmului dat. (10p.)

SUBIECTUL al II-lea (30 de puncte)


Pentru fiecare dintre itemii 1 s, i 2 scriet, i pe foaia de examen litera corespunzătoare
răspunsului corect.
1. Care dintre următoarele expresii aritmetice determină pătratul sumei dintre a s, i b? (4p.)
a) pow(a,2)+pow(b,2) c) sqrt(a+b)
b) (a+b)*2 d) pow(a,2)+pow(b,2)+2*a*b
2. Se consideră citit caracterul c (literă mică a alfabetului englez). Care dintre următoarele
secvent, e de program determină transformarea caracterului c ı̂n literă mare? (4p.)
1
Profesor, Colegiul Nat, ional ,,Alexandru Odobescu”, Pites, ti, inalucan@gmail.com
PROBLEME DE INFORMATICĂ PENTRU EXAMENE 111

S1: c=c+32; S3: for(i=1;i<=16;i++)c=c-2;


S2: c=c-32; S4: for(i=1;i<=32;i++)c++;
a) S2 s, i S3; b) doar S1; c) doar S2; d) S1 s, i S4.

Scriet, i pe foaia de examen răspunsul pentru fiecare dintre cerint, ele următoare.
3. O fract, ie este dată prin două numere ı̂ntregi a s, i b, reprezentând număratorul, respectiv
numitorul acesteia. Scriet, i o secvent, ă de instruct, iuni care să verifice dacă fract, ia se poate
simplifica prin numărul d s, i afis, at, i numărătorul s, i numitorul fract, iei simplificate sau
mesajul NU, dacă aceasta nu se poate simplifica prin d. (6p.)
4. Un număr se numes, te ”rar” dacă mai mult de jumătate din cifrele sale sunt egale cu 0.
a) Scriet, i un algoritm pseudocod care cites, te repetat numere, până la ı̂ntâlnirea numărului
0 s, i afis, ează pe ecran numerele rare din care s-au extras cifrele egale cu 0. (10p.)
b) Ment, ionat, i rolul fiecărei variabile s, i specificat, i care sunt datele de intrare s, i care sunt
datele de ies, ire ale problemei. (6p.)
SUBIECTUL al III-lea (30 de puncte)
Pentru itemul 1, scriet, i pe foaia de examen litera corespunzătoare răspunsului
corect.
1. Se consideră s, irul de numere ı̂ntregi 20, 4, 3, 1, 10, 5. Se ordonează s, irul descrescător,
folosind metoda bulelor. După câte parcurgeri de s, ir se va obt, ine ordonarea. (4p.)
a) 3 b) 4 c) 2 d) 6

Scriet, i pe foaia de examen răspunsul pentru fiecare dintre cerint, ele următoare.
2. Înlocuit, i punctele de suspensie, ı̂n secvent, a următoare astfel ı̂ncât să se afis, eze secvent, a
de numere din dreapta. (6p.)

for ( i =1; i <=4; i ++) 1 1 1


for ( j =3; j >=1; j - -) 2 2 2
..................... 3 3 3
4 4 4
3. Se cites, te de la tastatură un s, ir cu n elemente, numere reale (n<100). Scriet, i un program
C/C++ care determină modificarea s, irului init, ial astfel ca numerele divizibile cu 10 să
se afle la ı̂nceputul s, irului, ordinea celorlalte elemente nefiind semnificativă. Elementele
s, irului vor fi afis, ate pe ecran, pe o singura linie, despărt, ite prin câte un spat, iu. Dacă ı̂n
s, ir nu există numere divizibile cu 10, se va afis, a pe ecran mesajul NU EXISTA. (10p.)
Exemplu: Dacă, pentru n = 6 se citesc numerele 2 10 3 20 5 6, se va afis, a 10 20 2 3
5 6.
4. Fis, ierul bac.in cont, ine pe prima linie trei numere ı̂ntregi n, a, b, iar pe următoarea linie
n numere reale. Afis, at, i pe ecran cel mai mare număr real care se afla in intervalul [a, b]
s, i numărul sau de aparit, ii, despărt, ite prin câte un spat, iu. Dacă fis, ierul bac.in nu cont, ine
numere din intervalul [a, b] se va afis, a mesajul NU EXISTA.
a) Descriet, i ı̂n limbaj natural un algoritm eficient de rezolvare a problemei. (4p.)
b) Scriet, i un program C/C++ pentru rezolvarea problemei. (6p.)
Exemplu: Dacă ı̂n fis, ierul bac.in se află numerele
112 PROBLEME DE INFORMATICĂ PENTRU EXAMENE

10 2 12
1 13 5 13 9 8 1 9 9 15
pe ecran se va afis, a 9 3.

Testul 2

2
Sofia-Elena Ciobanu

Limbajul C/C++
SUBIECTUL I (30 de puncte)
Pentru itemul 1, ı̂ncercuit, i litera corespunzătoare răspunsului corect.
1. Expresia C/C++: 6/8*3%2 are valoarea: (4p.)
a) 3 b) 0 c) 1 d) 2
2. Se consideră algoritmul următor, reprezentat ı̂n pseudocod:
intreg n , s , uc
citeste n ( numar natural cu cel putin 2 cifre )
cat timp n>0 executa
| s←0
| cat timp n>0 executa
| | uc←n %10
| | s←s + uc
| | _ n←n /10
| daca s > 9 atunci
|_ | _ n←s
scrie s

a) Scriet, i ce se afis, ează dacă se cites, te numărul 12349. (6p.)


b) Scriet, i trei numere distincte astfel ı̂ncât, ı̂n urma executării algoritmului, pentru
fiecare dintre acestea să se afis, eze valoarea 7. (4p.)
c) Scriet, i ı̂n pseudocod un algoritm echivalent cu cel dat, ı̂nlocuind structura cât timp
. . . execută cu o altă structură repetitivă. (6p.)
d) Scriet, i programul C/C++ corespunzător algoritmului dat. (10p.)

SUBIECTUL al II-lea (30 de puncte)


Pentru fiecare dintre itemii 1 s, i 2 ı̂ncercuit, i litera corespunzătoare răspunsului
corect.

1. Care este numărul total de atribuiri efectuate la executarea următoarei secvent, e de


instruct, iuni? (4p.)
t =1; z =19;
while ( z ==19) z = z * z ;
if ( t == z ) t = t *3;

2
Profesor, Colegiul Nat, ional Zinca Golescu”, Pites, ti, stefanescu sofia@yahoo.com

PROBLEME DE INFORMATICĂ PENTRU EXAMENE 113

a) 3 b) 4 c) 1 d) 2
2. Variabilele ı̂ntregi x s, i y memorează numere naturale Ce expresie de mai jos este echivalentă
cu expresia (x+y)%3==1 ? (4p.)
a) (x%3==0) && (y%3==0) && (x%3==1) && (y%3==1)
b) (x%3==0) && (y%3==1) k (x%3==1) && (y%3==0)
c) (x%3==1) k (y%3==0) && (x%3==0) k (y%3==0)
d) (x%3==0) && (y%3==0) && (x%3==0) k (y%3==1)
Scriet, i răspunsul pentru fiecare din cerint, ele următoare.
3. Pentru o dulceat, ă sunt memorate pret, ul, precum s, i date despre numele fructului s, i
cantitatea de zahăr folosită pentru un borcan.
Variabilele ı̂ntregi pret1, fruct1 s, i zahar1 memorează pret, ul, tipul de fruct folosit s, i
cantitatea de zahăr specifică unui anumit tip de dulceat, ă, iar variabilele pret2, fruct2 s, i
zahar2 memorează pret, ul, tipul de fruct folosit s, i cantitatea de zahăr specifică unui alt
tip de dulceat, ă.
Să se scrie o secvent, ă de instruct, iuni C/C++ care, ı̂n urma executării, să afis, eze pret, ul
dulcet, ei cu o cantitate mai mică de zahăr, iar dacă ambele borcane de dulceat, ă au aceeas, i
cantitate de zahăr să se afis, eze mesajul ,,ZAHAR IN MOD EGAL”. (6p.)
4. Se citesc n numere naturale. Calculat, i produsul tuturor numerelor care ı̂ncep cu cifra 7.
a) Scriet, i ı̂n pseudocod algoritmul de rezolvare. (10p.)
b) Precizat, i rolul tuturor variabilelor care au intervenit ı̂n algoritmul realizat s, i indicat, i
datele de intrare s, i datele de ies, ire ale problemei enunt, ate. (6p.)

SUBIECTUL al III-lea (30 de puncte)


Pentru itemul 1, scriet, i litera corespunzătoare răspunsului corect.

1. Variabilele a s, i b sunt de tip ı̂ntreg s, i memorează numere naturale nenule. Indicat, i


instruct, iunea care poate ı̂nlocui punctele de suspensie, astfel ı̂ncât ı̂n urma executării
secvent, ei obt, inute să se interschimbe valorile celor două variabile.
..........
b =a - b ;
a =a - b ;

a) a=a+b b) a=a-b c) b=b+a d) a=b-a (4p.)

Scriet, i răspunsul pentru fiecare dintre cerint, ele următoare.


2. Variabilele i s, i j sunt de tip ı̂ntreg. Scriet, i expresia care poate ı̂nlocui punctele de suspensie,
astfel ı̂ncât, ı̂n urma executării secvent, ei, să se afis, eze suma elementelor pare ce se află pe
primul pătrat concentric al unei matrice cu 8 linii s, i 8 coloane, numerotate de la 1 la 8.
(6p.)
for ( i =1; i <=8; i ++)
for ( j =1; j <=8; j ++)
if (.................)
if ( a [ i ][ j ]%2==0)
s = s + a [ i ][ j ];
cout < < s ;
114 PROBLEME DE INFORMATICĂ PENTRU EXAMENE

3. Scriet, i un program C/C++ care cites, te de la tastatură un număr natural n s, i cele n


elemente ale unui tablou unidimensional, numere ı̂ntregi. Ordonat, i descrescător elementele
aflate pe pozit, iile impare ale tabloului unidimensional. (10p.)
4. Fis, ierul Numere.txt cont, ine pe o singură linie, separate prin câte un spat, iu cel mult 50
de numere ı̂ntregi. Scriet, i un program care cites, te numerele din fis, ierul Numere.txt s, i
afis, ează pe ecran, separate prin câte un spat, iu ı̂n ordine crescătoare, toate numerele din
fis, ier care au proprietatea de a fi palindrom. Dacă fis, ierul nu cont, ine astfel de numere se
va afis, a mesajul NU EXISTA.
a) Descriet, i ı̂n limbaj natural algoritmul utilizat. (4p.)
b) Scriet, i programul C/C++ corespunzător. (6p.)

Testul 3

3
Maria Miroiu

Limbajul C/C++
SUBIECTUL I (30 de puncte)
Pentru itemul 1, ı̂ncercuit, i litera corespunzătoare răspunsului corect.
1. Care este expresia echivalentă cu expresia C/C++: !(x>=0 && x<=9)? (4p.)
a) x<0 && x>9; c) x<0 ||x>9;
b) x>=0 ||x<=9; d) !(x>=0) && (x>9.
2. Se consideră algoritmul de mai jos, descris ı̂n pseudocod. S-a notat cu x%y restul ı̂mpărt, irii
lui x la y.
citeste n ( numar natural nenul )
s←0
cat timp n>0 executa
| c←n %10
| daca c %2=1 atunci
| | pentru i←1 , c executa
| | _ |_ s←s + i
| _ n←n /10
scrie s

a) Scriet, i ce se va afis, a dacă pentru variabila n se cites, te valoarea (6p.)


b) Scriet, i un număr ı̂ntreg strict pozitiv ce poate fi citit ca valoare a variabilei n astfel
ı̂ncât algoritmul să afis, eze valoarea 0. (4p.)
c) Scriet, i ı̂n pseudocod un algoritm, echivalent cu cel dat, ı̂nlocuind structura pentru
... executa cu o structură repetitivă cu test init, ial. (6p.)
d) Scriet, i programul C/C++ corespunzător algoritmului dat. (10p.)

SUBIECTUL al II-lea (30 de puncte)


Pentru fiecare dintre itemii 1 s, i 2 scriet, i pe foaia de examen litera corespunzătoare
răspunsului corect.

1. Presupunând că lungimile catetelor unui triunghi dreptunghic sunt memorate ı̂n variabilele
c1 s, i c2, care este expresia C/C++ cu care se poate determina lungimea ipotenuzei?
3
Lect. univ. dr., Universitatea din Pites, ti, maria.miroiu@yahoo.com
PROBLEME DE INFORMATICĂ PENTRU EXAMENE 115

a) pow(c1,2)+pow(c2,2) c) sqrt(c1+c2)
b) sqrt(pow(c1,2)+pow(c2,2)) d) pow(c1,2)+pow(c2,2)+2*c1*c2
2. Se consideră secvent, a de cod C/C++ de mai jos, ı̂n care n memorează valoarea unui
număr natural nenul dat, iar toate celelalte variabile sunt de tip ı̂ntreg. (4p.)
ucp = 1;
for ( d =1; d <= n ; d ++)
if ( n % d ==0)
......

Expresia care poate ı̂nlocui punctele de suspensie astfel ı̂ncât la sfârs, itul secvent, ei C/C++
obt, inute variabila ucp să memoreze ultima cifră a produsului tuturor divizorilor variabilei
n este: (4p.)
a) ucp=(ucp+d)%10; c) ucp=d%10*ucp;
b) ucp=n/d%10*upc; d) ucp=(ucp*d)%10;

Scriet, i răspunsul pentru fiecare din cerint, ele următoare.


3. Se consideră două date de nas, tere definite prin variabile ı̂ntregi zi1, luna1, an1, respectiv
zi2, luna2, an2. Scriet, i o expresie C/C++ care să aibă valoarea 1 dacă cele două persoane
sunt născute ı̂n aceeas, i zi s, i aceeas, i lună, dar ani diferit, i. (6p.)
4. Pentru un număr natural, cifra de ordin 1 este cifra unităt, ilor, cifra de ordin 2 este cifra
zecilor, cifra de ordin 3 este cifra sutelor, s, .a.m.d. Rotunjirea unui număr la cifra de ordin
k se realizează ı̂n funct, ie de valoarea cifrei imediat din dreapta. De exemplu, pentru
n=27432 rotunjirea la cifra de ordin k=2 (cifra zecilor) conduce la numărul 27430 pentru
că cifra unităt, ilor este mai mică decât 5, iar pentru n=27472 rotunjire la cifra de ordin
k=3 (cifra sutelor) conduce la 27500 pentru că cifra zecilor este mai mare sau egală decât
5.

a) Scriet, i un algoritm ı̂n pseudocod care cites, te două numere naturale n s, i k (n≥1,
k≥1) s, i determină numărul natural obt, inut prin rotunjirea lui n la ordinul k dacă
variabila k are valoarea mai mică decât numărul de cifre ale lui n, iar ı̂n caz contrar
afis, ează mesajul ,,Eroare”. (10p.)
b) Ment, ionat, i rolul fiecărei variabile s, i specificat, i care sunt datele de intrare s, i care sunt
datele de ies, ire ale problemei. (6p.)

SUBIECTUL al III-lea (30 de puncte)


Scriet, i pe foaia de examen răspunsul corect pentru fiecare dintre cerint, ele
următoare:

1. Indicat, i expresia care poate ı̂nlocui punctele de suspensie astfel ı̂ncât, ı̂n urma executării
secvent, ei obt, inute, să se afis, eze matricea de mai jos. (4p.)
for ( i =1; i <=4; i ++) 0 0 0 0
{ for ( j =1; j <=4; j ++) 0 1 2 3
cout < <......; | printf ("% d " ,......); 0 1 3 4
cout < < endl ; | printf ("% n "); }
0 2 4 6
116 PROBLEME DE INFORMATICĂ PENTRU EXAMENE

a) i*j/2 c) (i-1)*j/2
b) i*(j-1)/2 d) (i-1)*(j-1)/2

Scriet, i pe foaia de examen răspunsul corect pentru fiecare dintre cerint, ele
următoare
2. În secvent, a de mai jos, toate variabilele sunt de tip ı̂ntreg. Înlocuit, i punctele de suspensie
astfel ı̂ncât, ı̂n urma executării secvent, ei obt, inute, valoarea variabilei ok să fie 1 dacă
există cel put, in un număr par ı̂ntre cele 10 numere citite, sau valoarea 0 ı̂n caz contrar.
(6p.)
ok = ......;
for ( i =1; i <=10; i ++)
{ cin > > k ; | scanf ("% d " ,& k );
......; }
3. Se citesc de la tastatură valorile a două variabile ı̂ntregi, n (2<n<100) s, i k (1<k<n),
apoi un s, ir de n elemente, numere ı̂ntregi. Scriet, i un program C/C++ care determină
modificarea s, irului init, ial, astfel ı̂ncât primele k elemente să fie ordonate descrescător, iar
ultimele n-k elemente să fie ordonate crescător. Elementele s, irului modificat vor fi afis, ate
pe ecran, pe o singură linie, despărt, ite prin câte un spat, iu. (10p.)
Exemplu: Pentru n = 8, k=3 s, i elementele 4 7 3 8 9 16 10 6, se va afis, a 7 4 3 6 8
9 10 16.
4. Fis, ierul bac.txt cont, ine pe prima linie valoarea naturală nenulă a variabilei n, iar pe
următoarea linie un s, ir de n numere naturale, despărt, ite prin spat, ii. Afis, at, i pe ecran
valoarea pară cea mai mare s, i numărul/numerele de ordine din s, ir al/ale valorii pare
maximă, despărt, ite printr-un spat, iu. Dacă fis, ierul bac.txt nu cont, ine numere pare, se va
afis, a pe ecran mesajul Nu exista.
a) Descriet, i ı̂n limbaj natural un algoritm eficient de rezolvare a problemei. (4p.)
b) Scriet, i un program C/C++ pentru rezolvarea problemei. (6p.)

Exemplu: Dacă ı̂n fis, ierul bac.txt se află numerele:

9
3 16 2 7 15 4 16 6 8

pe ecran se va afis, a 16 2 7.

Testul 4

4
Maria Miroiu

Limbajul Pascal
SUBIECTUL I (30 de puncte)
Pentru itemul 1, scriet, i pe foaia de examen litera corespunzătoare răspunsului
corect.

1. Care este expresia echivalentă cu expresia Pascal: not((x>=0) and (x<=9))? (4p.)
4
Lect. univ. dr., Universitatea din Pites, ti, maria.miroiu@yahoo.com
PROBLEME DE INFORMATICĂ PENTRU EXAMENE 117

a) (x<0) and (x>9) c) (x<0) or (x>9)


b) (x>=0) or (x<=9) d) not(x>=0) and (x>9)
2. Se consideră algoritmul următor, descris ı̂n pseudocod. S-a notat cu x%y restul ı̂mpărt, irii
lui x la y.
a) Scriet, i ce se va afis, a dacă pentru variabila n se cites, te valoarea 1234. (6p.)
b) Scriet, i un număr ı̂ntreg strict pozitiv ce poate fi citit ca valoare a variabilei n astfel
ı̂ncât algoritmul să afis, eze valoarea 0. (4p.)
c) Scriet, i ı̂n pseudocod un algoritm, echivalent cu cel dat, ı̂nlocuind structura pentru
... executa cu o structură repetitivă cu test init, ial. (6p.)
d) Scriet, i programul Pascal corespunzător algoritmului dat. (10p.)
citeste n ( numar natural nenul )
s←0
cat timp n>0 executa
| c←n %10
| daca c %2=1 atunci
| | pentru i←1 , c executa
| | _ |_ s←s + i
| _ n←n /10
scrie s

SUBIECTUL al II-lea (30 de puncte)


Pentru fiecare dintre itemii 1 s, i 2 scriet, i pe foaia de examen litera corespunzătoare
răspunsului corect.

1. Presupunând că lungimile catetelor unui triunghi dreptunghic sunt memorate ı̂n variabilele
c1 s, i c2, care este expresia Pascal cu care se poate determina lungimea ipotenuzei?(4p.)
a) sqr(c1)+sqr(c2) c) sqrt(c1+c2)
b) sqrt(sqr(c1)+sqr(c2)) d) sqr(c1)+sqr(c2)+2*c1*c2
2. Se consideră secvent, a de cod Pascal de mai jos, ı̂n care n memorează valoarea unui număr
natural nenul dat, iar toate celelalte variabile sunt de tip ı̂ntreg.
ucp :=1;
for d :=1 to n do
if n mod d =0 then
......

Expresia care poate ı̂nlocui punctele de suspensie astfel ı̂ncât la sfârs, itul secvent, ei Pascal
obt, inute variabila ucp să memoreze ultima cifră a produsului tuturor divizorilor variabilei
n este: (4p.)
a) ucp:=(ucp+d) mod 10; c) ucp:=d mod 10*ucp;
b) ucp:=n/d mod 10*upc; d) ucp:=(ucp*d) mod 10.

Scriet, i pe foaia de examen răspunsul corect pentru fiecare dintre cerint, ele
următoare:
3. Se consideră două date de nas, tere definite prin variabile ı̂ntregi zi1, luna1, an1, respectiv
zi2, luna2, an2. Scriet, i o expresie Pascal care să aibă valoarea 1 dacă cele două persoane
sunt născute ı̂n aceeas, i zi s, i aceeas, i lună, dar ani diferit, i. (6p.)
118 PROBLEME DE INFORMATICĂ PENTRU EXAMENE

4. Pentru un număr natural, cifra de ordin 1 este cifra unităt, ilor, cifra de ordin 2 este cifra
zecilor, cifra de ordin 3 este cifra sutelor, s, .a.m.d. Rotunjirea unui număr la cifra de ordin
k se realizează ı̂n funct, ie de valoarea cifrei imediat din dreapta. De exemplu, pentru
n=27432 rotunjirea la cifra de ordin k=2 (cifra zecilor) conduce la numărul 27430 pentru
că cifra unităt, ilor este mai mică decât 5, iar pentru n=27472 rotunjire la cifra de ordin
k=3 (cifra sutelor) conduce la 27500 pentru că cifra zecilor este mai mare sau egală decât
5.

a) Scriet, i un algoritm ı̂n pseudocod care cites, te două numere naturale n s, i k (n≥1,
k≥1) s, i determină numărul natural obt, inut prin rotunjirea lui n la ordinul k dacă
variabila k are valoarea mai mică decât numărul de cifre ale lui n, iar ı̂n caz contrar
afis, ează mesajul ,,Eroare”. (10p.)
b) Ment, ionat, i rolul fiecărei variabile s, i specificat, i care sunt datele de intrare s, i care sunt
datele de ies, ire ale problemei. (6p.)

SUBIECTUL al III-lea (30 de puncte)


Pentru itemul 1 scriet, i pe foaia de examen litera corespunzătoare răspunsului
corect.
1. Indicat, i expresia care poate ı̂nlocui punctele de suspensie astfel ı̂ncât, ı̂n urma executării
secvent, ei obt, inute, să se afis, eze matricea de mai jos. (4p.)
for i :=1 to 4 do 0 0 0 0
begin for j :=1 to 4 do 0 1 2 3
write (......); 0 1 3 4
writeln ;
end ; 0 2 4 6

a) i*j div 2 c) (i-1)*j div 2


b) i*(j-1) div 2 d) (i-1)*(j-1) div 2
2. În secvent, a de mai jos, toate variabilele sunt de tip ı̂ntreg. Înlocuit, i punctele de suspensie
astfel ı̂ncât, ı̂n urma executării secvent, ei obt, inute, valoarea variabilei ok să fie true dacă
există cel put, in un număr par ı̂ntre cele 10 numere citite, sau valoarea false ı̂n caz contrar.
(6p.)
ok := ......;
for i :=1 to 10 do
begin
read ( k );
......;
end ;
3. Se citesc de la tastatură valorile a două variabile ı̂ntregi, n (2<n<100) s, i k (1<k<n),
apoi un s, ir de n elemente, numere ı̂ntregi. Scriet, i un program Pascal care determină
modificarea s, irului init, ial, astfel ı̂ncât primele k elemente să fie ordonate descrescător, iar
ultimele n-k elemente să fie ordonate crescător. Elementele s, irului modificat vor fi afis, ate
pe ecran, pe o singură linie, despărt, ite prin câte un spat, iu. (10p.)
Exemplu: Pentru n = 8, k=3 s, i elementele 4 7 3 8 9 16 10 6, se va afis, a 7 4 3 6 8
9 10 16.
4. Fis, ierul bac.txt cont, ine pe prima linie valoarea naturală nenulă a variabilei n, iar pe
următoarea linie un s, ir de n numere naturale, despărt, ite prin spat, ii. Afis, at, i pe ecran
valoarea pară cea mai mare s, i numărul/numerele de ordine din s, ir al/ale valorii pare
PROBLEME DE INFORMATICĂ PENTRU EXAMENE 119

maximă, despărt, ite printr-un spat, iu. Dacă fis, ierul bac.txt nu cont, ine numere pare, se va
afis, a pe ecran mesajul Nu exista.
a) Descriet, i ı̂n limbaj natural un algoritm eficient de rezolvare a problemei. (4p.)
b) Scriet, i un program Pascal pentru rezolvarea problemei. (6p.)
Exemplu: Dacă ı̂n fis, ierul bac.txt se află numerele:
9
3 16 2 7 15 4 16 6 8

pe ecran se va afis, a 16 2 7.
120 PROBLEME DE INFORMATICĂ PENTRU EXAMENE

Teste pentru examenul de Bacalaureat, specializarea


Matematică-Informatică

Testul 1

1
Cristina Constantinescu

Limbajul C/C++
SUBIECTUL I (30 de puncte)
Pentru itemul 1, scriet, i pe foaia de examen litera corespunzătoare răspunsului
corect.
1. Indicat, i expresia care are valoarea 1 dacă s, i numai dacă valorile variabilelor x se află ı̂n
afara intervalului (-3,3). (4p.)
a) x*x - 9 <= 0 c) x*x - 9 >= 0
b) (x > -3 || x < 3) d) !(x < -3 && x > 3)
2. Se consideră algoritmul următor, reprezentat ı̂n pseudocod. S-a notat cu x%y restul
ı̂mpărt, irii lui x la y.
citeste c ( numar natural cuprins intre 1 si 9)
citeste n ( numar natural nenul )
x ← 1
y ← 0
pentru i = 1 , c executa
| x ← x * 10
|_ y ← y * 10 + 9
x ← x / 10
cat timp (x <= y si x < n ) executa
| z ← x
| i ← z % 10
| z ← z / 10
| cat timp ( z 6= 0 si z %10 = i ) executa
| |_ z ← z / 10
| daca z = 0 atunci
| |_ scrie x
|_ x ← x + 1

a) Scriet, i numărul afis, at dacă pentru variabila c se cites, te valoarea 4, iar pentru n se cites, te
valoarea 8784. (6p.)
b) Scriet, i un număr ce poate fi citit pentru c astfel ı̂ncât dacă pentru n se cites, te valoarea
300, algoritmul să nu afis, eze nimic. (4p.)
c) Scriet, i ı̂n pseudocod un algoritm, echivalent cu cel dat, ı̂n care să se ı̂nlocuiască secvent, a
ce cont, ine cele două structuri repetitive cu test init, ial cu o secvent, ă ce cont, ine o singură
structură repetitivă. (6p.)
d) Scriet, i programul C/C++ corespunzător algoritmului dat. (10p.)
1
Profesor, Colegiul Nat, ional ,,Zinca Golescu”, cristina.constantinescu70@gmail.com
PROBLEME DE INFORMATICĂ PENTRU EXAMENE 121

SUBIECTUL al II-lea (30 de puncte)


Pentru fiecare dintre itemii 1 s, i 2 scriet, i pe foaia de examen litera corespunzătoare
răspunsului corect.
1. Se consideră graful neorientat dat prin ma- 0 1 0 1 1
tricea de adiacent, ă alăturată. 1 0 0 0 1
Stabilit, i dacă se poate obt, ine un arbore 0 0 0 1 1
prin eliminarea unora dintre muchiile gra- 1 0 1 0 0
fului. (4p.)
1 1 1 0 0
a) Da, prin eliminarea exact a unei muchii. c) Da, prin eliminarea exact a trei muchii.
b) Da, prin eliminarea exact a două muchii. d) Nu.

2. Într-un graf neorientat cu 10 muchii, fiecare nod are gradul 2. Care este numărul maxim
de componente conexe din care poate fi format graful? (4p.)
a) 1 b) 3 c) 4 d) 5

Scriet, i pe foaia de examen răspunsul pentru fiecare dintre cerint, ele următoare.
3. Variabilele e1 s, i e2, declarate alăturat, memorează numele s, i data aparit, iei pentru câte o
carte. Ştiind că cele două cărt, i au apărut ı̂n ani diferit, i, scriet, i secvent, a de instruct, iuni
care afis, ează pe ecran denumirea cărt, ii apărută mai recent. (6p.)
struct data_a { int an , luna , zi ; } ;
struct carte { char nume [20]; data_a d ; } e1 , e2 ;
4. Variabila n este de tip ı̂ntreg (n par, 1≤n≤25), iar variabila a este un tablou bidimensional
cu n linii s, i n coloane, numerotate de la 1 la n, tablou care memorează elementele pozitive
cu maxim 4 cifre. Scriet, i o secvent, ă de program C/C++ care să afis, eze pe ecran suma
elementelor aflate strict deasupra celor două diagonale, diagonala principală s, i diagonala
secundară a matricei a. Se vor declara convenabil celelalte variabilele folosite ı̂n secvent, ă.
1 2 3 4
5 6 7 8
Exemplu: pentru n = 4 s, i a = se va calcula 2+3=5 s, i se va afis, a 5.
9 10 11 12
13 14 15 16
(6p.)
5. Se consideră un text cu cel mult 100 de caractere (litere mici ale alfabetului englez, cifre,
spat, ii) ı̂n care cuvintele sunt separate prin unul sau mai multe spat, ii. Un cuvânt este
format numai din litere sau numai din caractere cifre. Înaintea primului cuvânt s, i după
ultimul cuvânt nu există spat, ii.
Scriet, i un program C/C++ care cites, te de la tastatură un cuvânt s, i apoi determină s, i
afis, ează pe ecran numărul cuvintelor formate doar din caractere cifre, precum s, i textul
obt, inut din textul init, ial prin ı̂nlocuirea cuvântului 10 cu zece s, i s, tergerea celorlalte
cuvinte formate doar din caractere cifre.
Exemplu: pentru textul
Ana are de rezolvat 10 probleme la informatica 100 la matematica 23 la chimie
10 la fizica
122 PROBLEME DE INFORMATICĂ PENTRU EXAMENE

se va afis, a
4
Ana are de rezolvat zece probleme la informatica la matematica la chimie zece
la fizica (10p.)

SUBIECTUL al III-lea (30 de puncte)


Pentru itemul 1, scriet, i pe foaia de examen litera corespunzătoare răspunsului
corect.
1. Se consideră un număr natural nenul n având exact k cifre, cifrele lui fiind distincte două
câte două, iar printre cele k cifre se găses, te s, i cifra 0. Permutând cifrele lui n se obt, in
alte numere naturale. Câte dintre numerele obt, inute, inclusiv n, au exact k cifre? (4p.)
a) k!-(k-1)! b) k! c) (k-1)! d) (k+1)!

Scriet, i pe foaia de examen răspunsul pentru fiecare dintre cerint, ele următoare.
2. Se consideră subprogramul f definit void f ( int a )
alăturat, ı̂n care a este număr ı̂ntreg nenul. {
Ce se afis, ează pe ecran la apelul lui f(3), if (a >0){
dar la apelul lui f(123)? (6p.) cout < < a %10;
f ( a /10);
cout < < a %10;
}
}

3. Se consideră subprogramul divizori, cu trei parametri. Primeste prin intermediul parame-


trului n un număr natural nenul 2≤n≤10000 s, i returnează prin intermediul parametrilor
d1 s, i d2 cel mai mic, respectiv cel mai mare divizor prim al său (1<d1≤d2≤n).
Pentru n=6, se obt, in d1=2 s, i d2=3, iar pentru n=7 se obt, in d1=7 s, i d2=7.
a) Scriet, i definit, ia completă a subprogramului divizori. (6p.)
b) Scriet, i un program care cites, te două numere naturale a s, i b, de cel mult 4 cifre
fiecare, s, i determină folosind apeluri utile ale subprogramului divizori toate numerele
naturale cuprinse ı̂n intervalul [a,b] cu proprietatea că sunt numere prime. Numerele
determinate se afis, ează pe ecran cu spat, ii ı̂ntre ele.
Exemplu: Pentru a=12, b=20 se vor afis, a numerele: 13 17 19 (4p.)
4. Fis, ierul text numere.in memorează cel mult 1000000000 de numere naturale s, i cel
put, in 2, fiecare cu cel mult nouă cifre. Numerele sunt separate prin câte un spat, iu.
Se consideră că un s, ir format din cel put, in trei termeni formează un ,,platou” dacă
diferent, a dintre oricare termen al acestuia, ı̂ncepând cu cel de-al doilea, s, i cel aflat pe
pozit, ia anterioară ı̂n s, ir este constantă s, i mai mare ca zero.
Se cere să se citească numerele din fis, ierul numere.in s, i să se afis, eze pe ecran numărul
maxim de termeni ai unei secvent, e din s, ir, secvent, ă care formează un ,,platou”. Dacă nu
există astfel de secvent, ă se afis, ează pe ecran mesajul Nu exista.
Se utilizează un algoritm eficient din punctul de vedere al memoriei s, i al timpului de
executare.
Exemplu: dacă fis, ierul numere.in cont, ine numerele
PROBLEME DE INFORMATICĂ PENTRU EXAMENE 123

3 8 13 18 7 8 1 0 -4 -8 9 13 17 21 25 -6 -1 3 7 17 18
se afis, ează pe ecran valoarea 5 (corespunzătoare secvent, ei 9 13 17 21 25, care reprezintă
o secvent, ă mai lungă decât “platourile” cu elementele: 3 8 13 18 s, i -1 3 7 s, i lungimile 4,
respectiv 3).
a) Descriet, i ı̂n limbaj natural algoritmul utilizat, justificând eficient, a acestuia. (4p.)
b) Scriet, i programul C/C++ corespunzător algoritmului descris. (6p.)

Testul 2

Nicoleta Voica 2 , Adrian Voica 3

Limbajul C/C++
SUBIECTUL I (30 de puncte)
Pentru itemul 1, scriet, i pe foaia de examen litera corespunzătoare răspunsului
corect.
1. Variabilele a, b, c, d s, i x sunt de tip real. Care dintre următoarele expresii C/C++ au
valoarea 1 dacă s, i numai dacă x ∈/ [a, b] ∪ [c, d] ∪ {−1, 0, 1}? (4p.)
a) x>=a && x<=b ||x>=c && x<=d ||x==-1 ||x==0 ||x==1
b) !(x>=a && x<=b) && !(x>=c && x<=d ) && !( x==-1 ||x==0 ||x==1)
c) x<a && x>b ||x<=c && x<=d ||(x!=-1 && x!=0 && x!=1)
d) (x<a ||x>b ||x<c ||x>d ) && (x!=-1 && x!=0 && x!=1)
Scriet, i pe foaia de examen răspunsul pentru fiecare dintre cerint, ele următoare.
2. Se consideră algoritmul de mai jos, scris ı̂n pseudocod.
citeste x ( numar intreg )
m←0
cat timp x >0 executa
| d←2
| k←0
| y←x
| cat timp y6= 1 executa
| | p←0
| | cat timp y % d =0 executa
| | | y←[ y / d ]
| | |_ p←1
| | _ k←k + p ; d←d +1
| daca k≥m atunci m←k
| |_ nr←x
| _ citeste x ( numar intreg )
scrie m , nr

S-a notat cu [x] partea ı̂ntreagă a numărului real x iar cu x%y restul ı̂mpărt, irii numărului
ı̂ntreg x la numărul ı̂ntreg nenul y.
2
Profesor, Colegiul Nat, ional ,,Zinca Golescu”, Pites, ti, nvoica71@yahoo.fr
3
Profesor, Liceul Teoretic Ion Barbu”, Pites, ti, avoica71@yahoo.com

124 PROBLEME DE INFORMATICĂ PENTRU EXAMENE

a) Scriet, i ce valori vor fi afis, ate dacă se citesc pe rând valorile: 15, 13, 305, 12600,
72, 1155, 32, 56, 0. (6p.)
b) Precizat, i două seturi de date de intrare distincte astfel ı̂ncât să se afis, eze pentru m,
de fiecare dată, valoarea 1. (6p.)
c) Scriet, i ı̂n pseudocod un algoritm echivalent cu cel dat care să utilizeze doar două
structuri repetitive. (4p.)
d) Scriet, i programul C/C++ corespunzător algoritmului dat. (10p.)
SUBIECTUL al II-lea (30 de puncte)

1. Fie declarările de mai jos


struct bac1 struct bac2
{ {
float a , b ; int x , y ;
}; struct bac1 z ;
float t ;
};

şi următoarele declaraţii: bac1 y , b ;


bac2 x , a ;

Care dintre următoarele construcţii este corectă? (4p.)


a) x.b.z b) b.a.z c) a.z.a d) a.y.b
2. Se consideră o structură statică de tip coadă s, i o structură statică de tip stivă. Notăm cu
ADS(x) operat, ia prin care se adaugă informat, ia x ı̂n stivă, ADC(x) operat, ia prin care se
adaugă informat, ia x ı̂n coadă, ELS operat, ia prin care se elimină un element din stivă s, i se
adaugă ı̂n coadă, ELC operat, ia prin care se elimină un element din coadă s, i de adaugă
ı̂n stivă. Care este cont, inutul stivei s, i cont, inutul cozii ı̂n urma executării următoarelor
operat, ii?
ADS(5), ADS(8), ADC(3), ELS, ADC(7), ADS(1), ELC, ELC, ADS(2), ELS, ADC(6)
(4p.)
S : 5, 1, 7, 8 S : 5, 1, 3, 8 S : 5, 1, 3, 8; S : 5, 1, 7, 8
a) b) c) d)
C : 3, 2, 6 C : 6, 2, 7 C : 7, 2, 6 C : 6, 2, 3
3. Se consideră graful neorientat cu 10 noduri, reprezentat prin următoarea listă de muchii:
[1, 4], [1, 5], [1, 7], [2, 6], [3, 4], [3, 10], [5, 6], [7, 8], [7, 9].
Să se verifice dacă graful este arbore. În caz afirmativ să se precizeze lungimea celui
mai lung lant, elementar din arbore, iar ı̂n caz contrar să se precizeze câte muchii trebuie
eliminate sau adăugate ı̂n graf astfel ı̂ncât să devină arbore. (6p.)
4. Ştiind că x este un tablou bidimensional cu 4 linii s, i 4 coloane (numerotate de la 1 la 4)
s, i elemente de tip ı̂ntreg să se precizeze care va fi cont, inutul tabloului după executarea
secvent, ei de instruct, iuni de mai jos? (6p.)
a =1; b =0;
for ( i =4; i >=1; i - -)
for ( j =4; j >= i ; j - -)
{ x [ i ][ j ] = a + b ;
c = x [ i ][ j ];
PROBLEME DE INFORMATICĂ PENTRU EXAMENE 125

x [ j ][ i ] = c ;
a = b ; b = c ;}
5. Scriet, i un program ı̂n C/C++ care cites, te un text de maxim 250 de caractere litere ale
alfabetului englez sau spat, ii s, i determină s, i afis, ează pe ecran numărul total de cuvinte din
text, iar pe următorul rând, separate prin câte un spat, iu, cuvintele din text ı̂n care primul
s, i ultimul caracter sunt ambele vocale sau ambele consoane. (10p.)
Exemplu: pentru textul La bacalaureat la proba de Informatica a fost un su-
biect cu un sir de caractere
se va afis, a:
15
bacalaureat Informatica a fost subiect sir

SUBIECTUL al III-lea (30 de puncte)

1. Utilizând metoda backtracking, sunt generate ı̂n ordine lexicografică toate anagramele
cuvântului ecran. Să se precizeze care sunt cuvintele generate imediat ı̂nainte s, i după
secvent, a următoare: ernac, ernca, nacer . (4p.)
a) ercan, b) ercna, c) ercna, d) ercan,
nacre narce nacre narce
2. Fie următorul subprogram recursiv:
void bac ( int x , int d )
{ if (d <= x / d )
{
if ( x % d ==0) cout < <d < <" ";
bac (x , d +1);
if ( x % d ==0) cout < < x /d < <" ";
}
}

Ce va afis, a subprogramul ı̂n urma apelurilor bac(13, 1) s, i bac(56, 1)? (6p.)


3. a) Scriet, i definit, ia completă a subprogramului bac cu doi parametri, care primes, te prin
intermediul primului parametru x un număr natural nenul de minim 2 cifre s, i maxim
9 cifre nenule s, i returnează prin intermediul celui de-al doilea parametru, y, numărul
obt, inut din x prin eliminarea primei s, i a ultimei cifre. Subprogramul returnează
valoarea 1 dacă numărul x ı̂ncepe s, i se termină cu aceeas, i cifră. (6p.)
Exemplu: dacă x=12341, subprogramul returnează valoarea 1 deoarece numărul x
ı̂ncepe s, i se termină cu aceeas, i cifră, iar y=234.
b) Scriet, i un program C/C++ care cites, te de la tastatură n numere naturale nenule
(1≤n≤100) de minim 2 cifre s, i maxim 9 cifre nenule s, i folosind apeluri utile ale
subprogramului de la punctul a) determină s, i afis, ează pe ecran câte dintre cele n
numere sunt palindrom. În cazul ı̂n care nu există ı̂n s, ir numere palindrom se va
afis, a un mesaj corespunzător. (4p.)
Exemplu: pentru n=5 s, i numerele 12341, 454, 11, 89, 4567, se va afis, a 2
(numerele 454 s, i 11).
4. Fis, ierul text bac.in cont, ine cel mult 100000 de numere naturale.
a) Folosind un algoritm eficient din punct de vedere al memoriei utilizate s, i al timpului
126 PROBLEME DE INFORMATICĂ PENTRU EXAMENE

de executare scriet, i un program C/C++ care cites, te numerele din fis, ier s, i determină
primul s, i ultimul număr din s, ir, precum s, i pozit, iile acestora, care au cele mai multe
cifre egale cu 1 ı̂n scrierea lor ı̂n baza 2.
Rezultatul se va afis, a ı̂n fis, ierul text bac.out. (6p.)
b) Descriet, i ı̂n limbaj natural metoda utilizată justificând eficient, a acesteia. (4p.)
Exemplu: dacă fis, ierul bac.in cont, ine numerele 4, 5, 9, 13, 6, 11, 10, 14, 2, 1,
se va afis, a ı̂n bac.out 13, 14, pozit, iile 4, respectiv 8.

Testul 3

Nicoleta Voica 4 , Adrian Voica 5

Limbajul Pascal
SUBIECTUL I (30 de puncte)
Pentru itemul 1, scriet, i pe foaia de examen litera corespunzătoare răspunsului
corect.
1. Variabilele a, b, c, d s, i x sunt de tip real. Care dintre următoarele expresii Pascal au
valoarea 1 dacă s, i numai dacă x ∈
/ [a, b] ∪ [c, d ] ∪ {-1, 0, 1}? (4p.)
a) x >=a and x <=b or x >=c and x <=d or x ==-1 or x ==0 or x ==1
b) not(x >=a and x <=b) and not(x >=c and x <=d ) and not( x ==-1 or x ==0 or
x ==1)
c) x <a and x >b or x <=c and x <=d or (x <>-1 and x <>0 and x <>1)
d) (x <a or x >b or x <c or x >d ) and (x <>-1 and x <>0 and x<>1)
Scriet, i pe foaia de examen răspunsul pentru fiecare dintre cerint, ele următoare.
2. Se consideră algoritmul de mai jos, reprezentat ı̂n pseudocod.
citeste x ( numar intreg )
m←0
cat timp x >0 executa
| d←2
| k←0
| y←x
| cat timp y6= 1 executa
| | p←0
| | cat timp y % d =0 executa
| | | y←[ y / d ]
| | |_ p←1
| | _ k←k + p ; d←d +1
| daca k≥m atunci
| | _ m←k ; nr←x
| _ citeste x ( numar intreg )
scrie m , nr
4
Profesor, Colegiul Nat, ional ,,Zinca Golescu”, Pites, ti, nvoica71@yahoo.fr
5
Profesor, Liceul Teoretic Ion Barbu”, Pites, ti, avoica71@yahoo.com

PROBLEME DE INFORMATICĂ PENTRU EXAMENE 127

S-a notat cu [x] partea ı̂ntreagă a numărului real x iar cu x%y restul ı̂mpărt, irii numărului
ı̂ntreg x la numărul ı̂ntreg nenul y.
a) Scriet, i ce valori vor fi afis, ate dacă se citesc pe rând valorile: 15, 13, 305, 12600,
72, 1155, 32, 56, 0. (6p.)
b) Precizat, i două seturi de date de intrare distincte astfel ı̂ncât să se afis, eze pentru m,
de fiecare dată, valoarea 1. (6p.)
c) Scriet, i ı̂n pseudocod un algoritm echivalent cu cel dat care să utilizeze doar două
structuri repetitive. (4p.)
d) Scriet, i programul Pascal corespunzător algoritmului dat. (10p.)
SUBIECTUL al II-lea (30 de puncte)

1. Fie declarările de mai jos


type bac1 = record type bac2 = record
a , b : real ; x , y : integer ;
end ; z : bac1 ;
t : real ;
end ;

şi următoarele declaraţii:


var y , b : bac1 ;
x , a : bac2 ;

Care dintre următoarele construcţii este corectă? (4p.)


a) x.b.z b) b.a.z c) a.z.a d) a.y.b
2. Se consideră o structură statică de tip coadă s, i o structură statică de tip stivă. Notăm cu
ADS(x) operat, ia prin care se adaugă informat, ia x ı̂n stivă, ADC(x) operat, ia prin care se
adaugă informat, ia x ı̂n coadă, ELS operat, ia prin care se elimină un element din stivă s, i se
adaugă ı̂n coadă, ELC operat, ia prin care se elimină un element din coadă s, i de adaugă
ı̂n stivă. Care este cont, inutul stivei s, i cont, inutul cozii ı̂n urma executării următoarelor
operat, ii?
ADS(5), ADS(8), ADC(3), ELS, ADC(7), ADS(1), ELC, ELC, ADS(2), ELS, ADC(6)
(4p.)
S : 5, 1, 7, 8 S : 5, 1, 3, 8 S : 5, 1, 3, 8; S : 5, 1, 7, 8
a) b) c) d)
C : 3, 2, 6 C : 6, 2, 7 C : 7, 2, 6 C : 6, 2, 3
3. Se consideră graful neorientat cu 10 noduri, reprezentat prin următoarea listă de muchii:
[1, 4], [1, 5], [1, 7], [2, 6], [3, 4], [3, 10], [5, 6], [7, 8], [7, 9].
Să se verifice dacă graful este arbore. În caz afirmativ să se precizeze lungimea celui
mai lung lant, elementar din arbore, iar ı̂n caz contrar să se precizeze câte muchii trebuie
eliminate sau adăugate ı̂n graf astfel ı̂ncât să devină arbore. (6p.)
4. Ştiind că x este un tablou bidimensional cu 4 linii s, i 4 coloane (numerotate de la 1 la 4)
s, i elemente de tip ı̂ntreg să se precizeze care va fi cont, inutul tabloului după executarea
secvent, ei de instruct, iuni de mai jos? (6p.)
a =1; b =0;
for i :=4 downto 1 do
128 PROBLEME DE INFORMATICĂ PENTRU EXAMENE

for j :=4 downto i do


begin
x [ i ][ j ]:= a + b ;
c := x [ i ][ j ];
x [ j ][ i ]:= c ;
a := b ; b := c ;
end ;
5. Scriet, i un program ı̂n Pascal care cites, te un text de maxim 250 de caractere litere ale
alfabetului englez sau spat, ii s, i determină s, i afis, ează pe ecran numărul total de cuvinte din
text, iar pe următorul rând, separate prin câte un spat, iu, cuvintele din text ı̂n care primul
s, i ultimul caracter sunt ambele vocale sau ambele consoane. (10p.)
Exemplu: pentru textul La bacalaureat la proba de Informatica a fost un su-
biect cu un sir de caractere se va afis, a: 15 .

SUBIECTUL al III-lea (30 de puncte)

1. Utilizând metoda backtracking, sunt generate ı̂n ordine lexicografică toate anagramele
cuvântului ecran. Să se precizeze care sunt cuvintele generate imediat ı̂nainte s, i după
secvent, a următoare: ernac, ernca, nacer . (4p.)
a) ercan, b) ercna, c) ercna, d) ercan,
nacre narce nacre narce
2. Fie următorul subprogram:
procedure bac (x , d : integer )
begin
if d <= x / d then
begin
if x div d =0 then write (d , ’ ’);
bac (x , d +1);
if x div d =0 then write ( x div d , ’ ’);
end ;
end ;

Ce va afis, a subprogramul ı̂n urma apelurilor bac(13, 1) s, i bac(56, 1)? (6p.)


3. a) Scriet, i definit, ia completă a subprogramului bac cu doi parametri, care primes, te prin
intermediul primului parametru x un număr natural nenul de minim 2 cifre s, i maxim
9 cifre nenule s, i returnează prin intermediul celui de-al doilea parametru, y, numărul
obt, inut din x prin eliminarea primei s, i a ultimei cifre. Subprogramul returnează
valoarea 1 dacă numărul x ı̂ncepe s, i se termină cu aceeas, i cifră.
Exemplu: dacă x=12341, subprogramul returnează valoarea 1 deoarece numărul x
ı̂ncepe s, i se termină cu aceeas, i cifră, iar y=234. (6p.)
b) Scriet, i un program Pascal care cites, te de la tastatură n numere naturale nenule
de minim 2 cifre s, i maxim 9 cifre, 1 ≤ n ≤ 100, s, i folosind apeluri utile ale
subprogramului de la punctul a) determină s, i afis, ează pe ecran câte dintre cele n
numere sunt palindrom. În cazul ı̂n care nu există ı̂n s, ir numere palindrom se va
afis, a un mesaj corespunzător. (4p.)
Exemplu: pentru n=5 s, i numerele 12341, 454, 11, 89, 4567, se va afis, a 2
(numerele 454 s, i 11).
PROBLEME DE INFORMATICĂ PENTRU EXAMENE 129

4. Fis, ierul text bac.in cont, ine cel mult 100000 de numere naturale.
a) Folosind un algoritm eficient din punct de vedere al memoriei utilizate s, i al timpului
de executare scriet, i un program Pascal care cites, te numerele din fis, ier s, i determină
primul s, i ultimul număr din s, ir, precum s, i pozit, iile acestora, care au cele mai multe
cifre egale cu 1 ı̂n scrierea lor ı̂n baza 2. Rezultatul se va afis, a ı̂n fis, ierul text bac.out.
(6p.)
b) Descriet, i ı̂n limbaj natural metoda utilizată justificând eficient, a acesteia. (4p.)
Exemplu: dacă fis, ierul bac.in cont, ine numerele 4, 5, 9, 13, 6, 11, 10, 14, 2, 1,
se va afis, a ı̂n bac.out 13, 14, pozit, iile 4, respectiv 8.

Testul 4

6
Grat, iela Ghiordunescu

Limbajul C/C++
SUBIECTUL I (30 de puncte)
Pentru itemul 1, scriet, i pe foaia de examen litera corespunzătoare răspunsului
corect.
1. Ce afis, ează expresia 28 / 5 + 28 % 5 / 3? (4p.)
a) 6 b) 6.6 c) 7 d) 6.5

Scriet, i pe foaia de examen răspunsul pentru fiecare dintre cerint, ele următoare.
2. Se consideră algoritmul următor, descris ı̂n pseudocod. S-a notat cu x%y restul ı̂mpărt, irii
numărului ı̂ntreg x la numărul ı̂ntreg nenul y s, i cu [a] partea ı̂ntreagă a numărului real a.
citeste x ( numar natural )
y←0
z←0
p←1
cat timp x6= 0 executa
| daca x %2=0 atunci
| | y←y?10+ x %10
| altfel
| | z←p?( x %10)+ z
| | _ p←p?10;
| _ x←[ x /10]
scrie y ," " , z

a) Scriet, i valorile ce se vor afis, a ı̂n urma executării algoritmului de mai sus dacă se
cites, te x=102035. (6p.)
b) Scriet, i un număr de 3 cifre care poate fi citit pentru variabila x astfel ı̂ncât algoritmul
să afis, eze valorile 0 1. (4p.)
c) Scriet, i ı̂n pseudocod un algoritm, echivalent cu cel dat, ı̂n care să se ı̂nlocuiască
structura cat timp...executa cu o structură repetitivă de alt tip. (6p.)
6
Profesor, Colegiul Nat, ional ,,Zinca Golescu”, Pites, ti, grati 1999@yahoo.com
130 PROBLEME DE INFORMATICĂ PENTRU EXAMENE

d) Scriet, i programul C/C++ corespunzător algoritmului dat. (10p.)

SUBIECTUL al II-lea (30 de puncte)


Pentru fiecare dintre itemii 1 s, i 2 scriet, i pe foaia de examen litera corespunzătoare
răspunsului corect.
1. Matricea de adiacent, ă a unui graf neorientat cu 8 linii are exact 20 valori nenule s, i suma
elementelor de pe fiecare linie este mai mare sau egală cu 2. Care este valoarea maximă
pe care o poate avea gradul unui nod ı̂ntr-un astfel de graf? (4p.)
a) 7 b) 6 c) 5 d) 4
2. Se consideră un arbore cu rădăcină reprezentat ı̂n memorie cu ajutorul vectorului de
tat, i T=(2, 3, 0, 3, 3, 2, 6, 6, 4, 9). Stabilit, i care dintre următoarele noduri sunt
extremităt, i finale ale unor lant, uri de lungime pară care au ca extremitate init, ială rădăcina
arborelui. (4p.)
a) 2 2 4 5 b) 10 3 c) 1 6 9 d) 2 4 5

Scriet, i pe foaia de examen răspunsul pentru fiecare dintre cerint, ele următoare.
3. În declararea de mai jos, câmpurile x s, i y ale ı̂nregistrării pot memora coordonatele
carteziene ale unui punct din planul xOy. Scriet, i condit, ia care verifică dacă punctele A, B
s, i C formează un triunghi echilateral. (6p.)
struct punct { float x , y ; } A ,B , C ;
4. Scriet, i ce se afis, ează pe ecran ı̂n urma executării secvent, ei de program de mai jos, ı̂n care
variabila s memorează un s, ir de cel mult 20 caractere, iar variabila i este de tip ı̂ntreg.
(6p.)
strcpy (s ," bacalaureat ");
for ( int i =1; i < strlen ( s ); i ++)
if ( strchr (" aeiou " , s [ i ]))
strcpy ( s +i , s + i +1);
cout < < s ;
5. Se consideră o progresie aritmetică cu primul termen p s, i rat, ia r. Să se construiască o
matrice pătratică cu n linii s, i n coloane care să cont, ină termenii acestei progresii astfel:
- prima coloană va cont, ine primii n termeni, ı̂n ordine, de sus ı̂n jos
- a doua coloană va cont, ine următorii n termeni, ı̂n ordine, de sus ı̂n jos s, i as, a mai
departe.
Programul va afis, a pe ecran elementele matricei, câte o linie a matricei pe o linei a
ecranului, elementele de pe fiecare linie fiind separate prin câte un spat, iu. (10p.)
Exemplu: Pentru p=-2, r=2 s, i n=3 matricea va fi

−2 4 10
0 6 12
2 8 14
PROBLEME DE INFORMATICĂ PENTRU EXAMENE 131

SUBIECTUL al III-lea (30 de puncte)


Pentru itemul 1, scriet, i pe foaia de examen litera corespunzătoare răspunsului
corect.
1. Utilizând metoda backtracking se generează numere de 3 cifre impare, cifre care apart, in
mult, imii {9, 8, 1, 6, 3, 2} Primele 4 solut, ii sunt 999, 991, 993, 919. Care este cea de
a 9 - a solut, ie? (4p.)
a) 931 b) 939 c) 933 d) 193
2. Subprogramul sub este definit ı̂n dreapta. void sub ( int x , int nr ) {
Ce se afis, ează ca urmare a apelului if ( x * nr >0) {
sub(2024,2)? (6p.) if ( x %4==0)
cout < <x < <" ";
sub (x -4 , nr -1);
}
cout < <x < <" ";
}

3. Scriet, i definit, ia completă a unui subprogram f cu trei parametri: a un număr natural


nenul având maxim 8 cifre, n un număr natural nenul mai mic ca 10 s, i s care returnează
suma primelor n zecimale ale numărului real a−1 . (10p.)
Exemplu: Pentru f(4,3,s), s va avea valoarea 7 deoarece 4−1 = 0.25000 s, i suma primelor
3 zecimale este 2+5+0=7.
4. În fis, ierul bac.in se află cel mult un milion de numere naturale cu maxim 9 cifre separate
ı̂ntre ele prin câte un spat, iu.
a) Scriet, i ı̂n limbajul C/C++ un algoritm, eficient din punct de vedere al memoriei s, i
al timpului de execut, ie, care cites, te din fis, ier datele existente s, i determină s, i afis, ează
ı̂n fis, ierul bac.out, numarul cel mai mare din fis, ierul bac.in care este egal cu suma
numerelor vecine. În cazul ı̂n care nu există astfel de numere se afis, ează mesajul NU
EXISTA. (8p.)
b) Descriet, i succint ı̂n limbaj natural, metoda de rezolvare folosită, explicând ı̂n ce
constă eficient, a ei. (2p.)
Exemplu:
bac.in bac.out
2 2 70 3 33 30 7 17 10 33

Testul 5
7
Cătălina Enescu
Limbajul C/C++
SUBIECTUL I (30 de puncte)
Pentru itemul 1, scriet, i pe foaia de examen litera corespunzătoare răspunsului
corect.
1. Variabila x este de tip real. Care dintre următoarele expresii C/C++ are valoarea 1 dacă
s, i numai dacă numărul real memorat ı̂n variabila x nu apart, ine intervalului (1,8]? (4p.)
7
Profesor, Liceul Teoretic ,,Ion Cantacuzino”, Pites, ti, catalina.enescu@yahoo.com
132 PROBLEME DE INFORMATICĂ PENTRU EXAMENE

a) (x>1) && (x<=8) c) (x<=1) || (x>8)


b) (x<=1) && (x>8) d) (x<1) || (x>8)
2. Algoritmul de mai jos este reprezentat ı̂n pseudocod.
citeste x ( numar natural )
nr←0
cat timp x !=0 executa
| citeste y ( numar natural )
| daca x %2 = y %2 atunci
| | _ nr←nr + 1
| _ x←y
scrie nr

a) Ce se va afis, a pentru s, irul de valori 2 4 6 5 7 4 3 0? (6p.)


b) Dat, i un exemplu de s, ir de cel put, in două valori pentru care valoarea afis, ată va fi 0.
(4p.)
c) Scriet, i ı̂n pseudocod un algoritm echivalent cu cel dat, ı̂nlocuind adecvat structură
cat timp... executa cu o structură repetitivă de alt tip. (6p.)
d) Scriet, i programul C/C++ corespunzător algoritmului dat. (10p.)

SUBIECTUL al II-lea (30 de puncte)


Pentru fiecare dintre itemii 1 s, i 2 scriet, i pe foaia de examen litera corespunzătoare
răspunsului corect.
1. Un graf neorientat are 9 noduri. Numărul minim de muchii pe care trebuie să le aibă
graful astfel ı̂ncât să nu existe noduri izolate este: (4p.)
a) 5 b) 8 c) 9 d) 36
2. Cât, i frat, i are nodul 1 din arborele cu rădăcină cu 7 noduri, numerotate de la 1 la 7, având
următorul vector ,,de tat, i”: (5,1,5,1,0,7,5)? (4p.)
a) 0 b) 1 c) 2 d) 3

Scriet, i pe foaia de examen răspunsul pentru fiecare dintre cerint, ele următoare.
3. Pentru un elev, ı̂n variabila e se memorează următoarele informat, ii: nume (un s, ir de
cel mult 20 de caractere), data nas, terii (ziua, luna s, i anul) s, i media anuală. Ştiind că
expresiile C/C++ de mai jos au ca valori ziua (un număr natural din intervalul [1,31]),
luna (un număr natural din intervalul [1,12]) s, i anul nas, terii (număr natural), respectiv
media anuală (un număr real), scriet, i definit, ia unei structuri cu eticheta elev, care permite
memorarea informat, iilor pentru un elev, s, i declarat, i corespunzător variabila e.
e.nume, e.data.zi, e.data.luna, e.data.an, e.medie (6p.)
4. Scriet, i un program C/C++ care cites, te de la tastatură un s, ir de cel mult 50 de caractere
(litere mici s, i mari ale alfabetului englez, cifre s, i spat, ii) s, i afis, ează pe ecran litera mare
cel mai des ı̂ntâlnită ı̂n s, irul citit. Dacă există mai multe litere mari cu număr maxim
de aparit, ii, programul o va afis, a pe prima dintre ele ı̂n ordine alfabetică. Dacă s, irul nu
cont, ine litere mari, atunci pe ecran se va afis, a mesajul nu.
Exemplu: Dacă se cites, te s, irul
MERGEM LA MARE
PROBLEME DE INFORMATICĂ PENTRU EXAMENE 133

atunci se va afis, a: E (pentru că literele E s, i M apar de cele mai multe ori ı̂n s, ir s, i E este
prima dintre ele ı̂n ordine alfabetică). (6p.)
5. Scriet, i un program C/C++ care cites, te de la tastatură un număr natural n (n∈[3,50]) s, i
construies, te ı̂n memorie un tablou bidimensional cu n linii s, i n coloane, astfel ı̂ncât:
a) fiecare element aflat pe diagonala secundară va avea valoarea n;
b) elementele aflate strict deasupra diagonalei secundare vor fi primele n(n-1)/2 pătrate
perfecte nenule;
c) elementele aflate strict sub diagonala secundară vor avea valoarea 1.
Programul afis, ează pe ecran tabloul obt, inut, câte o linie a tabloului pe câte o linie a
ecranului, elementele fiecărei linii fiind separate prin câte un spat, iu. (10p)
1 4 9 4
16 25 4 1
Exemplu: Dacă n=4, se afis, ează pe ecran
36 4 1 1
4 1 1 1
SUBIECTUL al III-lea (30 de puncte)
Pentru itemul 1, scriet, i pe foaia de examen litera corespunzătoare răspunsului
corect.
1. Se consideră subprogramul f, declarat ı̂n int f ( int n ){
dreapta. if ( n ==0)
Valoarea f(23169) este: (4p.) return 0;
else
a) 21 if ( n %2==0)
b) 15 return n %10+ f ( n /10);
c) 5 else
return f ( n /10);
d) 8
}
Scriet, i pe foaia de examen răspunsul pentru fiecare dintre cerint, ele următoare.
2. Utilizând metoda backtracking, se generează toate posibilităt, ile de a forma echipe diferite
de câte 4 persoane din mult, imea {Ionut, , Maria, Mihai, Ana, Mihaela, Gabriel},
astfel ı̂ncât ı̂n fiecare echipă să fie exact doi băiet, i s, i două fete. Primele două solut, ii
generate sunt, ı̂n această ordine, (Ionut, , Maria, Mihai, Ana), (Ionut, , Maria, Mihai,
Mihaela,). Scriet, i, ı̂n ordinea obt, inerii, ultimele două solut, ii generate. (6p.)
3. Se consideră subprogramul perechi, cu cinci parametri: a, b, c, n numere naturale
distincte, v vector cu cel mult 100 de elemente numere naturale distincte. Subprogramul
afis, ează toate perechile (x,y) cu proprietatea că x s, i y sunt numere diferite din v, care
verifică ecuat, ia ax2 + by2 = c. Dacă nu există astfel de valori, se va afis, a Nu.
Exemplu: Dacă a=1, b=1, c=25, n=5, v=(3 18 5 0 4), atunci se vor afis, a perechile
(3,4) (4,3) (0,5) (5,0), nu neapărat ı̂n această ordine. (10p.)
4. Fis, ierul bac.in cont, ine pe prima linie două valori naturale, m s, i n (m≤100, n≤100), pe
a doua linie un s, ir de m numere ı̂ntregi, iar pe a treia linie un s, ir de n numere ı̂ntregi.
Fiecare dintre aceste s, iruri sunt ordonate strict crescător, iar elementele lor au cel mult 9
cifre fiecare. Numerele de pe acelas, i rând sunt separate prin câte un spat, iu. Se cere să se
afis, eze pe ecran câte dintre elementele celui de al doilea s, ir nu se regăsesc s, i ı̂n primul s, ir.
Exemplu: Dacă fis, ierul are cont, inutul
67
1 2 3 4 7 20
134 PROBLEME DE INFORMATICĂ PENTRU EXAMENE

3 5 7 8 9 20 24
se va afis, a valoarea 4 (numerele care respectă condit, ia sunt 5 8 9 24).
a) Descriet, i ı̂n limbaj natural algoritmul proiectat, justificând eficient, a acestuia. (2p.)
b) Scriet, i programul C/C++ corespunzător algoritmului descris. (8p.)
PROBLEME DE INFORMATICĂ PENTRU EXAMENE 135

Teste pentru admiterea la facultate

Testul 1
1
Viorel Păun

1. Fie mult, imea M = {1, 2, . . . , n} s, i A1 , A2 , . . . , Am submult, imi ale mult, imii M .


Spunem că două mult, imi Ai s, i Aj fac parte din aceeas, i familie dacă Ai s, i Aj au cel put, in
un element comun. În plus dacă Ai s, i Aj sunt din aceeas, i familie s, i Ai s, i Ak sunt din
aceeas, i familie atunci Ai , Aj s, i Ak sunt din aceeas, i familie.
Elaborat, i un program C++ pentru determinarea numărului de familii ce se formează s, i
a mult, imilor ce formează fiecare familie. Pentru implementare considerăm n ≤ 1000 s, i
m ≤ 50.
2. a) Fiind date n numere ı̂ntregi a1 , a2 , . . . , an , nu ı̂n mod necesar diferite, să se determine
printr-un program C++ o submult, ime a acestei mult, imi de numere cu proprietatea
că suma elementelor sale este divizibilă prin n.
b) Evaluat, i complexitatea s, i eficient, a algoritmului utilizat.

Testul 2
1
Viorel Păun

1. Fie mult, imea M = {1, 2, . . . , n} s, i A1 , A2 , . . . , Am submult, imi ale mult, imii M .


Elaborat, i un program C++ pentru a determina un număr minim de mult, imi

Ai1 , Ai2 , . . . , Aik ∈ {A1 , A2 , . . . , Am }

astfel ı̂ncât
Ai 1 ∪ Ai 2 ∪ . . . ∪ Ai k = A1 ∪ A2 ∪ . . . ∪ A m .
Pentru implementare considerăm n ≤ 1000 s, i m ≤ 50.
2. a) Elaborat, i un program C++ pentru calculul integralei
π
Z2
sinn x cosm xdx
0

utilizând formulele de recurent, ă dependente de n s, i m.


b) Elaborat, i un program C++ pentru calculul aproximativ al integralei

Z1
2
e−x dx.
0

Indicat, ie. Se pot utiliza, de exemplu, sumele Riemann sau Darboux.

1
Lect. univ. dr., Universitatea din Pites, ti, viop23@yahoo.com
136 PROBLEME DE INFORMATICĂ PENTRU EXAMENE

Teste grilă pentru admiterea la facultate

Testul 1
1
Cristina Tudose

1. Se consideră următoarea secvent, ă de cod:


int a = 15 , b = 023 , c = 0 x52 ;
int x = a - b + c ;
cout < < x ;

Care dintre următoarele variante este rezultatul afis, at după executarea acestei secvent, e de
cod?
a) 15 b) 78 c) 116 d) 44
2. În urma secvent, ei de cod
int a = 5 , *p , b = 2;
p = &a;
* p *= a ;
p = &b;
b *= a ;
cout < <a < <" " < <b < <" " < <* p ;

se tipăres, te:
a) 5 2 2 b) 5 25 50 c) 25 50 50 d) 25 2 50
3. Ce se va afis, a ı̂n urma executării programului următor?
int f1 ( int );
int f2 ( int );
int f1 ( int n ){
if ( n ==0) return 1;
return f1 (n -1)+ f2 (n -1);
}
int f2 ( int n ){
if ( n ==0) return 2;
return 2+ f1 (n -1) - f2 (n -1);
}
int main (){
cout < < f1 (4) < <" " < < f2 (4);
return 0;
}

a) 3 1 b) 2 7 c) 10 10 d) 10 8
4. Fie funct, ia:
void f ( int &a , int b ){
int t ;
t=a;
a=b;
b=t;
}
1
Lect. univ. dr., Universitatea din Pites, ti, cristina.tudose21@gmail.com
PROBLEME DE INFORMATICĂ PENTRU EXAMENE 137

Se consideră două variabile ı̂ntregi a = 200, b = 300. Ce valoare va avea suma a + b după
apelul f (a, b)?
a) 500 b) 501 c) 600 d) 401
5. Fie secvent, a de cod
int i , j , s =0;
for ( i =0; i < n ; i ++)
for ( j =0; j < n ; j ++){
if ( a [ i ][ j ] % 2) continue ;
s += a [ i ][ j ];
}

unde a este o matrice pătratică de numere ı̂ntregi, având dimensiunea n, cu liniile s, i


coloanele numerotate de la 0. Ce se va salva ı̂n variabila s după executarea secvent, ei?
a) Suma elementelor pare. b) Primul element par.
c) Suma elementelor impare. d) Primul element impar.
6. Se consideră următoarea secvent, ă de cod:
int ** a ;
int x [3]={2 ,7 ,3};
int y [3]={33 ,44 ,22};
a = new int *[2];
a [0]= y ;
a [1]= x ;
x [1]=55;
int s =0;
for ( int i =0; i <2; i ++)
s += a [ i ][ i ];
cout < < s ;

Care va fi valoarea variabilei s după executarea acesteia?


a) 88 b) 40 c) 46 d) 50
7. Fie v un vector sortat cu n elemente ı̂ntregi, pe pozit, ii numerotate de la 1 la n. Secvent, a
de cod de mai jos realizează căutarea binară a numărului x ı̂n vectorul v.
int p =1 , u =n , poz = -1;
while (p <= u )
{
int m =( p + u )/2;
if ( v [ m ]== x )
{
poz = m ;
break ;
}
if (x < v [ m ])
u =m -1;
else
p = m +1;
}
cout < < poz ;

Dacă v = (2, 4, 7, 10, 25, 36, 45, 78, 89, 92, 95, 98, 100), n = 13 s, i x = 2, numărul de
ı̂njumătăt, iri ale intervalului este:
138 PROBLEME DE INFORMATICĂ PENTRU EXAMENE

a) 1 b) 2 c) 3 d) 0
8. Se consideră o stivă şi o coadă init, ial vide. Se introduc pe rând ı̂n coadă primele s, ase
numere prime, ı̂n ordine descrescătoare. Se extrag apoi din coadă trei elemente s, i se
adaugă ı̂n stivă, ı̂n ordinea ı̂n care au fost extrase. Care este elementul din vârful stivei
după executarea acestor operat, ii?
a) 5 b) 13 c) 7 d) 2
9. Fie v un vector cu n elemente ı̂ntregi, pe pozit, ii numerotate de la 1 la n s, i secvent, a de
cod de mai jos care realizează ordonarea crescătoare a elementelor acestuia.
for ( i =1; i < n ; i ++)
for ( j = i +1; j <= n ; j ++)
if ( v [ i ] > v [ j ]){
aux = v [ i ]; v [ i ]= v [ j ]; v [ j ]= aux ;
}

Numărul total de comparat, ii ı̂ntre elementele vectorului este:


a) n(n + 1)/2 b) n(n − 1)/2
c) (n − 1)(n − 2)/2 d) n/2
10. Ce se afis, ează ca urmare a executării secvent, ei de cod următoare?
char a [12]=" INFORMATICA ";
for ( int i =2; i <12; i ++)
a [i -2]= a [ i ];
a [0]= ’ D ’;
a [2]= ’ G ’;
cout < < a ;

a) IORMATICA b) CADOGMATICA
c) DOGMATICA d) eroare
11. Ce calculează următoarea funct, ie?
int functie ( int a , int b ){
while ( a != b )
if (a > b )
a -= b ;
else
b -= a ;
return b ;
}

a) Cel mai mic multiplu comun al lui a s, i b. b) Cel mai mare divizor comun al lui a s, i b.
c) Modulul diferent, ei a − b. d) Cel mai mare număr dintre a s, i b.
12. Se foloses, te metoda backtracking pentru a genera submult, imile nevide ale mult, imii
{11, 12, 13, 14, 15} care cont, in numai elemente impare. Câte solut, ii s-au generat?
a) 6 b) 7 c) 8 d) 9
13. Numărul maxim de muchii ale unui graf neorientat ce are 10 noduri s, i două componente
conexe este egal cu:
a) 20 b) 100 c) 45 d) 36
14. Dacă G este un graf neorientat cu 10 noduri s, i 10 muchii atunci:
PROBLEME DE INFORMATICĂ PENTRU EXAMENE 139

a) G este conex b) G este un arbore


c) G cont, ine cicluri d) toate gradele nodurilor sunt egale cu 1.
15. Următoarele numere nu pot fi gradele nodurilor unui graf cu 6 noduri:
a) 1, 2, 2, 3, 4, 4 b) 1, 1, 2, 4, 4, 4
c) 1, 2, 2, 3, 3, 5 d) 1, 1, 3, 3, 4, 4.

Testul 2
2
Cristina Tudose

1. Ce se va tipări după executarea următoarei secvent, e de cod?


float x =2.25 e +02;
int t =2* x ;
cout < < t ;

a) 450 b) 4 c) 4.5 d) 5
2. În urma executării secvent, ei
int a = 5 , * p =& a ;
a *=* p ;
cout < <a < <" " < <* p ;

se va afis, a:
a) 5 5 b) 25 25 c) 25 5 d) 5 25
3. Fie doi vectori A s, i B ordonat, i crescător, cu m respectiv n elemente, pe pozit, ii numerotate
de la 1. Prin algoritmul de interclasare, prezentat mai jos, se obt, ine vectorul ordonat C
care cont, ine toate elementele din A s, i B.
i =1; j =1; k =1;
while (i <= m && j <= n )
{
if ( A [ i ] <= B [ j ])
{
C [ k ]= A [ i ]; i ++;
}
else
{
C [ k ]= B [ j ]; j ++;
}
k ++;
}
while (i <= m )
{
C [ k ]= A [ i ]; i ++; k ++;
}
while (j <= n )
{
C [ k ]= B [ j ]; j ++; k ++;
}
2
Lect. univ. dr., Universitatea din Pites, ti, cristina.tudose21@gmail.com
140 PROBLEME DE INFORMATICĂ PENTRU EXAMENE

Care este complexitatea algoritmului?

a) O(m + n) b) O(log2 (m + n))


c) O(log2 (m · n)) d) O(n · log2 (n))

4. Care va fi valoarea salvată ı̂n variabila s după executarea codului următor?

int ** a ,i ,j , s ;
a = new int *[3];
for ( int i =0; i <3; i ++){
a [ i ] = new int [3];
for ( j =0; j <3; j ++)
a [ i ][ j ]=2* i + j +2;
}
s =0;
for ( int i =1; i <3; i ++)
for ( int j =1; j <3; j ++)
s += a [ i ][ j ];

a) 38 b) 40 c) 26 d) 30

5. Ce se va tipări după executarea următoarei secvent, e?

int i , j ;
for ( i =1; i <=15; i ++)
for ( j =1; j <=15; j ++)
if (i > j ) break ;
cout < < i + j ;

a) 16 b) 3 c) 15 d) 17

6. Ce valoare va avea variabila intreagă y după executarea secvent, ei de cod următoare?

int x =30 , y =40;


if (( x < y ? y : x ) > 30)
y += x ;
else
y -= x ;

a) 10 b) 70 c) 40 d) 50

7. Fie funct, ia:

int f ( int n ){
if ( n ==0) return -1;
return n * f (n -1)+2;
}

Ce valoare va avea variabila intreagă x ı̂n urma apelului x = f (4)?

a) -1 b) 1 c) 58 d) 5
PROBLEME DE INFORMATICĂ PENTRU EXAMENE 141

8. Se foloses, te metoda backtracking pentru a genera toate numerele de patru cifre care au
suma cifrelor egală cu 12. Primele cinci solut, ii generate sunt: 1029, 1038, 1047, 1056, 1065.
A câta solut, ie este 1137?

a) a zecea b) a douăsprezecea
c) a unsprezecea d) nu este solut, ie
9. Ce se va afis, a ca urmare a executării următoarei secvent, e?
int i =5 , t =8;
switch ( i %4){
case 0: t -=3;
case 1: t /=4;
case 2: t +=5; break ;
case 3: t *=6;
}
i += - - t ;
cout < <t - -;
t -= i ++;

a) 5 b) 2 c) 6 d) 7

10. Se consideră o matrice pătratică de dimensiune n având liniile s, i coloanele numerotate de


la 0 la n − 1. Condit, ia ca elementul de pe linia i s, i coloana j să fie situat sub diagonala
secundară este:

a) i + j > n − 1 c) i + j < n − 1
b) i + j > n d) i + j < n
11. Ce se va afis, a ı̂n urma executării programului următor?
void func ( int a []){
for ( int i =1; i <4; i ++)
a [ i ]++;
}
int main (){
int x [5]={1 ,1 ,0 ,5 ,3};
func ( x );
cout < < x [1]+ x [4];
return 0;
}

a) 3 b) 4 c) 7 d) 5
12. În urma executării secvent, ei
unsigned int t =13 , i = 7;
t = (t > >2) + ( i ˆ5);
cout < < t ;

se tipăres, te:

a) 10 b) 5 c) 20 d) 30
142 PROBLEME DE INFORMATICĂ PENTRU EXAMENE

13. Înălt, imea minimă a unui arbore binar strict (orice nod are 0 sau 2 descendent, i) cu 2019
noduri este egală cu:
a) 11 b) 8 c) 9 d) 10
14. Numărul maxim de cicluri ale unui graf neorientat ce are 2018 noduri s, i toate gradele
nodurilor din mult, imea {0, 1, 2} este:
a) 672 b) 504 c) 1009 d) 673
15. Numărul minim de componente conexe ale unui graf neorientat ce are 10 noduri s, i 6
muchii este egal cu:
a) 2 b) 3 c) 4 d) 5
PROBLEME DE MATEMATICĂ PENTRU CONCURSURI 143

PROBLEME DE MATEMATICĂ PENTRU


CONCURSURI

Probleme propuse

Clasa a V-a

M 1. Arătat, i că oricare ar fi k număr natural, k ≥ 4, există numere de forma a1 a2 . . . ak astfel


ı̂ncât suma resturilor ı̂mpărt, irii lor la a1 , a2 , . . . , ak−1 , respectiv ak să fie 8k − 1.
Marian Haiducu, Pites, ti
M 2. Un ogar urmăres, te o vulpe care are 12 sărituri avans. Câte sărituri va face ogarul până să
ajungă vulpea s, tiind că, ı̂n timp ce ogarul face 7 sărituri, vulpea face 8 sărituri s, i că distant, a
parcursă de ogar ı̂n 5 sărituri este aceeas, i cu cea parcursă de vulpe ı̂n 6 sărituri?
Marius Antonescu, Cos, es, ti, Arges,
M 3. Figura 1 este formată din 4 pătrate ı̂n care sunt scrise numerele naturale a, b, c, d. Printr-o
operat, ie ı̂nt, elegem ı̂nlocuirea numerelor din două pătrate care au o latură comună cu succesorii
acestora. După mai multe operat, ii se obt, ine Figura 2. Ştiind că a + b + c + d = 43, aflat, i
numărul b.

Mugurel Simion, Ştefănes, ti, Arges,


M 4. Fie x, y, z numere naturale astfel ı̂ncât x are ultima cifră 5, y este un număr prim, y ≥ 5,
iar restul ı̂mpărt, irii lui z la 15 este egal cu 7. Dacă N = 3x + y + 2z, determinaţi valorile posibile
ale restului obţinut ı̂mpărţindu-l pe N la 6.
Andreea Oprea s, i Adrian Ţurcanu, Pites, ti
.
M 5. Să se determine cifra x astfel ı̂ncât 19x74 .. x3.
Iulia Cucu s, i Adrian Ţurcanu, Pites, ti

Clasa a VI-a

M 6. Pe o tablă sunt scrise numerele de la 1 la 2n − 1, n ≥ 4. Se s, terg, la ı̂ntâmplare, două


numere de pe tablă s, i se scrie suma lor. Este posibil ca, repetând procedeul, să obt, inem n − 1
numere distincte divizibile cu 5? Justificat, i răspunsul.
Marian Haiducu, Pites, ti
144 PROBLEME DE MATEMATICĂ PENTRU CONCURSURI

1 1
M 7. a) Câte fract, ii cu numărătorul 2018 sunt cuprinse ı̂ntre s, i .
11 10
1 1
b) Câte fract, ii cu numărătorul cel mult n sunt cuprinse ı̂ntre s, i , n ∈ N? ?
n+1 n
Radu Deaconescu, Pites, ti
M 8. Produsul divizorilor naturali ai numărului natural n este 2103240 . Scriind divizorii
numărului n ı̂n ordine crescătoare d1 = 1 < d2 < d3 < ... < dk = n, arătat, i că d1289 are cel put, in
11 cifre.
Florin Antohe, Galat, i
M 9. În triunghiul ABC bisectoarea unghiului A s, i mediatoarea laturii AB se intersectează
ı̂ntr-un punct M ∈ BC. Să se demonstreze că măsura unghiului dintre ele este mai mare de 30◦ .
Marius Antonescu, Cos, es, ti, Arges,
M 10. Fie p s, i q numere prime s, i S = p + q + p2 + q 2 + pq + p2 q + pq 2 + p2 q 2 .
a) Arătat, i că S este un număr compus.
b) Dacă p + q este divizibil cu 3, atunci arătat, i că S nu este pătrat perfect.
Mugurel Simion, Ştefănes, ti, Arges,

Clasa a VII-a

Ä äÄ ä
M 11. Fie a, b, c numere naturale astfel ı̂ncât n = 7a − 2b 7b − 2c (7c − 2a ) este natural.
Arătat, i că n nu este pătrat perfect.
Marian Haiducu, Pites, ti
m(^B) + m(^C)
M 12. Fie ∆ABC cu m(^A) = . Dacă BD, respectiv CE sunt ı̂nălt, imi ale
2
BC
triunghiului ABC, atunci arătat, i că DE = .
2
Corina Roxana Mărcus, anu, Pites, ti
M 13. Dacă a, b, c, d sunt numerele reale pozitive demonstrat, i că:
a4 b4 c4 d4
+ + + ≥ a + b + c + d. Când are loc egalitatea?
b2 c c2 d d2 a a2 c
Dorin Mărghidanu, Corabia
M 14. În triunghiul ABC, mediatoarea laturii [BC] s, i bisectoarea unghiului A sunt concurente
ı̂n punctul Q situat pe linia mijlocie [M N ] a triunghiului ABC, M ∈ (AB), N ∈ (AC). Arătat, i
că triunghiurile M QB s, i N QC au arii diferite.
Mihai Burdus, a, Pites, ti
1 1 1 1
M 15. a) Fie S1 = + + +...+ . Arătat, i că
1·2·3·4 2·3·4·5 3·4·5·6 101 · 102 · 103 · 104
1
S1 < .
18
b) Fie S2 = 1 · 2 · 3 · 4 + 2 · 3 · 4 · 5 + 3 · 4 · 5 · 6 + . . . + 101 · 102 · 103 · 104. Arătat, i că S2 nu
este pătrat perfect.
Radu Deaconescu, Pites, ti
PROBLEME DE MATEMATICĂ PENTRU CONCURSURI 145

Clasa a VIII-a

M 16. Fie a, b ∈ R? . Să se determine funct, ia f : R∗ → R, pentru care


Ç Ç å å
a Å ax ã 1 b 1 bx
· f (x) − ≤ ≤ · f − ,
b b x a x a

oricare ar fi x ∈ R∗ .

Dorin Mărghidanu, Corabia

M 17. Fie a, b, c numere reale pozitive cu abc = 1. Arătat, i că


Ç å
4 4 4 2 2 1 1 1
2
a(b + c) + b(a + c) + c(a + b) ≥ 12(ab + ac + bc) + 4(a + b + c ) ≥ 16 + + .
a b c

Mihai Florea Dumitrescu, Potcoava

M 18. Fie ABCD tetraedru ı̂n care m(^BAC)=m(^CBD)=90◦ . Dacă E este mijlocul

segmentului [CD], demonstrat, i inegalitatea 3 · CD > PABE , unde PABE reprezintă perimetrul
triunghiului ABE.

Florin Stănescu, Găes, ti

M 19. Fie funct, ia f : (0, ∞) → R, definită prin f (x) = ”partea ı̂ntreagă a volumului piramidei

triunghiulare regulate cu latura bazei de lungime x s, i ı̂nălt, imea de lungime 3”.

Să se calculeze partea ı̂ntreagă a numărului

1 1 1 1
A= + + + ... +
f (2) f (3) f (4) f (2018)

Mihai Burdus, a s, i Adrian Ţurcanu, Pites, ti

M 20. Fie V A1 A2 . . . An piramidă cu baza poligonul A1 A2 . . . An , n ∈ N, n ≥ 3, situat ı̂ntr-un


plan α. Pentru p ∈ {2, 3, . . . , n − 1} arbitrar fixat, fie G1 centrul de greutate al triunghiului
V Ap−1 Ap s, i G2 centrul de greutate al triunghiului V Ap Ap+1 . Fie P ∈ (V Ap−1 ) s, i R ∈ (V Ap+1 ).
Arătat, i că P Rkα dacă şi numai dacă P G1 , V Ap şi RG2 sunt concurente.

Marian Haiducu, Pites, ti


146 PROBLEME DE MATEMATICĂ PENTRU CONCURSURI

Clasa a IX-a

M 21. Demonstrat, i că pentru orice n ∈ N∗ s, i x ≥ 0 are loc inegalitatea


n
(2 + x)k
≥ 2n − 1.
X

k=1 2 + kx

Când are loc egalitatea?

Dorin Mărghidanu, Corabia


M 22. Demonstrat, i identitatea
" 2n−1 #
n−1
ñ ô
X n» o
k(k + 1) = , ∀ n ∈ N∗ ,
k=n 2

unde [x] s, i {x} reprezintă partea ı̂ntreagă, respectiv partea fract, ionară a numărului real x.

Nicolae Stăniloiu, Bocs, a

M 23. Rezolvat, i ı̂n (0, ∞)6 sistemul






 a1 + a2 + a3 − a4 − a5 − a6 = 3

 a21 + a22 + a23 + a24 + a25 + a26 = 9 .


a1 a2 a3 a4 a5 a6 = 1

Leonard Giugiuc s, i Diana Trăilescu, Drobeta Turnu Severin

M 24. Rezolvat, i ı̂n mult, imea numerelor reale ecuat, iile:

a) (4 cos2 x − 3)(4 cos2 3x − 3) = 2 sin x;


cos3 x + cos3 5x + cos3 9x + cos3 13x 3
b) = .
cos x + cos 5x + cos 9x + cos 13x 4

Marin Chirciu, Pites, ti

M 25. Fie ABC un triunghi având toate unghiurile mai mici decât 2π si fie T punctul Torricelli-
3 ,
Fermat al acestuia. Bisectoarele unghiurilor ^BT C, ^CT A s, i ^AT B intersectează laturile [BC],
[CA] s, i [AB] ı̂n punctele D, E s, i respectiv F . Arătat, i că AB + BC + CA ≥ 2(DE + EF + F D).

Leonard Giugiuc, România, Kadir Altintas, Turcia


s, i Miguel Ochoa Sanchez, Peru
PROBLEME DE MATEMATICĂ PENTRU CONCURSURI 147

Clasa a X-a

M 26. a) Determinat, i funct, ia strict crescătoare f : R → R care satisface condit, ia

2f (x) + f (f (x)) = 3x, ∀ x ∈ R.

b) Rezolvat, i ı̂n mult, imea numerelor reale ecuat, ia


Ä ä
3 xlog5 6 − 1 = 2x + (x + 1)log6 5 .

Marin Chirciu, Pites, ti

M 27. Rezolvat, i ı̂n R × R sistemul


√ √


 x− 3 y + 2x = y − 3
x + 2y
.

 22x − (9 − y) · 2x + 8 + 6x − 2y 2 = 0

Sorin Ulmeanu, Pites, ti


s  

… q »
M 28. Fie rn = n (n − 1) (n − 2) . . . 3 2 1, unde n ∈ N∗ . Arătat, i că:
å1− 1
(n − 1)2n + 1 2n
Ç
a) rn ≤ .
2n − 1
n √
b) ∈ [1, n ].
rn
Dorin Mărghidanu, Corabia

M 29. Fie ABC un triunghi cu AB 6= AC s, i fie D mijlocul laturii BC. În exteriorul triun-
ghiului ABC se construiesc triunghiurile BAM s, i CAN astfel ı̂ncât AM = AB, AN = AC s, i
m(^BAM ) = m(^CAN ) = α. Demonstrat, i că AD ⊥ M N dacă s, i numai dacă α = 90◦ .

Nicolae Stăniloiu, Bocs, a

M 30. Fie ABC un triunghi nedreptunghic, O centrul cercului circumscris acestuia, iar D, E
s, i F mijloacele laturilor [BC], [AC] s, i respectiv [AB]. Fie DO ∩ AC = {X}, EO ∩ AB = {Y }
s, i F O ∩ BC = {Z}. Arătat, i că

AX BY CZ (a2 − b2 )(b2 − c2 )(c2 − a2 )



· · =
a2 b 2 c 2

XC Y A ZB

(notat, iile fiind cele obis, nuite).

Van Khea, Cambodgia s, i Leonard Giugiuc, România


148 PROBLEME DE MATEMATICĂ PENTRU CONCURSURI

Clasa a XI-a

M 31. Fie A, B ∈ Mn (C) astfel ı̂ncât A = 2AB − BA. Demonstrat, i că

det(AB − BA) = 0.

(În legătură cu problema XI.459, R.M.T. nr. 2/2017, punctul a).)

Daniel Jinga, Pites, ti

M 32. Arătat, i că pentru orice matrice A, B ∈ M2 (R) au loc următoarele inegalităt, i:

a) det(A2 + B 2 − BA) ≥ det(AB − BA);

b) det ((A − B)(A + B) − 2(A2 + B 2 )) ≥ 4 det(AB − BA).

Florin Stănescu, Găes, ti

M 33. Rezolvat, i ı̂n mult, imea numerelor reale ecuat, ia

1 + 3 · 23x−2 · 15x−1 = 83x−2 − 153x−3 .

Marin Chirciu, Pites, ti

M 34. a) Fie f : [a, b] → R o funct, ie derivabilă, a, b ∈ R, a < b. Arătat, i că există c1 , c2 ∈ (a, b)
f (c1 ) − f (a) f (b) − f (c2 )
astfel ı̂ncât f 0 (c1 ) = s, i f 0 (c2 ) = .
b − c1 c2 − a
b) Demonstrat, i că pentru orice a, b ∈ R cu a < b există o infinitate de funct, ii f derivabile
pe [a, b] pentru care valorile c1 s, i c2 definite la punctul a) sunt unice s, i egale.

Dorin Mărghidanu, Corabia

M 35. Fie a, b, c, d ∈ [−1, ∞) astfel ı̂ncât a + b + c + d = 0. Arătat, i că

a2 + b2 + c2 + d2 + 5(abc + abd + acd + bcd) ≥ 4abcd.

Când are loc egalitatea?

Leonard Giugiuc s, i Diana Trăilescu, Drobeta Turnu Severin


PROBLEME DE MATEMATICĂ PENTRU CONCURSURI 149

Clasa a XII-a

M 36. Pentru orice grup G s, i orice număr natural nenul n notăm

Sn (G) = {a ∈ G | axn a = x, ∀ x ∈ G}.

a) Pentru n ≥ 2, demonstrat, i că dacă Sn (G) 6= Ø, atunci există m ∈ N∗ astfel ı̂ncât


xm = e, ∀ x ∈ G (e reprezintă elementul neutru al lui G). Rămâne adevărată afirmat, ia pentru
n = 1?
b) Demonstrat, i echivalent, a: există un grup G astfel ı̂ncât Sn (G) \ {e} 6= Ø dacă s, i numai
dacă n este impar.

Marin Chirciu, Pites, ti

M 37. Fie s un număr real apart, inând intervalului (12, 18) s, i ABC un triunghi astfel ı̂ncât

a + b + c = 6, a2 + b2 + c2 = s s, i R + r = 3 (a = BC, b = AC, c = AB, iar R s, i r sunt
raza cercului circumscris, respectiv raza cercului ı̂nscris triunghiului ABC). Exprimat, i aria
triunghiului ABC ı̂n funct, ie de s.

Leonard Giugiuc, Cristinel Mortici, România s, i Kadir Altintas, Turcia


Z b
1
M 38. Fie a, b ∈ R, a < b. Calculat, i integrala I = dx.
a (x − a)4 + (x − b)4

Daniel Jinga, Pites, ti


î ó
M 39. Determinat, i funct, iile continue f, g : 0, π2 → R care verifică simultan relat, iile
ï
πò
f 3 (x) − 3f (x)g 2 (x) = cos x, g 3 (x) − 3f 2 (x)g(x) = − sin x, ∀x ∈ 0,
2
s, i pentru care aria suprafet, ei plane cuprinse ı̂ntre graficele lor este:
a) maximă; b) minimă.

Dorin Mărghidanu, Corabia

M 40. Fie f : [0, 1] → R o funct, ie derivabilă astfel ı̂ncât f (0) = 0, f 0 (0) = 1, 0 < f 0 (x) < 1
pentru orice x ∈ (0, 1] s, i există f 00 (0) ∈ R∗ . Fie s, irul (xn )n≥0 astfel ı̂ncât x0 ∈ (0, 1] s, i
xn + f (xn )
xn+1 = , pentru orice n ≥ 0. Demonstrat, i că 0 < xn+1 < xn ≤ 1 pentru orice n ∈ N
2
s, i arătat, i că
å2
xn
Ç Z å Ç
3 4
lim n f (x) dx = .
n→∞ xn+1 f 00 (0)

Florin Stănescu, Găes, ti


150 PROBLEME DE INFORMATICĂ PENTRU CONCURSURI

PROBLEME DE INFORMATICĂ PENTRU


CONCURSURI

Probleme propuse

Clasa a IX-a

I 1 (becuatii). Tică s, i Ionică sunt buni prieteni, pasionat, i de două lucruri: matematică s, i
informatică. Aces, tia participă la un concurs de propunători de probleme pe echipe. Pentru a
avea succes Tică vrea să propună o problemă cu ecuat, ii, iar Ionică o problemă cu mult, imi. După
consultări repetate se decid să participe la concurs cu o problemă care foloses, te atât ecuat, ii cât s, i
mult, imi. În această problemă se dau N ecuat, ii de forma x2 = Ai + y 2 , 1 ≤ i ≤ N , s, i se notează
cu Si mult, imea solut, iilor celei de-a i-a ecuat, ie, Si = {(x, y)|x2 = Ai + y 2 , x, y numere naturale}.
Cerint, a problemei este determinarea a două numere B = card(S1 ) + card(S2 ) + . . . + card(Sn ),
respectiv C = card (S1 ∪ S2 ∪ . . . ∪ Sn ).
Cerint, ă
Se cunosc N s, i numerele A1 , A2 , . . . , AN s, i se cere să se determine numerele B s, i C.
Date de intrare
Din fis, ierul becuatii.in se citesc: numărul natural p de pe prima linie (ce poate avea două
valori: 1 sau 2, corespunzătoare uneia din cele două cerint, e, adică 1 pentru determinarea lui B,
iar 2 pentru determinarea lui C), numărul N de pe a doua linie s, i de pe a treia linie numerele
A1 , A2 , . . . , AN cu câte un spat, iu ı̂ntre ele.
Date de ies, ire
În fis, ierul becuatii.out se va scrie numărul B, dacă p = 1 s, i respectiv numărul C, dacă p = 2,
cu semnificat, ia din enunt, .
Restrict, ii s, i precizări
• 1 < N ≤ 10000, număr natural
• Ai sunt numere naturale nenule mai mici strict decât 1000, 1 ≤ i ≤ N
• Pentru o mult, ime M , card(M ) reprezintă numărul de elemente al mult, imii M
• Pentru mult, imile M s, i P , notăm cu M ∪ P reuniunea dintre M s, i P
Exemple
becuatii.in becuatii.out Explicat, ie
1 3 p = 1 s, i deci afis, ăm B. Prima ecuat, ie
2 este x2 = 1 + y 2 , iar a doua ecuat, ie este
1 21 x2 = 21 + y 2 . Mult, imile solut, iilor sunt
S1 = {(1, 0)} s, i S2 = {(11, 10), (5, 2)}. În
acest caz B = 1 + 2 = 3.
PROBLEME DE INFORMATICĂ PENTRU CONCURSURI 151

becuatii.in becuatii.out Explicat, ie


2 3 p = 2 s, i deci afis, ăm C. Prima ecuat, ie
2 este x2 = 1 + y 2 , iar a doua ecuat, ie este
1 21 x2 = 21 + y 2 . Mult, imile solut, iilor sunt
S1 = {(1, 0)} s, i S2 = {(11, 10), (5, 2)}, deci
S1 ∪ S2 = {(1, 0), (11, 10), (5, 2)}, deci C =
card(S1 ∪ S2 ) = 3.
Timp maxim de execut, ie: 0.04 secunde/test.
Memorie totală disponibilă 4 MB.
Doru Anastasiu Popescu, Pites, ti (Micul Gates, 2017)
I 2 (castel). În pivnit, a unui castel se află un cufăr ce cont, ine obiecte de valoare foarte mare.
Toate us, ile până la pivnit, ă se deschid dacă se introduce un cod numeric format din cifre zecimale,
cod care este scris pe us, ă pentru a facilita deplasarea mai us, oară prin castel. Pentru a obt, ine
aceste obiecte trebuie să se treacă de două obstacole:
1. us, a de la intrarea ı̂n pivnit, ă, care se deschide doar dacă se introduce un cod format din
cifre zecimale, notat cu codP ;
2. capacul cufărului, care se deschide dacă se cunoas, te un cod format din cifre zecimale,
notat cu codC.
Codurile de deschidere a celor N us, i ale castelului, prin care se trece pentru a ajunge ı̂n fat, a
pivnit, ei, se cunosc: x1 , x2 , . . . , xN , numere naturale.
Codul codP , necesar deschiderii pivnit, ei, se obt, ine determinând numărul de zerouri de la
sfârs, itul numărului M ax!, unde M ax este cel mai mare dintre codurile celor N us, i, x1 , x2 , . . . , xN .
Codul codC, necesar deschiderii cufărului, se obt, ine determinând numărul de divizori ai
produsului factorialelor codurilor de pe cele N us, i ale castelului: x1 ! · x2 ! · . . . · xN !.
Cerint, ă
Cunoscând numărul de us, i, N , s, i codurile lor x1 , x2 , . . . , xN , se cere:
1. codul codP de deschidere a us, ii pivnit, ei;
2. codul codC de deschidere a cufărului.
Date de intrare
Fis, ierul de intrare castel.in cont, ine pe prima linie un număr natural p. Pentru toate testele
de intrare, numărul p poate avea doar valoarea 1 sau 2.
Pe linia a doua se află N , iar pe linia a treia se află numerele x1 , x2 , . . . , xN , separate prin
câte un spat, iu, reprezentând codurile de deschidere a celor N us, i din castel.
Date de ies, ire
Dacă valoarea lui p este 1, se va rezolva numai punctul 1) din cerint, ă.
În acest caz, ı̂n fis, ierul de ies, ire castel.out se va scrie un singur număr natural reprezentând
codul codP de deschidere a us, ii pivnit, ei.
152 PROBLEME DE INFORMATICĂ PENTRU CONCURSURI

Dacă valoarea lui p este 2, se va rezolva numai punctul 2) din cerint, ă.
În acest caz, ı̂n fis, ierul de ies, ire castel.out se va scrie un singur număr natural, reprezentând
codul de deschidere a cufărului modulo 9973, adică numărul codC % 9973.
Restrict, ii s, i precizări

• 1 ≤ N ≤ 10000
• 1 ≤ x1 , x2 , . . . , xN ≤ 10000
• a%b reprezintă restul ı̂mpărt, irii lui a la b
• k! este notat, ia pentru produsul 1 · 2 · 3 · . . . · k
• Pentru determinarea corectă a codului pivnit, ei, codP , se acordă 30% din punctaj

Exemple
castel.in castel.out Explicat, ie
1 1 p=1
5 codP este numărul de zerouri ı̂n care se
36462 termină numărul 6! = 720, adică 1.
castel.in castel.out Explicat, ie
2 294 p=2
5 codC este numărul de divizori ai produsului
36462 3! · 6! · 4! · 6! · 2!, adică 294.
Timp maxim de execut, ie: 1 secundă/test.
Memorie totală disponibilă 4 MB, din care 2 MB pentru stivă.
Violeta Bălan, Slatina

I 3 (relief ). Vladino a ı̂nceput să studieze foarte mult formele de relief. Ca să nu lase informatica
pe planul doi, profesorul său ı̂i propune să rezolve o problemă cu trei forme de relief: munt, i,
dealuri s, i văi, pentru un s, ir cu n numere naturale x1 , x2 , . . . , xn .
Muntele se defines, te ca fiind o secvent, ă maximală de numere din s, irul dat de forma
xi , . . . , xj , . . . , xk , unde xi < xi+1 < . . . . < xj , xj > xj+1 > . . . > xk , i < j < k.
Dealul se defines, te ca fiind o secvent, ă maximală de numere din s, irul dat de forma
xi , . . . , xj , . . . , xk , . . . , xh , unde xi < xi+1 < . . . < xj , xj = xj+1 = . . . = xk , xk > xk+1 >
. . . > xh , i < j < k < h.
Valea se defines, te ca fiind o secvent, ă maximală de numere din s, irul dat de forma xi , . . . , xj ,
unde xi = xi+1 = . . . = xj , i < j.
Cerint, ă
Cunoscând numărul n s, i cele n numere x1 , x2 , . . . , xn , se cere:
1. de câte ori apare cel mai mic număr ı̂n s, irul x;
2. numerele de munt, i, dealuri s, i văi.
PROBLEME DE INFORMATICĂ PENTRU CONCURSURI 153

Date de intrare
Fis, ierul de intrare relief.in cont, ine pe prima linie un număr natural p. Pentru toate testele
de intrare, numărul p poate avea doar valoarea 1 sau 2.

Pe linia a doua se află n, iar pe următoarea linie numerele x1 , x2 , . . . , xn separate ı̂ntre ele
prin câte un spat, iu.

Date de ies, ire


Dacă valoarea lui p este 1, se va rezolva numai punctul 1) din cerint, ă.

În acest caz, ı̂n fis, ierul de ies, ire relief.out se va scrie un singur număr natural reprezentând
de câte ori apare cel mai mic număr ı̂n s, irul x.

Dacă valoarea lui p este 2, se va rezolva numai punctul 2) din cerint, ă.

În acest caz, ı̂n fis, ierul de ies, ire relief.out se vor scrie trei numere separate prin câte un
spat, iu, reprezentând numerele de munt, i, dealuri s, i văi.

Restrict, ii s, i precizări

• 1 ≤ n ≤ 100000
• Numerele x1 , x2 , . . . , xn sunt ≤ 1010
• Pentru rezolvarea corectă a cerint, ei 1 se acordă 20% din punctaj

Exemple
relief.in relief.out Explicat, ie
1 3 p=1
14 Cel mai mic număr ı̂n s, ir este 4, care apare
4 5 6 5 5 5 7 8 11 11 10 6 4 4 de 3 ori.
relief.in relief.out Explicat, ie
2 113 p=2
14 Avem muntele:
4 5 6 5 5 5 7 8 11 11 10 6 4 4 4565
Avem dealul:
5 7 8 11 11 10 6 4
Avem văile:
555
11 11
4 4.
Timp maxim de execut, ie: 1 secundă/test.

Memorie totală disponibilă 4 MB, din care 2 MB pentru stivă.


Doru Anastasiu Popescu, Pites, ti (Info-Oltenia, 2016)
I 4 (prime). Lui Vladino ı̂i plac mult jocurile cu piese ce se as, ează pe table de dimensiune
m × n, precum cele de s, ah. De această dată Vladino are tot atâtea piese câte pătrăt, ele are
154 PROBLEME DE INFORMATICĂ PENTRU CONCURSURI

tabla de s, ah, adică m · n, pe ele aflându-se numerele 1, 2, . . . , m · n. Toate piesele sunt as, ezate
pe tablă ı̂n ordine crescătoare pe linii de la stânga la dreapta, ca mai jos:
1 2 ... n
n+1 n+2 ... 2n
...
(m − 1)n (m − 1)n + 1 . . . m · n
Robertino vine ı̂n vizită la Vladino s, i ı̂l pune pe acesta din urmă să t, ină ochii ı̂nchis, i, timp ı̂n
care ia de pe tabla de s, ah trei piese, apoi ı̂i propune să scrie un program care să determine
numerele de pe piesele lipsă, cunoscând suma S a numerelor rămase pe tablă s, i faptul că acestea
sunt numere prime. Dacă există mai multe solut, ii, se va alege cea care dă numărul cel mai mic
prin alăturarea celor trei piese.
Cerint, ă
Cunoscând numerele m, n s, i S se cere:
1. numerele de pe piesele luate de Robertino;
2. pozit, ia ı̂n tablou (linie coloană) a celui mai mare număr de pe piesele luate de Robertino.
Date de intrare
Fis, ierul de intrare prime.in cont, ine pe prima linie un număr natural p. Pentru toate testele de
intrare, numărul p poate avea doar valoarea 1 sau 2.
Pe linia a doua se află m, n si S, separate ı̂ntre ele prin câte un spat, iu.
Date de ies, ire
Dacă valoarea lui p este 1, se va rezolva numai punctul 1) din cerint, ă.
În acest caz, ı̂n fis, ierul de ies, ire prime.out se vor scrie cele trei numere de pe piesele luate
de Robertino, ı̂n ordine crescătoare.
Dacă valoarea lui p este 2, se va rezolva numai punctul 2) din cerint, ă.
În acest caz, ı̂n fis, ierul de ies, ire prime.out se va scrie pozit, ia ı̂n tablou (linie coloană) a
celui mai mare număr de pe piesele luate de Robertino.
Restrict, ii s, i precizări
• 1 ≤ m, n ≤ 1000
• 1 ≤ S ≤ 1010
Exemple
prime.in prime.out Explicat, ie
1 2 7 11 p=1
3 4 58 În tabloul de mai jos numerele de pe piesele
luate de Robertino sunt scrise ı̂ngros, at:
1234
5678
9 10 11 12
PROBLEME DE INFORMATICĂ PENTRU CONCURSURI 155

prime.in prime.out Explicat, ie


2 33 p=2
3 4 58 În tabloul de mai jos numerele de pe piesele
luate de Robertino sunt scrise ı̂ngros, at:
1234
5678
9 10 11 12
Cel mai mare număr de pe o piesă luată de
Robertino este 11, aflat pe linia 3, coloana
3.

Timp maxim de execut, ie: 1 secundă/test.

Memorie totală disponibilă 4 MB, din care 2 MB pentru stivă.

Doru Anastasiu Popescu, Pites, ti (Info-Oltenia, 2016)

I 5 (elevi). O s, coală nu are bibliotecă s, i de aceea elevii hotărăsc să doneze cărt, i pentru a o
ı̂nfiint, a. O mare parte dintre elevii s, colii donează cărt, i. Pentru ca să existe o evident, ă a cărt, ilor
din bibliotecă se ia decizia ca fiecare elev care donează cărt, i să ı̂s, i aleagă un număr natural s, i să
ı̂l folosească la scrierea codurilor pe cărt, ile lui.

Codurile de pe cărt, i respectă următoarele reguli:

1. Codurile scrise de un elev pe cărt, ile donate de el sunt toate cifrele numărului ales, eventual
ı̂n altă ordine.
2. Două cărt, i donate de acelas, i elev au coduri diferite.
3. Numerele alese de orice doi elevi au cel put, in o cifră diferită sau o cifră care apare de un
număr diferit de ori.
4. Codurile scrise pe cărt, i nu ı̂ncep cu cifra 0.

După ce s-a terminat operat, ia de donat, ie s-au obt, inut n cărt, i cu codurile x1 , x2 , . . . , xn . Se
dores, te să se cunoască numărul de cifre folosite la scrierea codurilor pe toate cărt, ile s, i numărul
de elevi care au donat cărt, i.

Cerint, ă
Cunoscând numărul de cărt, i n s, i codurile lor x1 , x2 , . . . , xn , se cere:

1. numărul de cifre folosite la scrierea codurilor pe toate cărt, ile;

2. numărul de elevi care au donat cărt, i.

Date de intrare
Fis, ierul de intrare elevi.in cont, ine pe prima linie un număr natural p. Pentru toate testele de
intrare, numărul p poate avea doar valoarea 1 sau 2.

Pe linia a doua se află n, iar pe linia a treia se află codurile x1 , x2 , . . . , xn , separate prin câte
un spat, iu.
156 PROBLEME DE INFORMATICĂ PENTRU CONCURSURI

Date de ies, ire


Dacă valoarea lui p este 1, se va rezolva numai punctul 1) din cerint, ă.

În acest caz, ı̂n fis, ierul de ies, ire elevi.out se va scrie un singur număr natural, reprezentând
numărul de cifre folosite la scrierea codurilor pe toate cărt, ile.

Dacă valoarea lui p este 2, se va rezolva numai punctul 2) din cerint, ă.

În acest caz, ı̂n fis, ierul de ies, ire elevi.out se va scrie un singur număr natural, reprezentând
numărul de elevi care au donat cărt, i.

Restrict, ii s, i precizări

• 1 ≤ n ≤ 10000
• x1 , x2 , . . . , xn sunt numere naturale cu maxim 9 cifre
• Pentru rezolvarea corectă a primei cerint, e se acordă 20 de puncte, iar pentru cerint, a a
doua se acordă 80 de puncte

Exemple
elevi.in elevi.out Explicat, ie
1 16 p=1
5 Numerele au ı̂n ordine 3, 4, 2, 3, 4
121 1204 77 112 4201 cifre, ı̂n total 16 cifre.
elevi.in elevi.out Explicat, ie
2 3 p=2
5 Au donat cărt, i trei elevi. Primul
121 1204 77 112 4201 elev a donat cărt, ile cu codurile 121,
112, al doilea cărt, ile cu codurile
1204, 4201, iar al treilea o carte cu
codul 77.
Timp maxim de execut, ie: 1 secundă/test.

Memorie totală disponibilă 4 MB, din care 2 MB pentru stivă.


Violeta Bălan, Slatina

Clasa a X-a

I 6 (Scufit, a Ros, ie). Scufit, a Ros, ie pleacă de acasă prin pădure s, i trebuie să ajungă la bu-
nica.Traseul prin pădure este plin de obstacole. Ea trebuie să ajungă pe cel mai scurt traseu la
bunica. Scufit, a Ros, ie s, i-a făcut un plan s, i pentru a evita obstacolele a desenat harta pădurii
pe un caiet de matematică ı̂n care a colorat, ı̂n pătrăt, elele caietului, obstacolele cu ros, u iar pe
celelalte pătrăt, ele le-a lăsat libere (necolorate). A observat că harta are formă de pătrat. Pe
PROBLEME DE INFORMATICĂ PENTRU CONCURSURI 157

hartă se cunosc pozit, iile Scufit, ei s, i bunicii. Deplasările sunt permise doar pe orizontală s, i pe
verticală.
Ajutat, i-o pe Scufit, ă să ajungă pe traseul cel mai scurt de la locul ei la cel al bunicii, ocolind
obstacolele. Harta are latura de n pătrăt, ele, având aspectul unei matrice pătratice cu n linii
s, i n coloane. Liniile s, i respectiv coloanele sunt numerotate de la 1 la n. Elementele matricei
corespund zonelor pătrăt, elelor din foaia de caiet. O astfel de zonă poate fi colorată cu ros, u sau
cu alb. Scufit, a trebuie să treacă printr-un număr minim de zone libere.
Cerint, ă
Scriet, i un program care să determine numărul minim de zone libere pentru a alcătui traseul de
la Scufit, ă la bunică.
Date de intrare
Fis, ierul de intrare scufita.in cont, ine pe prima linie două valori naturale n s, i m separate
printr-un spat, iu, reprezentând dimensiunea hărt, ii, respectiv numărul de obstacole aflate pe hartă.
Fiecare dintre următoarele m linii cont, ine câte două numere naturale x s, i y separate printr-un
spat, iu, reprezentând pozit, iile obstacolelor de pe hartă (x reprezintă linia, iar y reprezintă
coloana zonei ı̂n care se află obstacolul). Ultima linie a fis, ierului cont, ine patru numere naturale
x1 , y1 , x2 , y2 , separate prin câte un spat, iu, x1 , y1 reprezintă linia respectiv coloana pozit, iei
Scufit, ei, iar x2 , y2 reprezintă linia respectiv coloana pozit, iei bunicii.
Date de ies, ire
Fis, ierul de ies, ire scufita.out va cont, ine o singură linie pe care va fi scris un număr natural
care reprezintă numărul minim de zone prin care a trecut Scufit, a.
Restrict, ii s, i precizări
• 1 ≤ n ≤ 175
• 1 ≤ m < n2
• Traseul ı̂ncepe cu zona unde se găses, te Scufit, a s, i se termină cu zona unde se găses, te bunica
• Pentru datele de test există ı̂ntotdeauna solut, ie
Exemplu
scufita.in scufita.out Explicat, ie
85 15 O modalitate de a constitui traseul este
23 parcurgerea primei linii, apoi a ultimei
36 coloane.
44
73
75
1188
Timp maxim de execut, ie: 1 secundă/test.
Memorie totală disponibilă 5 MB.
Grat, iela Ghiordunescu, Pites, ti (Dan Barbilian, 2018)
158 PROBLEME DE INFORMATICĂ PENTRU CONCURSURI

I 7 (parola). Un informatician dores, te să-l stimuleze pe fiul său să folosească reguli matematice
pentru a determina o parolă. Parola este formată din două numere ce sunt obt, inute din calcule.
Scopul informaticianului este ca fiul său să-s, i ı̂nsus, ească fract, iile zecimale neperiodice, periodice
simple, respectiv periodice mixte s, i să transforme o fract, ie zecimală ı̂n fract, ie ordinară. Fiul său
primes, te de la tată fract, ia zecimală, iar el pentru a sparge parola trebuie să determine două
numere ce reprezintă numărătorul respectiv numitorul fract, iei ordinare echivalente, cu număr
minim de cifre la numitor, cu numitorul fiind o cifră de 1 urmată eventual de zero-uri sau o
cifră de 9 sau mai multe, urmate eventual de zero-uri.
123 − 12 111 12500 125
De exemplu, 1,2(3) = = , iar 12,500 = = .
90 90 1000 10
Cerint, ă
Scriet, i un program care cites, te o fract, ie zecimală strict pozitivă s, i afis, ează numărătorul s, i
numitorul unei fract, ii ordinare echivalente constuită conform regulii de mai sus.
Date de intrare
Fis, ierul parola.in cont, ine pe o singură linie un s, ir de maxim 80 de caractere reprezentând
fract, ia zecimală. Caracterele din acest s, ir pot fi: cifre, eventual virgula zecimală ’,’ s, i parantezele
’(’ respectiv ’)’.
Date de ies, ire
Fis, ierul parola.out va cont, ine două linii. Pe prima linie se va scrie numărătorul fract, iei, pe a
doua linie se va scrie numitorul fract, iei.
Restrict, ii s, i precizări

• Partea ı̂ntreagă a unei fract, ii zecimale este formată din cel put, in o cifră
• S, irul citit poate cont, ine cel mult 77 de cifre s, i reprezintă o fract, ie zecimală corectă

Exemplu
parola.in parola.out Explicat, ie
5,67 567
567
100 5, 67 =
100

Timp maxim de execut, ie: 1 secundă/test.


Memorie totală disponibilă 5 MB.
Grat, iela Ghiordunescu, Pites, ti (Dan Barbilian, 2018)

I 8 (joc). De Crăciun, Ionel a primit cadou un joc de potrivire imagini. Toate piesele au aceleas, i
dimensiuni, forma pătratică s, i sunt realizate din lemn de cea mai bună calitate. O piesă se
ı̂mparte ı̂n N × N celule pătrate, identice ca dimensiuni, unele celule fiind pline, altele fiind
decupate. Pentru a dezvolta abilităt, ile de coordonare mână-ochi, copilul trebuie să as, eze piesele
pe un suport tot pătratic de dimensiune N × N . Piesele pot fi as, ezate cu oricare din cele două
fet, e deasupra. Astfel ele pot fi rotite atât pe orizontală cât s, i pe verticală. Anumite celule de
pe suport sunt prevăzute cu tije de lemn verticale, prin care se va ı̂ncerca plasarea pieselor.
PROBLEME DE INFORMATICĂ PENTRU CONCURSURI 159

Dimensiunile tijelor permit as, ezarea chiar s, i a tuturor pieselor, dacă acestea sunt decupate ı̂n
celulele corespunzătoare tijelor. Fiecare piesă este prevăzută cu elemente speciale de sust, inere,
astfel că dacă există N × N tije pe toată suprafat, a suportului, iar o piesă este decupată ı̂n toate
celulele pătrate, atunci piesa se poate as, eza pe suport. Nu este permisă as, ezarea unei piese care
are mai multe celule decupate decât numărul tijelor. Cum Ionel este prea mic, neavând suficientă
ı̂ndemânare de a manevra piesele pentru a le introduce prin tijele de pe suport, ajutat, i-l să le
as, eze, dacă este posibil.
Cerint, ă
Scriet, i un program care determină care dintre piese pot fi as, ezate pe suport.
Date de intrare
Fis, ierul de intrare joc.in cont, ine pe prima linie două numere naturale N s, i P despărt, ite
printr-un spat, iu. N este dimensiunea suportului, precum s, i a pieselor, iar P reprezintă numărul
pieselor. Urmează P + 1 blocuri de câte N linii fiecare. Primul bloc de N linii codifică suportul.
Valoarea 1 apare dacă suportul este prevăzut cu tijă verticală, iar valoarea 0 ı̂n caz contrar.
Următoarele P blocuri de câte N linii codifică fiecare câte o piesă. Pe fiecare linie sunt câte
N valori ı̂ntregi, despărt, ite printr-un singur spat, iu, valoarea 1 codifică partea decupată, iar
valoarea 0 partea care nu este decupată.
Date de ies, ire
În fis, ierul de ies, ire joc.out se vor scrie P linii, câte o valoare pe linie. Pe linia i se va scrie
valoarea 1 dacă piesa i poate fi as, ezată pe suport astfel ı̂ncât tijele să pătrundă prin port, iunile
decupate ale piesei s, i valoarea 0 ı̂n caz contrar.
Restrict, ii s, i precizări

• 1 < N, C ≤ 50

Exemplu
joc.in joc.out Explicat, ie
32 1 Piesa 1 se potrives, te perfect suportu-
010 0 lui, dacă se rotes, te ı̂n sens trigonome-
001 tric cu 90 de grade.
100 Piesa 2 nu se potrives, te suportului, in-
100 diferent de modul ı̂n care se ı̂ncearcă
001 as, ezarea sa prin tijele aflate pe suport.
010
001
001
010
Timp maxim de execut, ie: 1 secundă/test.
Memorie totală disponibilă 5 MB.
Cristina Constantinescu, Pites, ti (Dan Barbilian, 2018)
160 PROBLEME DE INFORMATICĂ PENTRU CONCURSURI

I 9 (descifrare). Taberele sunt de fiecare dată pline de activităt, i care rămân mult timp ı̂n
mint, ile copiilor. Timpul petrecut ı̂n compania altor copii oferă ocazia de a socializa, de a
face prieteni noi s, i de a participa la diverse activităt, i. Secretul unei tabere reus, ite este să fie
organizate mai multe tipuri de activităt, i. În vacant, a de iarnă, mai mult, i elevi ai clasei a IV-a
de la s, coala Amicii Istet, i au mers ı̂mpreună ı̂n tabără. Unul dintre jocurile care le-a trezit
interes a fost cel care le oferea posibilitatea să descifreze un mesaj. Astfel, ei aveau la dispozit, ie
un dict, ionar format din cuvinte uzuale precum s, i o ı̂ns, iruire de cifre, fără cifra 0. Echipa
câs, tigătoare trebuie să determine prima mesajul ce se ascunde ı̂n spatele cifrelor, constând
dintr-o succesiune de cuvinte separate prin câte un spat, iu.
Fiecare cifră are asociată un grup de litere. Cifra 2 este asociată literelor A, B sau C, cifra 3
este asociată literelor D, E s, i F, cifra 4 este asociată literelor G, H s, i I, cifra 5 este asociată
literelor J, K s, i L, cifra 6 este asociată literelor M, N s, i O, cifra 7 este asociată literelor P, Q, R
s, i S, cifra 8 este asociată literelor T, U s, i V, cifra 9 este asociată literelor W, X, Y s, i Z, iar cifra
1 este asociată tastei Spat, iu. Prin urmare, dacă mesajul este format din cifrele 2 8 1, textul
ascuns poate fi A T Spat, iu sau A U Spat, iu, etc, cuvintele corespunzătoare s, irurilor de cifre
putând fi: AT, AU, etc.
Pentru un posibil text ascuns obt, inut prin descifrarea consecutivă a cifrelor mesajului, se
opresc doar cuvintele care există ı̂n dictionar, iar fiecare literă a unui cuvânt care nu este ı̂n
dictionar va fi ı̂nlocuită cu un caracter *. Se vor verifica toate posibilităt, ile de formare a unui
cuvânt conform regulilor de mai sus. Dacă există mai multe cuvinte care corespund unei anumite
secvent, e de cifre, va fi afis, at primul.
Cerint, ă
Scriet, i un program care să descifreze mesajul transmis.
Date de intrare
Fis, ierul de intrare desc.in cont, ine:
M - numărul de cuvinte din dict, ionar;
C1
C2
...
CM - C1 , C2 , . . . , CM reprezintă cuvintele din dict, ionar;
N - numărul de cifre din mesajul ce urmează a fi decriptat;
cif1 cif2 . . . cifN - cifrele ce urmează a fi decriptate.
Date de ies, ire
Fis, ierul de iesire desc.out cont, ine pe o singură linie mesajul generat respectând regulile date.
Restrict, ii s, i precizări
• 1 ≤ M, N ≤ 100
• cifi ∈ {1, 2, . . . , 9}, oricare i ∈ {1, 2, . . . , N }
PROBLEME DE INFORMATICĂ PENTRU CONCURSURI 161

• Cuvintele C1 , C2 , . . . , CM sunt ordonate alfabetic s, i sunt scrise cu majuscule, formate doar


din litere ale alfabetului englez
• Lungimea oricărui cuvânt este de maxim 100 de litere
Exemplu
desc.in desc.out Explicat, ie
5 * ** ACASA Pentru cele 5 cuvinte existente ı̂n
ACASA dict, ionar, mesajul ascuns ı̂n spatele
ANA cifrelor se află ı̂n fis, ierul de ies, ire.
LA
MERGE
SCOALA
11
21231222721
Timp maxim de execut, ie: 1 secundă/test.
Memorie totală disponibilă 5 MB.
Cristina Constantinescu, Pites, ti (Dan Barbilian, 2018)
I 10 (cecuatii). Ionică s, i Tică sunt buni prieteni, pasionat, i de două lucruri: matematică s, i
informatică. Aces, tia participă la un concurs de propunători de probleme pe echipe. Pentru a
avea succes, Tică vrea să propună o problemă cu ecuat, ii, iar Ionică o problemă cu mult, imi. După
consultări repetate se decid să participe la concurs cu o problemă care foloses, te atât ecuat, ii cât s, i
mult, imi. În această problemă se dau N ecuat, ii de forma Ai x + Bi y + Ci z = Di , 1 ≤ i ≤ N s, i se
notează cu Si mult, imea {x + y + z|Ai x + Bi y + Ci z = Di , x, y, z numere naturale}, 1 ≤ i ≤ N .
Cerint, a problemei este determinarea a două numere naturale N1 s, i N2 , unde N1 = M ax+F M ax,
respectiv N2 = card(S1 ∪ S2 . . . ∪ SN ). M ax este cel mai mare număr dintre card(Si ), 1 ≤ i ≤ N ,
iar F M ax este de câte ori apare M ax printre card(S1 ), card(S2 ), . . . , card(SN ).
Cerint, ă
Se cunosc N s, i numerele A1 , B1 , C1 , D1 , A2 , B2 , C2 , D2 , . . . , AN , BN , CN , DN s, i se cere să se
determine numerele N1 s, i N2 .
Date de intrare
Din fis, ierul cecuatii.in se citesc: numărul natural p de pe prima linie, ce poate avea două
valori: 1 sau 2, corespunzătoare uneia din cele două cerint, e, adică 1 pentru determinarea lui N1
s, i 2 pentru determinarea lui N2 ; numărul N de pe a doua linie s, i, de pe următoarele N linii,
câte patru numere naturale separate prin câte un spat, iu reprezentând coeficient, ii ecuat, iilor:
Ai , Bi , Ci , Di , 1 ≤ i ≤ N .
Date de ies, ire
În fis, ierul cecuatii.out se va scrie numărul N1 , dacă p = 1 s, i respectiv numărul N2 , dacă
p = 2, cu semnificat, ia din enunt, .
Restrict, ii s, i precizări
• 1 < N ≤ 1000, număr natural
162 PROBLEME DE INFORMATICĂ PENTRU CONCURSURI

• Ai , Bi , Ci , Di sunt numere naturale nenule mai mici strict decât 1000, 1 ≤ i ≤ N


• Pentru o mult, ime M , card(M ) reprezintă numărul de elemente ale mult, imii M
• Pentru mult, imile M s, i P , notăm cu M ∪ P reuniunea dintre M s, i P

Exemple
cecuatii.in cecuatii.out Explicat, ie
1 3 p = 1 s, i deci afis, ăm N1 . Prima ecuat, ie este
2 x + y + z = 1, iar a doua x + 2y + z = 2.
1111 Obt, inem solut, iile (1,0,0), (0,1,0), (0,0,1)
1212 pentru prima ecuat, ie s, i (1,0,1), (0,1,0),
(0,0,2), (2,0,0) pentru a doua ecuat, ie. S1 =
{1}, S2 = {1, 2}, M ax = 2, F M ax = 1,
deci N1 = 3.
cecuatii.in cecuatii.out Explicat, ie
2 2 p = 2 s, i deci afis, ăm N2 . Prima ecuat, ie este
2 x + y + z = 1, iar a doua x + 2y + z = 2.
1111 Obt, inem solut, iile (1,0,0), (0,1,0), (0,0,1)
1212 pentru prima ecuat, ie s, i (1,0,1), (0,1,0),
(0,0,2), (2,0,0) pentru a doua ecuat, ie. S1 =
{1}, S2 = {1, 2}, S1 ∪ S2 = {1, 2}, deci
N2 = 2.
Timp maxim de execut, ie: 0.04 secunde/test.

Memorie totală disponibilă 4 MB.


Doru Anastasiu Popescu, Pites, ti (Micul Gates, 2017)

Clasele a XI-a s, i a XII-a

I 11 (arici). Într-o livadă de formă dreptunghiulară există doi arici care doresc să ajungă la
căsut, a aricilor. În livadă sunt m rânduri de pomi, iar pe fiecare din cele m rânduri există n
pomi la o distant, ă de 1 dam. La baza fiecărui pom există un număr de mere (un număr natural
de exact două cifre).

Fiecare dintre cei doi arici dores, te să ajungă cât mai repede la căsut, a aricilor s, i astfel se
organizează o competit, ie ı̂ntre ei, câs, tigătorul fiind ariciul care ajunge pe drumul cel mai scurt,
drum măsurat ı̂n distant, a parcursă de arici ı̂n dam de la pozit, ia lui din livadă la căsut, a aricilor.
Fiecare din cei doi arici are un cod de acces, un număr natural de exact 5 cifre nenule. Pe baza
acestui cod aricii au sau nu acces la merele căzute din pom. Un arici are acces la merele de la
baza unui pom dacă numărul de mere căzute are cel put, in o cifră care se găses, te ı̂n codul de
acces al ariciului. Fiecare arici se găses, te init, ial lângă un anumit pom din livadă. Aricii se pot
PROBLEME DE INFORMATICĂ PENTRU CONCURSURI 163

deplasa prin livadă, de la un pom la fiecare din cei cel mult 4 pomi vecini la care au acces, doar
pe direct, iile paralele cu laturile livezii.

Cerint, ă
Cunoscând dimensiunile livezii m s, i n, numărul de mere căzute din fiecare din cei m · n pomi,
coordonatele L, C unde se află căsut, a aricilor, coordonatele celor doi arici s, i codurile lor de
acces, determinat, i:

1. numărul X de pomi la care au acces ambii arici;

2. numărul N r al ariciului câs, tigător (1 sau 2, corespunzător cu numărul de ordine al


ariciului), distant, a minimă Dmin pe care o parcurge ariciul câs, tigător pentru a ajunge la căsut, a
aricilor s, i numărul N rM de pomi de unde ar putea pleca ariciul câs, tigător astfel ı̂ncât să
parcurgă aceeas, i distant, ă minimă până la căsut, a aricilor.

Date de intrare
Fis, ierul de intrare arici.in cont, ine:

- pe prima linie un număr natural p; pentru toate testele de intrare, numărul p poate avea
doar valoarea 1 sau 2;

- pe a doua linie două numere naturale nenule m, n, separate prin câte un spat, iu, cu
semnificat, iile din enunt, ;

- pe linia a treia două numere naturale nenule L, C, separate prin câte un spat, iu, cu
semnificat, iile din enunt, ;

- pe liniile a patra s, i a cincea câte trei numere naturale reprezentând coordonatele pomilor
unde se găsesc init, ial cei doi arici (linia, coloana) s, i codul de acces al aricilor: x1 , y1 , c1 , respectiv
x2 , y2 , c2 ;

- pe următoarele m linii câte n numere naturale de exact două cifre, separate prin câte un
spat, iu, reprezentând numărul de mere căzute din fiecare pom al livezii.

Date de ies, ire


Dacă valoarea lui p este 1, se va rezolva numai punctul 1) din cerint, ă.

În acest caz, ı̂n fis, ierul de ies, ire arici.out se va scrie un singur număr natural, X, repre-
zentând numărul total de pomi la care au acces ambii arici.

Dacă valoarea lui p este 2, se va rezolva numai punctul 2) din cerint, ă.

În acest caz, fis, ierul de ies, ire arici.out va cont, ine numerele naturale N r, Dmin s, i N rM ,
ı̂n această ordine, separate prin câte un spat, iu.

Restrict, ii s, i precizări

• 2 ≤ m, n, L, C, x1 , y1 , x2 , y2 ≤ 300
• 11111 ≤ c1 , c2 ≤ 99999
• 10 ≤ numărul de mere căzute din fiecare pom ≤ 99
164 PROBLEME DE INFORMATICĂ PENTRU CONCURSURI

• Pentru toate testele problemei există solut, ie, adică există cel put, in un arici care poate
ajunge la căsut, a aricilor
• Dacă cei doi arici parcurg aceeas, i distant, ă până la căsut, a aricilor, este considerat câs, tigător
primul arici
• Mere căzute din pom se pot găsi s, i ı̂n locuri la care aricii nu au acces
• Pentru rezolvarea corectă a primei cerint, e se acordă 20% din punctaj, iar pentru cerint, a a
doua 80% din punctaj
• Pentru primele 30% dintre testele care verifică cerint, a 2) avem m ≤ 20 s, i n ≤ 20
• Pentru determinarea corectă a numerelor N r s, i Dmin se acordă 80% din punctajul testului,
iar pentru determinarea corectă a numărului N rM se acordă 20% din punctaj
Exemple
arici.in arici.out Explicat, ie
1 24 p=1
56 Există 24 de pomi ı̂n livadă la care au
35 acces ambii arici, s, i anume tot, i pomii cu
1 2 12351 except, ia celor 6 pomi care au numărul de
5 1 46123 mere căzute 46, 78, 89 sau 96.
15 12 37 12 78 19
16 17 18 19 25 20
21 25 26 89 25 25
24 78 29 31 24 89
23 25 46 25 96 25
arici.in arici.out Explicat, ie
2 153 p=2
56 Ariciul cu numărul de ordine 1 se află la
35 pomul de coordonate (1,2) s, i poate ajunge
1 2 12351 la căsut, a aricilor pe traseul:
5 1 46123 (1,2),(1,3),(1,4),(2,4),(2,5),(3,5),
15 12 37 12 78 19 parcurgând o distant, ă de 5 dam.
16 17 18 19 25 20 De asemenea mai există s, i alt, i doi pomi de
21 25 26 89 25 25 la care ar putea pleca ariciul 1 s, i ar parcurge
24 78 29 31 24 89 aceeas, i distant, ă de 5 dam până la căsut, a
23 25 46 25 96 25 aricilor, s, i anume pomii de coordonate (2,1)
s, i (3,2).
Ariciul 2 poate ajunge la căsut, a aricilor
parcurgând o distant, ă de 6 dam, pe traseul:
(5,1),(5,2),(5,3),(5,4),(5,5),(4,5),(3,5).
Timp maxim de execut, ie: 0.5 secunde/test.
Memorie totală disponibilă 8 MB, din care 2 MB pentru stivă.
Adriana Oprea, Pites, ti
PROBLEME DE INFORMATICĂ PENTRU CONCURSURI 165

I 12 (cuvinte). Se consideră două cuvinte formate numai din litere mici ale alfabetului englez,
notate cu c1 si c2 . Pentru aceste cuvinte se dores, te să se rezolve două cerint, e.

Prima cerint, ă este legată de verificarea condit, iei de subs, ir de caractere a lui c2 ı̂n c1 , adică
dacă se pot s, terge caractere din c1 astfel ı̂ncât să se obt, ină c2 . Dacă c2 este subs, ir ı̂n c1 se dores, te
s, irul de pozit, ii pe care se află, fără spat, ii. Numărul obt, inut se va numi P OZ. De exemplu
pentru c1 =ababcd s, i c2 =abd, răspunsul este afirmativ, P OZ putand fi 126 sau 346. Pozit, iile
ı̂n cuvinte sunt 1, 2, 3, . . . .

A doua cerint, ă este să se construiască un nou cuvânt c3 cu litere distincte sau cu o singură
literă care să se repete de două ori, astfel ı̂ncât să respecte regulile:

1. literele din c3 trebuie să apară ı̂n c1 sau c2 ;


2. litera care se repetă trebuie să apară ı̂n total de cel put, in două ori ı̂n c1 s, i c2 .

Cerint, ă
Cunoscând cele două cuvinte c1 s, i c2 se cere:

1. să se verifice dacă c2 este subcuvânt ı̂n c1 s, i să se determine cel mai mic număr P OZ, ı̂n
caz afirmativ, altfel se va afis, a cifra 0;

2. numărul de cuvinte c3 distincte ce se pot construi cu regulile din enunt, , de la a doua


cerint, ă.

Date de intrare
Fis, ierul de intrare cuvinte.in cont, ine pe prima linie un număr natural p. Pentru toate testele
de intrare, numărul p poate avea doar valoarea 1 sau 2.

Pe linia a doua se află cuvântul c1 , iar pe linia a treia cuvântul c2 .

Date de ies, ire


Dacă valoarea lui p este 1, se va rezolva numai punctul 1) din cerint, ă.

În acest caz, ı̂n fis, ierul de ies, ire cuvinte.out se va scrie rezultatul de la cerint, a 1, adică 0
sau cel mai mic P OZ cu semnificat, ia din enut, .

Dacă valoarea lui p este 2, se va rezolva numai punctul 2) din cerint, ă.

În acest caz, ı̂n fis, ierul de ies, ire cuvinte.out se va scrie numărul de la cerint, a 2, modulo
9901.

Restrict, ii s, i precizări

• 1 ≤ număr caractere din c1 ≤ 250,


• 1 ≤ număr caractere din c2 ≤ 250
• Pentru rezolvarea corectă a cerint, ei 1 se acordă 20% din punctaj
• k modulo p reprezintă restul ı̂mpărt, irii ı̂ntregi a lui k la p
166 PROBLEME DE INFORMATICĂ PENTRU CONCURSURI

Exemple
cuvinte.in cuvinte.out Explicat, ie
1 126 p=1
ababcd P OZ poate fi 126 sau 346, iar
abd 126 este cel mai mic. Pen-
tru P OZ=126, ı̂nseamnă că se
păstrează din c1 literele de pe
pozit, iile 1, 2, 6, se s, terg cele de
pe pozit, iile 3, 4, 5 s, i se obt, ine c2 .
cuvinte.in cuvinte.out Explicat, ie
2 29 p=2
adcd Cuvintele ce se pot forma cu res-
ad pectarea regulilor din enunt, sunt:
a, d, c,
ad, da, ac, ca, dc, cd, aa, dd,
dac, dca, adc, acd, cda, cad,
daa, ada, aad, caa, aca, aac,
cdd, dcd, ddc, add, dad, dda.
Timp maxim de execut, ie: 1 secundă/test.
Memorie totală disponibilă 4 MB, din care 2 MB pentru stivă.
Doru Anastasiu Popescu, Pites, ti (Info-Oltenia, 2016)
I 13 (litere). În fiecare pătrăt, el al unei table de s, ah de dimensiune m × n se află câte o piesă
pe care este scrisă o literă mică din alfabetul englez. Georgică dores, te să-s, i verifice cunos, tint, ele
de informatică folosind tabla de s, ah s, i piesele. Astfel el dores, te să determine:
1. numărul de cuvinte N r de lungime k ce se pot forma, cu litere distincte s, i ı̂n ordine
alfabetică, folosind piese de pe tabla de s, ah;
2. lungimea minimă L a unui cuvânt format cu piese vecine de pe tabla de s, ah ce cont, in
numai vocale, astfel ı̂ncât să cont, ină o piesă de pe prima linie s, i o piesă de pe ultima linie;
două piese sunt vecine, dacă au o latură comună.
Cerint, ă
Cunoscând numerele m, n, k s, i piesele de pe tabla de s, ah se cere:
1. numărul N r cu semnificat, ia de mai sus;
2. numărul L cu semnificat, ia de mai sus.
Date de intrare
Fis, ierul de intrare litere.in cont, ine pe prima linie un număr natural p. Pentru toate testele
de intrare, numărul p poate avea doar valoarea 1 sau 2.
Pe linia a doua se află m, n s, i k, separate prin câte un spat, iu, iar pe următoarele m linii
literele tabloului, fără spat, iu ı̂ntre ele.
PROBLEME DE INFORMATICĂ PENTRU CONCURSURI 167

Date de ies, ire


Dacă valoarea lui p este 1, se va rezolva numai punctul 1) din cerint, ă.
În acest caz, ı̂n fis, ierul de ies, ire litere.out se va scrie numărul N r.
Dacă valoarea lui p este 2, se va rezolva numai punctul 2) din cerint, ă.
În acest caz, ı̂n fis, ierul de ies, ire litere.out se va scrie numărul L.
Restrict, ii s, i precizări

• 1 ≤ m, n ≤ 100

• 1 ≤ k ≤ 26

• Pentru rezolvarea corectă a cerint, ei 1 se acordă 40% din punctaj

Exemple
litere.in litere.out Explicat, ie
1 3 p=1
342 Se pot forma 3 cuvinte:
abab ab, ae, be
eaaa
bbbe
litere.in litere.out Explicat, ie
2 4 p=2
342 Se pot forma două cuvinte
abab (ı̂ngros, ate mai jos):
eaaa abab
bbbe eaaa
bbbe
respectiv:
abab
eaaa
bbbe
Cel mai scurt, aaae, are 4 litere.

Timp maxim de execut, ie: 1 secundă/test.


Memorie totală disponibilă 4 MB, din care 2 MB pentru stivă.

Doru Anastasiu Popescu, Pites, ti (Info-Oltenia, 2016)

I 14 (veverit, a). Într-un parc din oras, se află n pomi, pentru care se cunosc coordonatele
carteziene pe harta oras, ului. Pomii sunt numerotat, i cu 1, 2, . . . , n. În parc se mai află o veverit, ă
foarte isteat, ă, dar care poate sări din pom ı̂n pom doar dacă distant, a dintre aces, tia este cel mult
168 PROBLEME DE INFORMATICĂ PENTRU CONCURSURI

egală cu k. În cazul ı̂n care veverit, a vrea să se deplaseze de la un pom la altul mai ı̂ndepărtat,
aceasta trebuie să ris, te să fie alergată de câini.

Veverit, a dores, te să afle:

- de câte ori trebuie să ris, te să fie alergată de câini pentru a se deplasa ı̂n fiecare pom;

- distant, a cea mai mică pe care trebuie să o parcurgă pe jos pentru a se deplasa ı̂n tot, i pomii
din parc.

Cerint, ă
Cunoscând numerele n, k s, i coordonatele pomilor, determinat, i:

1. de câte ori trebuie să ris, te veverit, a să fie alergată de câini pentru a se deplasa ı̂n fiecare
pom;

2. distant, a cea mai mică pe care trebuie să o parcurgă pe jos veverit, a pentru a se deplasa ı̂n
tot, i pomii din parc.

Date de intrare
Fis, ierul de intrare veverita.in cont, ine pe prima linie un număr natural p. Pentru toate testele
de intrare, numărul p poate avea doar valoarea 1 sau 2.

Pe linia a doua se află n si k, separate printr-un spat, iu, iar pe următoarele n linii perechi de
numere naturale de forma x y, ce reprezintă coordonatele pomilor.

Date de ies, ire


Dacă valoarea lui p este 1, se va rezolva numai punctul 1) din cerint, ă.

În acest caz, ı̂n fis, ierul de ies, ire veverita.out se va scrie un singur număr natural repre-
zentând de câte ori trebuie să ris, te să fie alergată de câini veverit, a pentru a se deplasa ı̂n fiecare
pom.

Dacă valoarea lui p este 2, se va rezolva numai punctul 2) din cerint, ă.

În acest caz, ı̂n fis, ierul de ies, ire veverita.out se va scrie distant, a cea mai mică pe care
trebuie să o parcurgă pe jos veverit, a pentru a se deplasa ı̂n tot, i pomii din parc.

Restrict, ii s, i precizări

• 1 ≤ n ≤ 500
• Coordonatele punctelor sunt numere naturale mai mici sau egale cu 1000000
• Pentru rezolvarea corectă a cerint, ei 1 se acordă 20% din punctaj
• 20% din testele cu p = 2 au n ≤ 20
• Pentru distant, a dintre două puncte se va folosi distant, a Manhattan, adică distant, a dintre
P (x1 , y1 ) s, i Q(x2 , y2 ) este
|x1 − x2 | + |y1 − y2 |
PROBLEME DE INFORMATICĂ PENTRU CONCURSURI 169

Exemple
veverita.in veverita.out Explicat, ie
1 1 p=1
58 Veverit, a poate să sară ı̂ntre pomii:
5 12 1 s, i 3, 3 s, i 5, 2 s, i 4.
100 14 Trebuie să ris, te să meargă pe jos
28 o singură dată ı̂ntre doi pomi.
100 8
10 8
veverita.in veverita.out Explicat, ie
2 90 p=2
58 Veverit, a poate să sară ı̂ntre pomii:
5 12 1 s, i 3, 3 s, i 5, 2 s, i 4.
100 14 Trebuie să meargă pe jos ı̂ntre po-
28 mii 4 s, i 5 pe o distant, ă egală cu
100 8 90.
10 8

Timp maxim de execut, ie: 1 secundă/test.

Memorie totală disponibilă 4 MB, din care 2 MB pentru stivă.

Doru Anastasiu Popescu, Pites, ti, Gabriel Nicolae, Bucures, ti (Info-Oltenia, 2016)

I 15 (case). Cu mult timp ı̂n urmă, mai multe case s-au construit fără un plan urbanistic,
formând o comună. În zilele noastre, primarul comunei vrea să ı̂mpartă comuna ı̂n sate pentru
a gestiona mai us, or problemele comunităt, ii. Pentru a simplifica lucrurile, primarul dores, te
ca satele să aibă suprafat, a pătratică cu laturile paralele cu axele hărt, ii. Primarul cunoas, te
coordonatele caselor, casele fiind numerotate cu 1, 2, . . . , n, coordonatele colt, urilor din stânga-jos
s, i laturile suprafet, elor celor k sate. Satele sunt numerotate cu 1, 2, . . . , k. Suprafet, ele satelor
sunt disjuncte. Din păcate se poate ı̂ntâmpla ca unele case să fie ı̂n afara satelor.

Primarul dores, te să acceseze fonduri europene pentru un sistem de canalizare pe care să-l
dezvolte ulterior. Pentru acest lucru ı̂ntocmes, te un plan fiecărui sat, ı̂n care trasează segmente
ı̂ntre anumite case astfel ı̂ncât să se formeze un poligon convex; pe acolo va trece sistemul de
canalizare, astfel ı̂ncât toate casele satului care nu sunt legate prin aceste segmente să fie ı̂n
interiorul poligonului.

Cerint, ă
Cunoscând numerele n, k, coordonatele caselor s, i datele suprafet, elor satelor (coordonatele
colt, ului stânga-jos s, i lungimea suprafet, ei), determinat, i:

1. numărul de case care nu fac parte din niciun sat;

2. numărul de case ce vor face parte din sistemul de canalizare al fiecărui sat i, i = 1, 2, . . . , k.
170 PROBLEME DE INFORMATICĂ PENTRU CONCURSURI

Date de intrare
Fis, ierul de intrare case.in cont, ine pe prima linie un număr natural p. Pentru toate testele de
intrare, numărul p poate avea doar valoarea 1 sau 2.

Pe linia a doua se află n s, i k, pe linia a treia se află n perechi de numere, una după alta,
separate prin câte un spat, iu, perechi de forma abscisă ordonată, reprezentând coordonatele
caselor, iar pe linia a patra se află k triplete de forma abscisă ordonată lungime, separate ı̂ntre
ele prin spat, ii, reprezentând coordonatele colt, ului din stânga-jos s, i lungimea suprafet, ei satelor.

Date de ies, ire


Dacă valoarea lui p este 1, se va rezolva numai punctul 1) din cerint, ă.

În acest caz, ı̂n fis, ierul de ies, ire case.out se va scrie un singur număr natural reprezentând
numărul de case care nu fac parte din niciun sat.

Dacă valoarea lui p este 2, se va rezolva numai punctul 2) din cerint, ă.

În acest caz, ı̂n fis, ierul de ies, ire case.out se vor scrie, separate prin câte un spat, iu, numerele
de case ce vor face parte din sistemul de canalizare al fiecărui sat i, i = 1, 2, . . . , k.

Restrict, ii s, i precizări

• 1 ≤ n ≤ 1000
• 1 ≤ k ≤ 10
• Pentru rezolvarea corectă a cerint, ei 1 se acordă 20% din punctaj
• Coordonatele caselor sunt numere naturale mai mici sau egale cu 109
• O casă dacă se găses, te pe un segment al sistemului de canalizare ı̂nseamnă că face parte
din acest sistem
• O casă aflată chiar pe frontiera satului se presupune că face parte din sat
• Satele sunt disjuncte, adică nu au interior sau frontieră comună

Exemple
case.in case.out Explicat, ie
1 1 p=1
71 Avem casele cu coordonatele:
6 10 8 0 20 0 2 5 4 5 4 0 6 0 C1 (6, 10), C2 (8, 0), C3 (20, 0),
2 0 10 C4 (2, 5), C5 (4, 5), C6 (4, 0), C7 (6, 0)
s, i un singur sat cu suprafat, a
pătratică având colt, ul din stânga-
jos de coordonate (2, 0) s, i latura de
lungime 10.
C3 este singura casă care nu face
parte din sat.
PROBLEME DE INFORMATICĂ PENTRU CONCURSURI 171

case.in case.out Explicat, ie


2 5 p=2
71 Avem casele cu coordonatele:
6 10 8 0 20 0 2 5 4 5 4 0 6 0 C1 (6, 10), C2 (8, 0), C3 (20, 0),
2 0 10 C4 (2, 5), C5 (4, 5), C6 (4, 0), C7 (6, 0)
s, i un singur sat cu suprafat, a
pătratică având colt, ul din stânga-
jos de coordonate (2, 0) s, i latura de
lungime 10.
Casele C1 , C2 , C4 , C6 , C7 vor face
parte din sistemul de canalizare.
Timp maxim de execut, ie: 1 secundă/test.
Memorie totală disponibilă 4 MB, din care 2 MB pentru stivă.
Doru Anastasiu Popescu, Pites, ti, Gabriel Nicolae, Bucures, ti (Info-Oltenia, 2016)
ISTORIOARE DIN LUMEA MATEMATICII S, I A
INFORMATICII

Formula de alungare a lui L. Euler

1
Stelian Corneliu Andronescu

Marele matematician Leonhard Euler ı̂s, i câs, tigase reputat, ia de a rezolva orice problemă cu
care era confruntat.
În timpul retragerii sale la curtea Ecaterinei cea Mare, el l-a cunoscut pe marele filosof
francez Denis Diderot. Diderot era un ateu convins s, i avea să-s, i petreacă viat, a convertindu-i pe
rus, i la ateism. Înfuriată, Ecaterina i-a cerut lui Euler să pună capăt eforturilor acestui francez
fără Dumnezeu. Euler s-a gândit o vreme la această problemă s, i a pretins că are o dovadă
algebrică a existent, ei lui Dumnezeu. Ecaterina cea Mare i-a invitat pe Euler s, i pe Diderot la
palat s, i s, i-a adunat curtenii spre a asculta dezbaterea teologică. Euler a apărut ı̂n fat, a publicului
s, i a spus:
a + bn
,,Domnule, = x, deci Dumnezeu există. Răspundet, i!”
n
Fără a ı̂nt, elege mare lucru din algebră, Diderot n-a fost ı̂n stare să-i replice lui Euler s, i a
rămas fără grai. Umilit, a părăsit St. Petersburg-ul s, i s-a ı̂ntors la Paris.
L. Euler (15.04.1707 − 18.09.1783) a fost cel mai prolific s, i unul dintre cei mai mari ma-
tematicieni ai tuturor timpurilor, dominând cu autoritate matematica secolului al XVIII-lea.
Marele savant, a reunit o intuit, ie formidabilă cu o imaginat, ie creatoare except, ională, o memorie
inegalabilă, abilităt, i de calcul extraordinare s, i o putere de muncă fantastică.
Opera lui L. Euler a impulsionat puternic ı̂ntreaga dezvoltare a matematicii.

Bibliografie

[1] S. Singh, Marea Teoremă a lui Fermat, Ed. Humanitas, Bucures, ti, 1998.

[2] P. Minut, , 300 de ani de la nas, terea lui Leonhard Euler (1707-1783), Recreat, ii Matematice,
nr. 1/2007.

1
Lect. univ. dr., Universitatea din Pites, ti, corneliuandronescu@yahoo.com

172
Revistă sponsorizată de
ROWEB Development SRL
s, i
OSF Global Services SRL

S-ar putea să vă placă și